Chapter 6. Types and Pain Flashcards Preview

Pain Medicine Board Review > Chapter 6. Types and Pain > Flashcards

Flashcards in Chapter 6. Types and Pain Deck (270)
Loading flashcards...
1
Q
  1. A45-year-old patient with metastatic breast carcinoma
    is prescribed 30 mg of sustained-release
    morphine (MS Contin) twice a day and one
    15-mg tablet of immediate-release morphine
    (MSIR) every 6 hours as needed for breakthrough
    pain. On her routine follow-up visit she
    reports that she routinely uses MSIR four times
    a day with satisfactory pain control on most days
    and no major side effects. What would be your
    best course of action in this situation?
    (A) Prescriptions should be left unchanged
    (B) MS Contin should be changed to 40 mg
    of OxyContin twice a day and 5 mg of
    oxycodone every 6 hours as needed for
    breakthrough pain
    (C) Fentanyl patch of 25 μg/h should
    replace MS Contin with 15 mg of MSIR
    every 6 hours as needed for breakthrough
    pain
    (D) MS Contin should be increased to 60 mg
    twice a day with MSIR 15 mg every
    6 hours as needed for breakthrough pain
    (E) MS Contin should be increased to
    60 mg twice a day, and MSIR should be
    discontinued
A
  1. (D) If a patient routinely uses breakthrough
    medications, the daily total amount should be
    converted to a sustained-release dose and
    added to the current maintenance dose.
2
Q
333. Approximately in what percentage of patients
with malignancies does pain unrelated to cancer
occur?
(A) Less than 2%
(B) 3%
(C) 7.5%
(D) 11%
(E) 25%
A
  1. (B) Approximately 3% of pain syndromes in
    cancer patients are unrelated to the underlying
    malignancy or cancer treatment. Most commonly,
    pain is caused by degenerative disc disease,
    arthritis, fibromyalgia, or migraine and
    has often predated the diagnosis of cancer.
3
Q
  1. There is a significant incidence of neuropathic
    pain in a cancer patient with brachial plexopathy.
    The etiology of the brachial plexopathy in such
    a patient may be caused by direct tumor infiltration
    or radiation fibrosis. Electrophysiologic
    evaluation with nerve conduction velocity (NCV)
    study and electromyography (EMG) helps to
    distinguish between the two etiologies. Which of
    the following findings of NCV/EMG is the most
    helpful to differentiate between the direct tumor
    infiltration and the radiation fibrosis etiologies of
    brachial plexopathy?
    (A) Segmental nerve conduction slowing
    (B) Myokymia
    (C) Fibrillation potentials
    (D) Positive sharp waves
    (E) Decreased amplitude of the compound
    muscle action potential (CMAP)
A
  1. (B) Segmental nerve conduction slowing, fibrillation
    potentials, positive sharp waves, and
    decreased amplitude CMAPs are all helpful in
    determining the presence of brachial plexopathy
    in general. Myokymia is present in 63% of
    patients with radiation fibrosis induced
    brachial plexopathy. Brachial plexopathy
    caused by direct tumor infiltration has a low
    incidence of myokymia. Myokymia is a continuous
    but brief involuntary muscle twitching
    that gives the appearance of wormlike rippling
    of the muscle. It
4
Q
335. If bony metastases are present, which primary
cancer location has the best 5-year survival
prognosis?
(A) Myeloma
(B) Breast
(C) Prostate
(D) Thyroid
(E) Kidney
A
  1. (D) A5-year survival for a cancer patient with
    documented skeletal metastases varies widely
    depending on location of the primary tumor:
    myeloma—10%; breast—20%; prostate—25%;
    lung—less than 5%; kidney—10%; thyroid—
    40%; melanoma—less than 5%.
5
Q
  1. The most frequent spinal cord symptom or sign
    in patients with carcinomatous meningitis is
    (A) nuchal rigidity
    (B) back pain
    (C) reflex asymmetry
    (D) positive straight leg raise test
    (E) weakness
A
  1. (C) Reflex asymmetry occurs in 67% of patients
    with carcinomatous meningitis and is the most
    frequent spinal cord–related sign. The frequency
    of nuchal rigidity, back pain, positive straight leg raise test, and weakness is 11%,
    25%, 13%, and 33%, respectively.
6
Q
  1. Which of the following would most likely be
    responsible for the central pain syndrome?
    (A) Epidural spinal cord compression
    (B) Metastatic bony destruction of the
    vertebrae with a nerve root compression
    (C) Metastatic involvement of the cranial
    nerves
    (D) Carcinomatous meningitis
    (E) Radiation myelopathy
A
  1. (E) Central pain syndromes are relatively rare in
    cancer patients. Although epidural spinal cord
    compression is almost always painful, central pain
    is not the predominant symptom. Nociceptive
    input from progressive bony destruction by
    metastases is the usual cause of pain, with or without
    concurrent radicular pain from nerve root
    compression. Radiation myelopathy is the central
    pain syndrome.
7
Q
338. The majority of patients with epidural metastasis
have the following pattern of pain:
(A) Local
(B) Radicular
(C) Referred
(D) Funicular
(E) All of the above
A
  1. (A) The most common pattern of pain in patients
    with epidural metastasis is local. Local pain over
    the involved vertebral body, which results from
    the involvement of the vertebral periosteum, is
    dull and exacerbated by recumbency.
    Radicular pain from compressed or damaged
    nerve roots is usually unilateral in the
    cervical and lumbosacral regions and bilateral
    in the thorax. The pain is experienced in the
    overlying spine, deep in certain muscles supplied
    by the compressed root, and in the cutaneous
    distribution of the injured root.
    Referred pain has a deep aching quality
    and is often associated with tenderness of
    subcutaneous tissues and muscles at the site
    of referral. The typical examples of referred
    pain pattern include buttocks and posterior
    thigh pain with lumbosacral spine involvement;
    pain in the flank, groin, and anterior
    thigh in the upper lumbar spine involvement;
    midscapular and shoulder pain in the cervicothoracic
    epidural disease.
    Funicular pain usually occurs some distance
    below the site of compression and it has hot or cold qualities in a poorly localized nondermatomal
    distribution. It presumably results
    from compression of the ascending sensory
    tracts in the spinal cord.
8
Q
  1. All of the following are true about the World
    Health Organization (WHO) analgesic ladder,
    EXCEPT
    (A) it is a method for relief of cancer pain
    based on a small number of relatively
    inexpensive drugs
    (B) it has three steps
    (C) step one involves the use of opioids
    (D) it suggests to use only one drug from
    each group at a time
    (E) it is a simple and effective method for
    controlling cancer pain
A
  1. (C) The WHO analgesic ladder is based on the
    premise that most patients throughout the world
    gain adequate pain relief if health care professionals
    learn how to use a few effective and
    relatively inexpensive drugs well. Step 1 of the
    ladder involves the use of nonopioids. If this
    step is ineffective, go to step 2 and add an
    opioid for mild to moderate pain. Step 3 substitutes
    an opioid for moderate to severe pain
    in step 2. Only one drug from each group
    should be used at a time. Adjuvant drugs can
    be used in all steps.
9
Q
340. The following are all true about methadone,
EXCEPT
(A) it has a highly variable oral bioavailability
(B) it is a low cost medication
(C) it has no known active metabolites
(D) it has N-methyl-D-aspartate (NMDA)
receptor agonist properties
(E) it has high lipid solubility
A
  1. (D) Methadone has a variable oral bioavailability
    between 41% and 99% and, therefore,
    should be started with extra caution (low initial
    dose and slow subsequent increases).
    Methadone differs from all other opioids by its
    noncompetitive antagonist activity at the
    NMDAreceptors. Activation of NMDA receptors
    has been shown to play a role in development
    of tolerance to analgesic effects of
    opioids, as well as in the pathologic sensory
    states, such as neuropathic pain, inflammatory
    pain, ischemic pain, allodynia, and spinal
    states of hypersensitivity.
10
Q
  1. 58-years-old patient with metastatic prostate
    cancer is taking sustained-release morphine
    (MS Contin) every 8 hours with a total daily
    dose of 225 mg with optimal pain control.
    Because of some circumstances, he has to be
    converted to transdermal therapeutic system
    fentanyl (TTS-fentanyl). What is the correct
    dose of fentanyl patch equivalent to the current
    dose of MS Contin for this patient?
    (A) 25 μg/h every 72 hours
    (B) 50 μg/h every 48 hours
    (C) 75 μg/h every 72 hours
    (D) 100 μg/h every 48 hours
    (E) 125 μg/h every 72 hours
A
  1. (C) As a rough guide for conversion, the 8-hourly
    dose of MS Contin (225/3 = 75 mg in this case)
    can be considered equal to the micrograms per
    hour dose of TTS-fentanyl. In one study, most
    people had satisfactory pain profiles with frequency
    of administration of every 3 days. Only
    in 24% of subjects in the study required different
    frequency of administration varying from
    48 to 60 hours.
11
Q
  1. Which of the following is true with respect to
    central pain syndromes?
    (A) The most common cause of central pain
    state are lesions located in the brainstem
    (B) The Wallenberg syndrome (lateral
    medullar syndrome) is characterized by
    contralateral facial sensory loss and
    Horner syndrome
    (C) The most common lesions that produce
    thalamic pain syndrome are infarctions
    (D) Spinal cord lesions rarely cause sensory
    deficits
    (E) Central pain syndromes of spinal origin
    usually respond to epidural steroids
A
  1. (C)
    A. The most common cause of central pain
    states are spinal cord lesions.
    B. The Wallenberg syndrome is usually vascular
    in origin, and characterized by crossed
    sensory findings that include ipsilateral
    facial sensory loss, Horner syndrome, and contralateral body impairment of pain and
    temperature loss.
    C. The most common lesions that produce thalamic
    pain syndrome are infarctions, followed
    by arteriovenous malformations (AVMs),
    neoplasms, abscesses, plaque of multiple
    sclerosis, traumatic injury, and others.
    D. Spinal cord lesions are the most common
    cause of central pain syndromes and present
    with areas of sensory loss resulting
    from disruption of the spinothalamic tract.
    E. The treatment of central pain of spinal origin
    is complex with poor response to most
    forms of therapy.
12
Q
343. Peripheral neuropathy is a common pain syndrome
characterized by which of the following?
(A) Asymmetric paresthesias and proximal
motor impairment
(B) Proximal more than distal sensory
impairment
(C) Most peripheral neuropathies may be
classified as demyelinating, axonal, or
mixed
(D) Peripheral mononeuropathy is the most
common peripheral nerve disease in
patients with long-standing diabetes
mellitus
(E) Nerve conduction studies only measure
conduction through small unmyelinated
fibers, so impairment of the fast
conducting fibers may go undetected
A
  1. (C) Sensory symmetric impairment is commonly
    seen distally with progression to more
    proximal areas of the limbs as the disease progresses.
    Peripheral polyneopathy is the most
    common initial manifestation of diabetes mellitus.
    The nerve conduction studies measure
    only the fastest conducting fibers, leaving
    injury of small-diameter fibers, which transmit
    pain sensations, undiagnosed.
13
Q
  1. Events seen in the development of neuropathic
    pain are
    (A) following nerve injury, there is a
    decreased activity of the sodium channels
    which allows for abnormal conduction
    through pain facilitating fibers
    (B) wide dynamic range neurons in the
    dorsal horn respond with increased
    frequency as the intensity of the
    repeated afferent stimulus increases
    (C) an increase in potassium channels would
    facilitate an amplified afferent activity
    (D) C-polymodal nociceptors are activated
    by low-threshold mechanical, thermal,
    and chemical stimuli
    (E) γ-aminobutyric acid (GABA) and
    glycine are released in the dorsal horn
    and augment the response of second
    order neurons
A
  1. (B) Following nerve injury there is an increase
    in the expression of sodium channels in the
    neuroma and in the DRG. Consistent with the
    role of sodium channels in the development of
    neuropathic pain is blockage of their activity by
    low plasma concentrations of lidocaine. A reduction
    in potassium channel activity leads to
    increased afferent activity. The largest population
    of afferent axons is C-polymodal nociceptors
    that are activated by high-threshold
    mechanical, thermal, and chemical stimuli.
14
Q
  1. Examples of neuropathic pain conditions include
    all, EXCEPT
    (A) complex regional pain syndrome (CRPS)
    (B) diabetic peripheral neuropathy
    (C) postherpetic neuralgia (PHN)
    (D) Raynaud phenomenon
    (E) phantom limb pain
A
  1. (D) Raynaud phenomenon is not a neuropathic
    pain condition, but rather a vascular condition
    (although, potentially sympathetically mediated
    and/or sustained).
15
Q
  1. Which of the following conditions is more
    likely to be associated with neuropathic pain?
    (A) Traumatic nerve injury
    (B) Stroke
    (C) Syringomyelia
    (D) Multiple sclerosis
    (E) Large myelinated fiber neuropathy
A
  1. (C) Although not completely known some conditions
    predispose to the development of neuropathic
    pain. The relative frequency is 5% for
    patients with traumatic nerve injury, 8% for
    patients after stroke, 28% for patients with multiple
    sclerosis, and 75% for patients with
    syringomyelia. Neuropathies with predominant involvement of large myelinated fibers are usually
    not painful.
16
Q
  1. A patient with CRPS responds well to sympathetic
    ganglion block. The results of this block
    can lead you to say which of the following
    about this particular pain condition?
    (A) It is vascularly mediated
    (B) It is sympathetically mediated
    (C) It is sympathetically maintained
    (D) It is less severe than previously thought
    (E) It will not respond well to spinal cord
    stimulation
A
  1. (C) We do not know if it is sympathetically
    mediated (B) from the block since this does not
    provide evidence of etiology. We do not know
    the involvement of vascularity since the block
    is affecting sympathetic outflow and precludes
    vascular evidence (which could be mediated by
    a host of other physiologic events). There is no
    clinical evidence to support a less severe case (D)
    and, the evidence suggests that it will respond
    to spinal cord stimulation (E).
17
Q
348. Neuropathic pain can result in which of the
following condition?
(A) Central sensitization
(B) Allodynia
(C) Hyperalgesia
(D) B and C
(E) A, B, and C
A
  1. (E) Central sensitization is the reason for many
    of the symptoms including allodynia and
    hyperalgesia. Therefore, all are correct.
18
Q
  1. Potential neurophysiologic mechanisms underlying
    the development of neuropathic pain
    include
    (A) microglial activation in the spinal cord
    (B) cytokine production in the spinal cord
    (C) decreased glutamate release in the
    spinal cord
    (D) A and C
    (E) A and B
A
  1. (E) Cytokines are inflammatory mediators
    released by a variety of cells that regulate the
    inflammatory response. Systemic or local injection
    of cytokines in animal models causes mechanical
    and thermal hyperalgesia. Cytokines may cause
    excitation of nociceptors via the release of other
    mediators, like prostaglandins. At the level of the
    CNS, cytokines may be liberated by microglial
    cells. The best studied excitatory amino acid is
    glutamate. Glutamate may bind to ionotopic or
    metabotropic glutamate receptors. Peripheral and
    central activation of those receptors induces pain
    behaviors in animals. All basic science evidence
    suggests (A) and (B), but does not suggest (C).
19
Q
350. When the stimulus of light touch exerts pain
which of the following is exhibited?
(A) Hyperalgesia
(B) Allodynia
(C) Hypereflexemia
(D) Paresthesia
(E) Hypertouchemia
A
  1. (B) Following tissue damage, there is a decrease
    of the threshold for noxious stimuli (hyperalgesia),
    which may be associated to perception of
    pain to normally innocuous stimuli. This phenomenon
    is termed allodynia. Allodynia is most
    likely caused by plastic changes at the level of the
    primary sensory fibers and spinal cord neurons.
20
Q
351. Phantom pain refers to
(A) any sensation of the missing limb,
except pain
(B) painful sensations referred to the
missing limb
(C) spontaneous movement of the stump
ranging from small jerks to visible
contractions (jumpy stump)
(D) pain referred to the amputation stump
(E) B and D
A
  1. (B) Phantom sensation: any sensation of the
    missing limb, except pain (A).
    Stump contractions: spontaneous movement
    of the stump ranging from small jerks to
    visible contractions (jumpy stump) (C).
    Stump pain: pain referred to the amputation
    stump (D).
21
Q
  1. A 74-year-old male has a left lower extremity
    amputation after a long bout with uncontrolled
    diabetes mellitus (DM). What are the chances
    that this patient will develop phantom pain?
    (A) 33%
    (B) 49%
    (C) 55%
    (D) 90%
    (E) 75%
A
  1. (E) While ranges between 2% and 88% are
    quoted in the literature, most current studies state that between 60% and 80% of patients
    will develop phantom pain after amputation.
22
Q
  1. A vascular surgeon consults the pain team on
    a patient who is scheduled to undergo an
    amputation secondary to peripheral vascular
    disease. The patient has read about phantom
    pain on the Internet and would like to know
    when it would likely start. You tell the vascular
    surgeon that
    (A) the onset of phantom pain is usually
    within the first week after amputation
    (B) most studies have shown that phantom
    pain will start between 2 and 4 weeks
    after an amputation for peripheral
    vascular disease
    (C) the likelihood of her developing
    phantom pain in the first 6 months after
    amputation is low, but increases drastically
    between 6 and 9 months
    (D) the onset will likely be delayed for years
    (E) none of the above
A
  1. (A) Prospective studies in patients undergoing
    amputation mainly because of peripheral vascular
    disease have shown that the onset of
    phantom pain is usually within the first week
    after amputation.
    However, in a retrospective study of individuals
    who were congenital amputees or
    underwent amputation before the age of 6 years,
    Melzack and coworkers found that the mean
    time for onset of phantom limb pain was 9 years
    in the group of congenital amputees and 2.3 years
    in the group of individuals with early amputations.
    [Jensen TS, Krebs B, Nielsen J, et al.
    Phantom limb, phantom pain, and stump pain
    in amputees during the first 6 months following
    limp amputation.
23
Q
  1. The patient mentioned in the previous question
    develops early and severe phantom pain:
    (A) The patient is more likely to suffer from
    long-standing pain
    (B) The patient is less likely to suffer from
    long-standing pain
    (C) The patient is more likely to suffer incapacitating
    pain for 1 year that will
    subside rather abruptly
    (D) It is likely that the patient will develop
    neuropathic pain in the extremity
    contralateral to the amputation
    (E) The pain will likely be refractory to
    treatment with anticonvulsants
A
  1. (A) Patients who develop early and severe
    phantom pain are more likely to suffer from
    chronic pain, whereas individuals who are
    pain-free at the beginning are less likely to
    develop significant pain. However, prospective
    studies with a maximum follow-up period of
    2 years suggest that phantom pain may diminish
    with time.
24
Q
355. The number of amputees who have severe
phantom limb pain is
(A) 20% to 30%
(B) 60% to 80%
(C) 5% to 10%
(D) 1% to 2%
(E) 45% to 55%
A
  1. (C) While phantom limb pain is seen in 60% to 80% of amputees, only 5% to 10% have severe pain.
25
Q
  1. Preamputation pain
    (A) is more likely to lead to phantom pain if
    the amputation is traumatic
    (B) may sensitize the nervous system,
    explaining why some individuals may
    be more susceptible to development of
    chronic phantom pain
    (C) is more likely to lead to phantom pain if
    the amputation is secondary
    (D) is similar in character and localization to
    the subsequent phantom pain in 80% of
    patients
    (E) is less likely to lead to phantom pain if the
    amputation is in the upper extremities
A
  1. (B) Some retrospective studies, but not all have
    pointed to preamputation pain as a risk factor
    for phantom pain. It has been hypothesized that
    preoperative pain may sensitize the nervous
    system, explaining why some individuals may
    be more susceptible to development of chronic
    pain.
    A. It has been noted that patients with traumatic
    amputations, who had no pain prior
    to the amputation, develop pain to the same
    extent as patients with preoperative pain
    who endure amputations after significant
    medical pathology.
    C. There is no correlation between the development
    of phantom pain and whether the
    amputation was primary or secondary.
    Primary amputation is when the limb is lost
    at the time of the injury. Secondary amputation
    is when the limb is surgically removed
    in a hospital.
    D. Phantom pain may mimic preamputation
    pain in both character and localization.
    Preamputation pain may persist in some
    patients, but it is not the case in the majority
    of patients.
    E. Site of amputation has not been found to
    have a role in determining whether preamputation
    pain leads to phantom pain.
26
Q
  1. A 25-year-old left lower extremity amputee
    returns from Iraq. He experiences phantom
    pain, but is attempting to move forward in life.
    To ease his transition back into society which of
    the following is the next best step?
    (A) He should take as long as possible to
    grieve before he finds new employment
    (B) He should initially use a cosmetic
    prosthesis before embarking on the task
    of learning to use a functional one
    (C) He should absolutely refuse to ever
    have spinal anesthesia as it may worsen
    phantom pain
    (D) He should learn coping strategies as
    phantom pain is a psychological
    disturbance
    (E) None of the above
A
  1. (E)
    A. Amputees who experienced a long delay
    between the amputation and return to
    work, had difficulty in finding suitable
    jobs, and had fewer opportunities for
    promotion.
    B. The use of a functionally active prosthesis
    as opposed to a cosmetic prosthesis may
    reduce phantom pain.
    C. Spinal anesthesia in amputees may precipitate
    transient, difficult to treat phantom
    pain. Given the low incidence of recurrent
    phantom limb pain with spinal anesthesia,
    its transient nature, and the fact that it can
    be treated if it occurs, it has been concluded
    that spinal anesthesia is not contraindicated
    in patients with previous
    lower limb amputation.
    D. While there is no evidence that phantom
    pain represents a psychological disturbance,
    it may be triggered and precipitated
    by psychosocial factors. It has been shown
    that coping strategies are important for the
    experience of phantom pain Research has
    indicated that the way individuals cope
    with pain may influence pain, and physical
    and psychological adjustment.
27
Q
  1. Stump pain and phantom pain are often
    confused. There are, however, notable differences.
    Which of the following is true?
    (A) Unlike phantom pain, stump pain
    occurs in the body part that actually
    exists, in the stump that remains
    (B) Stump pain typically is described as a
    “sharp,” “burning,” “electric-like,” or
    “skin-sensitive” pain
    (C) Stump pain is usually caused by a
    neuroma
    (D) Surgical revision of the stump or
    removal of the neuroma is sometimes
    considered when treating stump pain
    (E) All of the above
A
  1. (E) Stump pain is located at the end of an amputated
    limb’s stump. Unlike phantom pain, it
    occurs in the body part that actually exists, in the
    stump that remains. It typically is described as
    a “sharp,” “burning,” “electric-like,” or “skinsensitive”
    pain. Some patients have spontaneous
    movements of the stump, ranging from slight,
    hardly visible jerks to severe contractions.
    Stump pain results from a damaged nerve
    in the stump region. Nerves damaged in the
    amputation surgery try to heal and may form
    abnormally sensitive regions, called neuromas.
    A neuroma can cause pain and skin sensitivity.
    Percussion of neuromas may increase
    nerve fiber discharge and augmentation of
    stump and phantom pain.
    No one treatment has been shown to be
    effective for stump pain. Because it is a pain
    caused by an injured peripheral nerve, drugs
    used for nerve pain may be helpful.
    If the stump pain affects a limb, revision
    of the prosthesis is sometimes beneficial.
    Other approaches also are tried in selected
    cases, including: nerve blocks, transcutaneous
    electrical nerve stimulation, surgical revision
    of the stump, or removal of the neuroma (this
    procedure may fail because the neuroma can
    grow back; some patients actually get worse
    after surgery), and cognitive therapies.
    Stump pain is common in the early postamputation
    period. Stump pain can also persist
    beyond the stage of postsurgical healing.
    Stump pain and phantom pain are strongly
    correlated. Phantom pain subsides with resolution
    of stump pain and that it is more prevalent
    in patients with phantom pain than in
    those without it.
    Careful sensory examination of amputation
    stumps may reveal areas of sensory abnormalities
    such as hypoesthesia, hyperalgesia, or allodynia.
    However, a correlation between phantom
    pain and the extent and degree of sensory abnormality
    has not been established.
28
Q
  1. A neuroma is an inflammation of a nerve that
    is seen universally after a nerve has been cut
    (ie, during an amputation). They show spontaneous
    and abnormal evoked activity following
    mechanical or chemical stimulation from
    the periphery. This results from
    (A) an increased and novel expression of
    sodium channels
    (B) hyperexcitability changes and
    reorganization of the thalamus
    (C) an increase in potassium efflux
    (D) increased activity in afferent C fibers
    (E) A and D
A
  1. (E) The ectopic and increased spontaneous and
    evoked activity from the periphery is assumed
    to be the result of an increased and also novel
    expression of sodium channels.
    Local anesthesia of the stump may reduce
    or abolish phantom pain temporarily.
    Decreasing peripheral output by locally anesthetizing
    stump neuromas with lidocaine
    reduced tap-evoked stump pain. On the other
    hand, there was a clear increase in pain when the potassium channel blocker, gallamine was
    injected in the perineuromal space. Both findings
    support the premise that abnormal input
    from peripheral nociceptors plays a role in
    pain generation.
29
Q
  1. Some amputees show an abnormal sensitivity
    to pressure and to repetitive stimulation of the
    stump, which can provoke attacks of phantom
    pain. Which of the following is the case in
    humans?
    (A) It can be reduced by giving the NMDA
    antagonist, ketamine
    (B) It can only be reduced by terminating
    the stimulation
    (C) It can be attributed to the general
    excitability of spinal cord neurons,
    where only C fibers gain access to
    secondary pain-signaling neurons
    (D) Sensitization of the dorsal horn may be
    mediated by glycine and serotonin
    (E) All of the above
A
  1. (A) The pharmacology of spinal sensitization
    entails an increased activity in NMDA
    receptor–operated systems, and many aspects
    of the central sensitization can be reduced by
    NMDA receptor antagonists In amputees, the
    evoked pain from repetitive stimulation can be
    reduced by the NMDA antagonist ketamine
    B. Terminating the stimulation is not the only
    way to reduce the pain.
    C. After a nerve is injured, there is an increase
    in the general excitability of spinal cord
    neurons, where C fibers and A-δ afferents
    gain access to secondary pain-signaling
    neurons.
    D. Sensitization of dorsal horn neurons is
    mediated by release of glutamate and neurokinin.
    This sensitization may present in
    several ways including: lowered threshold,
    increased persistent neuronal discharges
    with prolonged pain after stimulation, and
    expansion of peripheral receptive fields.
    The central sensitization may also be a
    result of a different type of anatomical reorganization.
    Substance P is normally expressed
    in small afferent fibers, but following nerve
    injury, it may be expressed in large A-β fibers.
    This phenotypic switch of large A-β fibers into
    nociceptive-like nerve fibers may be one of
    the reasons why nonnoxious stimuli can be
    perceived as painful
30
Q
361. Of the following, which does not play a role in
the mechanism for generating phantom pain?
(A) Peripheral sensitization
(B) Central sensitization
(C) Cortical reorganization
(D) Increased thalamus response to
stimulation
(E) Sympathetic inhibition
A
  1. (E)
    E. The sympathetic nervous system may play
    a role in generating and, in uparticular, in
    maintaining, phantom pain.
    After limb amputation and deafferentation
    in adult monkeys, there is reorganization of the
    primary somatosensory cortex, and while these
    changes may be unique to the cortex, they may
    also be, at least in part, the result of changes at
    the level of the thalamus and perhaps even
    brain stem or spinal cord. After dorsal rhizotomy,
    the threshold to evoke activity in the
    thalamus and cortex decreased, and the mouth
    and chin invade cortices corresponding to the
    representation of arm and fingers that have lost
    their normal afferent input. In humans similar
    reorganization has been observed. In the thalamus,
    neurons that normally do not respond to
    stimulation in amputees begin to respond and
    show enlarged somatotropic maps. A cascade
    of events seems to be involved in generating
    phantom pain and it starts in the periphery,
    spinal cord, brain stem, thalamus, and finally
    ends in the cerebral cortex.
31
Q
  1. Pharmacologically treating phantom pain is
    not easy. Which of the following medications
    has not proven to be effective in well-controlled
    trials?
    (A) Tramadol
    (B) Gabapentin
    (C) Memantine
    (D) Amitriptyline
    (E) A and C
A
  1. (C)
    A. and D. Tramdol and amitriptyline have been
    found to be efficacious in treating phantom
    and stump pain in treatment naive patients.
    B. Gabapentin has been noted to be better
    than placebo in reducing phantom pain.
    Failure to pharmacologically provide pain
    relief should not be accepted until opioids
    have been tried. Intravenous (IV) and oral
    morphine have been shown to decrease phantom
    pain. Case reports have indicated that
    methadone may also be helpful.
    Other trials have not reported the same
    the success with an oral NMDA antagonist,
    memantine.
    Suggestions for the treatment of postamputation
    pain (no evidence) (Note: it is important
    to differentiate between early postoperative
    pain and chronic pain [pain persisting more
    than 4 weeks], and stump and phantom pain):
    Early postoperative pain
    Stump pain
    Conventional analgesics
    • Acetaminophen
    • NSAIDs
    • Opioids
    +/− combined with epidural pain treatment
    Stump and phantom pain
    If neuropathic pain clearly exists (paroxysms
    or abnormal stump sensitivity)—trial
    with TCAs or anticonvulsants.
    Chronic pain
    Stump pain
    • Local stump surgery: if obvious stump
    pathology is present, revisions should be
    considered; surgery should be avoided in
    cases of sympathetically maintained pain.
    • Local medical treatment: topical lidocaine or
    capsaicin can be tried in those who have
    stump pain but no obvious stump pathology.
    Stump and phantom pain (medical treatment,
    in order of preference)
    • Gabapentin 1200 to 2400 mg/d, slow titration.
    Max dose of 3600 mg/d.
    • TCAs (imipramine, amitriptyline, nortriptyline)
    100 to 125 mg/d, slow titration. Check
    electrocardiogram (ECG) before starting.
    Monitor plasma levels with dose greater
    than 100 mg/d. If sedation is wanted,
    amitriptyline should be used.
    • If the pain is mostly paroxysmal, lancinating,
    or radiating:
    • Oxcarbazepine 600 to 900 mg/d. Start at
    300 mg and increase by 300 mg daily.
    • Carbamazepine 450 mg/d. Start dose
    150 mg, daily increments of 150 mg.
    Monitor plasma levels after 10 days on
    maximum dose.
    • Lamotrigine 100 to 200 mg/d. Start dose
    25 mg/d, slow titration with increments
    of 25 mg/14 days (to avoid rash).
    • Opioids (long-acting) or tramadol.
    • If none of the above has an effect, refer the
    patient to the pain clinic.
    • In pain center: can perform IV lidocaine
    trial or ketamine trial. If the lidocaine test is
    positive—reconsider anticonvulsants. If the
    ketamine test is positive: consider memantine
    or amantadine.
    Physical therapy encompassing massage,
    manipulation, and passive range of motion
    may prevent trophic changes and vascular
    congestion in the stump. Transcutaneous electrical
    nerve stimulation, acupuncture, ultrasound,
    and hypnosis, may have a beneficial
    effect on stump and phantom pain.
32
Q
363. A65-year-old Vietnam War veteran with a left
below the knee amputation and phantom pain
has surgery on an amputation neuroma. He
should expect
(A) excellent resolution of his phantom pain
(B) short-term pain relief
(C) a likely infection and subsequent
complicated hospital course
(D) decreased pain only if he receives a
40-minute infusion of diphenhydramine
within 24 hours of the surgery
(E) none of the above
A
  1. (B) Surgery on amputation neuromas and more
    extensive amputation were accepted treatment modalities for stump and phantom pain in the
    past. Today, stump revision is probably done
    only in cases of obvious stump pathology, and
    in properly healed stumps there is almost never
    an indication for proximal extension of the
    amputation because of pain. Surgery should
    be avoided in cases of sympathetically maintained
    pain. Surgery may produce short-term
    pain relief but pain often reappears. The results
    of other invasive procedures such as dorsal
    root entry zone lesions sympathectomy and
    cordotomy have generally been nontherapeutic,
    and most of them have been abandoned.
33
Q
  1. Apatient has tingling sensations in a phantom
    limb that are uncomfortable and annoying
    but do not interfere with activities or sleep.
    According to the Sunderland classification of
    patients with phantom pain, what group is this
    patient in?
    (A) Group I
    (B) Group II
    (C) Group III
    (D) Group IV
    (E) None of the above
A
  1. (B) Classification of patients with phantom
    pain:
    Group I: Mild intermittent paresthesias that
    do not interfere with normal activity, work, or
    sleep.
    Group II: Paresthesias that are uncomfortable
    and annoying but do not interfere with activities
    or sleep.
    Group III: Pain that is of sufficient intensity,
    frequency, or duration to be distressful; however,
    some patients in this group have pain
    that is bearable, that intermittently interferes
    with their lifestyle, and that may respond to
    conservative treatment.
    Group IV: Nearly constant severe pain that
    interferes with normal activity and sleep.
34
Q
365. The gate-control theory of pain has been used
to explain phantom limb pain. It states that
(A) following significant destruction of
sensory axons by amputation, wide
dynamic range neurons are freed by
inhibitory control
(B) self-sustaining neuronal activity may
occur in spinal cord neurons
(C) if spontaneous spinal cord neuronal
activity increases by any amount, pain
may occur in the phantom limb
(D) A and B
(E) A, B, and C
A
  1. (D)
    A. and B. The gate control theory of pain, put
    forward by Ronald Melzack and Patrick
    David Wall in 1962, and again in 1965, is the
    idea that the perception of physical pain is
    not a direct result of activation of nociceptors,
    but instead is modulated by interaction
    between different neurons, both paintransmitting
    and non–pain-transmitting.
    The theory asserts that activation of nerves
    that do not transmit pain signals can interfere
    with signals from pain fibers and
    inhibit an individual’s perception of pain. It
    has been used to explain phantom limb
    pain. Following marked destruction of sensory
    axons by amputation, wide dynamic
    range neurons are freed by inhibitory control. Self-sustaining neuronal activity may then
    occur in spinal cord neurons.
    C. If the spontaneous spinal cord neuronal
    activity exceeds a critical level, pain may
    occur in the phantom limb.
    This loss of inhibitory control may lead to
    spontaneous discharges at any level in the
    CNS and may explain the lack of analgesia in
    paraplegics with phantom body pain after
    complete cordectomy Pain increases after
    blocking conduction are in line with the theory,
    as continued loss of peripheral sensory
    input would lead to further disinhibition.
    Sodium thiopental perpetuates CNS inhibition
    and has been reported to end phantom
    limb pain during spinal anesthesia. Melzack
    R, Wall PD. Mechanisms: a new theory. Agate
    control system modulates sensory input from
    the skin before it evokes pain perception and
    response. Science. 1965;150(3699).
35
Q
  1. All of the following are true about primary
    dysmenorrhea, EXCEPT
    (A) pain is transmitted via the thoracolumbar
    spinal segments and pelvic afferents
    (B) the etiology of pain includes myometrial
    contractions leading to intense intrauterine
    pressure and uterine hypoxia
    (C) prostaglandins and leukotriene production
    that sensitizes afferent pelvic nerves
    is part of its pathogenesis
    (D) endometriosis and adenomyosis are its
    most common causes
    (E) altered central receptivity of the afferent
    input from the pelvis is thought to be
    relevant in its development
A
  1. (D) Primary dysmenorrhea is defined as
    menstrual pain without pelvic pathology.
    Endometriosis and adenomyosis are the most
    common causes of secondary dysmenorrhea.
36
Q
367. All of the following are true about chronic
endometriosis, EXCEPT
(A) ovaries, cul-de-sac, uterine tubes, surface
of the bowel are among the most common
sites of pathologic implantation of
the functioning endometrial tissue
(B) retrograde menstruation, lymphatic
spread, and hematogenous spread of the
endometrial tissue are all thought to
play a role in endometriosis etiology
(C) pain occurs only with menses
(D) definitive diagnosis can be made by visualization
of the characteristic lesions without
a mandatory histologic confirmation
(E) leuprolide acetate (Lupron) may be an
effective treatment of the symptoms of
chronic endometriosis
A
  1. (C) The pain of endometriosis can occur with
    menses or sexual intercourse or can always be
    present. It can also mimic any known pelvic
    pathology. Answers A, B, D, and E are all correct.
37
Q
  1. All of the following are correct, EXCEPT
    (A) pudendal nerve takes origin from S2, S3,
    and S4 roots bilaterally
    (B) bilateral denervation of the inferior
    hypogastric nerves is as effective as a
    lumbar epidural block with respect to
    sensory input from the uterus and cervix
    (C) many patients with hymenal neuropathy
    are so emotional and complain so
    violently that the pelvic examination is
    not possible
    (D) patients with sympathetic pelvis syndrome
    have a deep pain in the pelvis
    not associated with physically
    detectable abdominal wall or muscle
    tenderness
    (E) ilioinguinal and iliohypogastric neuropathy
    is rarely associated with the surgeries
    in the lower abdominal wall area
A
  1. (E)
38
Q
  1. All of the common reasons for the inadequate
    management of acute pain in a hospital setting
    are true, EXCEPT
    (A) the common idea that pain is merely a
    symptom and not harmful in itself
    (B) the fact that opioids have no potential
    for addiction when administered strictly
    for acute pain
    (C) lack of understanding of the pharmacokinetics
    of various agents
    (D) lack of appreciation of variability in
    analgesic response to opioids
    (E) prescription of inappropriately low
    doses of opioids and thinking that
    opioids must not be given more often
    than every 4 hours
A
  1. (B) Opioids have the potential for addiction
    even when administered for acute pain.
    However, it is the exaggerated common fear
    of the potential for addiction to opioids that
    often interferes with adequate pain management.
    The rest of the answers are correct.
39
Q
  1. The following are true about pathologic
    (nonphysiological) pain, EXCEPT
    (A) it occurs in the context of central
    sensitization
    (B) it occurs in the context of peripheral
    sensitization
    (C) it outlasts the stimulus
    (D) it spreads to nondamaged areas
    (E) it is elicited by A-δ and C fibers, but not
    A-β fibers, which transmit touch sensation
A
  1. (E) It is recognized that long-term changes
    occur within the peripheral and central nervous
    system following noxious input. This neuroplasticity
    alters the body’s response to usual
    peripheral sensory input. In pathologic pain
    conditions, stimulation of A-β fibers, normally eliciting response to touch, may elicit pain.
40
Q
  1. Perioperative administration of NSAIDs
    (A) does not reduce the demand for opioids
    during and after the surgery
    (B) is contraindicated because of increased
    possibility of bleeding
    (C) has synergistic effect with opioids
    (D) has its analgesic effect only through
    peripheral mechanisms
    (E) is not associated with the concerns for
    postoperative bleeding
A
  1. (C) Even though there have been some concerns
    regarding the risks of perioperative NSAIDs, including intra- and postoperative bleeding,
    they continue to have a useful role. Combination
    of NSAIDs and opioids has a synergistic
    analgesic effect, as they act at the different sites
    of pain pathways. More new evidence is emerging
    that NSAIDs exert their analgesic effects also
    through the central mechanisms.
41
Q
372. All of the following are true about the NMDA
receptors, EXCEPT
(A) they are involved in development of
“windup” facilitation
(B) NMDA agonists reduce development of
tolerance to opioids
(C) NMDA receptors are involved in
development of central sensitization
(D) NMDA receptors are involved in
changes of peripheral receptive fields
(E) NMDA receptors are involved in
induction of oncogenes and long-term
potentiation
A
  1. (B) It has been demonstrated that the administration
    of an NMDA antagonist reduces the
    development of tolerance to morphine. The rest
    of the answers are correct.
42
Q
  1. As compared with somatic pain, all of the following
    are true about visceral pain, EXCEPT
    (A) it may follow the distribution of a
    somatic nerve
    (B) it is dull and vague
    (C) it is often periodic and builds to peaks
    (D) it is often associated with nausea and
    vomiting
    (E) it is poorly localized
A
  1. (A) The following are the usual features of the
    somatic pain: well localized, sharp and definite,
    often constant (sometimes periodic); it is rarely
    associated with nausea usually when it is deep
    somatic pain with bone involvement; it may be
    following the distribution of a somatic nerve.
    In contrast, the visceral pain: is poorly localized,
    diffuse, dull, and vague; it is often periodic and
    builds to peaks (sometimes constant); it is often
    associated with nausea and vomiting.
43
Q
374. The following statements are true regarding
preemptive analgesia, EXCEPT
(A) preemptive analgesia is helpful in
reducing postoperative pain in part by
reducing the phenomenon of central
sensitization
(B) early postoperative pain is not a significant
predictor of long-term pain
(C) local anesthetics, opioids, and NSAIDs
can be used for preemptive analgesia
(D) preemptive analgesia may have the
potential to prevent the development of
chronic pain states
(E) preemptive analgesia is thought to
reduce neuroplastic changes in the
spinal cord
A
  1. (B) It has been demonstrated that early postoperative
    pain is a significant predictor of longterm
    pain. The rest of the answers are correct.
44
Q
  1. The following statements are true regarding
    multimodal analgesia, EXCEPT
    (A) it may include NSAIDs, acetaminophen,
    local anesthetics, and opioids in the
    same patient
    (B) it is beneficial because of the synergistic
    action of the individual medications
    with different sites of action along the
    pain pathways
    (C) it is not very valuable owing to an
    increase in the incidence of side effects
    (D) it facilitates early mobilization of the
    postsurgical patient
    (E) it expedites return to normal parenteral
    nutrition
A
  1. (C) Multimodal analgesia makes it possible to
    significantly reduce the total consumption of
    opioids intra- and postoperatively. Therefore,
    opioid side-effects are minimized, including
    inevitable opioid-induced GI stasis that delays
    the resumption of normal enteral nutrition after
    surgery.
45
Q
376. All of the following statements about PHN are
correct, EXCEPT
(A) midthoracic dermatomes is one of the
most common sites for PHN
(B) men are affected more often than
women in a ratio of 3:2
(C) ophthalmic division of the trigeminal
nerve is one of the most common sites
for PHN
(D) PHN may occur in any dermatome
(E) PHN has an incidence of 9% to 14.3%
A
  1. (B) PHN affects women more often than men, in a ratio of approximately 3:2. The rest of the answers are correct.
46
Q
377. PHN is defined as
(A) any pain associated with the herpes
zoster
(B) pain caused by herpes zoster for more
than 1 month
(C) persistent pain with a significant
neuropathic component in a dermatomal
distribution
(D) pain caused by herpes zoster for more
than 3 months
(E) neuropathic pain in midthoracic dermatomes
caused by herpes simplex virus
A
  1. (B) PHN is defined as pain caused by herpes zoster for more than 1 month.
47
Q
378. Which of the following is true about the management
of PHN?
(A) Approximately 40% of patients with
PHN have either incomplete or no relief
from treatment
(B) Prevention of herpes zoster is not nearly
as important as a multimodal treatment
of PHN
(C) Current multimodal treatment of PHN
is nearly 100% effective, independent of
the duration of the symptoms
(D) Current multimodal treatment of PHN
is nearly 100% effective as long as it is
started within the first month of the
symptoms of PHN
(E) Current multimodal treatment of PHN
is nearly 100% effective as long as it is
started immediately after the first
symptoms of herpes zoster
A
  1. (A) As many as 40% of patients with PHN have
    either incomplete or no relief from treatment.
    Because of this, the future may lie with prevention
    through vaccination and early aggressive treatment
    of herpes zoster with antivirals and analgesics
    to reduce the extent of the nerve damage
    and sensitization that may correlate with PHN.
48
Q
  1. The following are true about the use of antidepressants
    in treatment of PHN, EXCEPT
    (A) amitriptyline has been shown to be
    effective in treatment of PHN, but has
    significant limitations in the long term
    because of its side effects
    (B) selective serotonin reuptake inhibitors
    (SSRIs) have been found to be equally or
    more effective in treatment of PHN than
    the older generation of tricyclic antidepressants
    (TCAs) or selective norepinephrine
    reuptake inhibitors (SNRIs)
    (C) SNRIs have been shown to be more effective
    than placebo in treatment of PHN
    (D) antidepressant therapy in PHN is built
    on sound, scientific basis
    (E) one of the significant side effects of
    TCAs is their anticholinergic properties
A
  1. (B) Experience with serotonergic antidepressants,
    such as clomipramine, trazodone, nefazodone,
    fluoxetine, and zimelidine, in PHN
    has been disappointing. The evidence supporting
    the use of noradrenergic agents is more
    compelling. The rest of the answers are correct.
49
Q
  1. Which of the following is true about use of
    opioids in the treatment of PHN?
    (A) The use of opioids is not justified for
    nonmalignant pain
    (B) Opioids tend to be less effective for the
    treatment of neuropathic pain than
    nonneuropathic pain
    (C) Opioids were not found to be useful in
    the treatment of PHN
    (D) The use of opioids should be avoided in
    combination with antidepressants
    because of the risk of excessive central
    nervous system (CNS) suppression
    (E) The use of opioids in PHN should be
    avoided owing to the increased
    potential of addiction
A
  1. (B) There has been evidence that opioids do not relieve neuropathic pain as well as nonneuropathic pain. However, there is also evidence that opioids have been successfully used for the treatment of PHN.
50
Q
381. Which of the following is the most common cause
of autonomic neuropathy in the developed
world?
(A) Leprosy
(B) Diabetes mellitus (DM)
(C) Human immunodeficiency virus (HIV)
infection
(D) Heavy metal poisoning
(E) Idiopathic etiology
A
  1. (B) DM is the most common cause of autonomic
    neuropathy, and peripheral neuropathy
    in general, in the United States, as well as in the
    rest of the developed world. Leprosy is the
    most common cause of peripheral neuropathy
    in the world.
51
Q
  1. Diabetic amyotrophy
    (A) has a poor prognosis
    (B) has better prognosis when it involves
    upper extremities
    (C) usually resolves within 1 to 2 years
    spontaneously
    (D) has better prognosis when the symptoms
    do not involve pain
    (E) it is directly related to hyperglycemia
A
  1. (C) Diabetic amyotrophy starts with pain and
    involves the lower extremities. It has a good
    prognosis and usually resolves spontaneously
    in 12 to 24 months. It is not directly related to
    hyperglycemia.
52
Q
383. The following are true about the distal sensorimotor
polyneuropathy, EXCEPT
(A) it is the most common neuropathic
manifestation of both type 1 and type 2
diabetes
(B) it starts distally and spreads proximally
(C) initial symptoms may involve
numbness and tingling in the toes or
feet
(D) it is a length-dependent neuropathy
(E) it is usually asymmetrical
A
  1. (E) Distal sensorimotor polyneuropathy is a
    symmetrical length-dependent process with
    dying-back or dropout of the longest nerve
    fibers—myelinated and unmyelinated. All
    other answers are correct.
53
Q
  1. The prevalence of diabetic neuropathy in DM patients is
    (A) less than 1% at diagnosis of DM, rising
    to 10% in patients diagnosed for longer
    than 5 years
    (B) about 10% at diagnosis of DM, rising to
    more than 50% in patients diagnosed for
    longer than 5 years
    (C) about 50% at diagnosis of DM, rising to
    almost 100% in patients diagnosed for
    longer than 5 years
    (D) about 50% at diagnosis of DM, and does
    not change significantly with time
    (E) no such studies have been done so far
A
  1. (B) It is generally agreed that the prevalence of
    neuropathy is about 10% at diagnosis of DM,
    rising to 50% or more in patients diagnosed for
    longer than 5 years
54
Q
  1. Patients with diabetic distal sensorimotor
    polyneuropathy initially may complain of
    numbness and tingling in the toes or feet,
    which then slowly spreads proximally over
    months to years. Eventually, numbness and
    tingling appear in the fingertips, as the symptoms
    of diabetic polyneuropathy progress to
    (A) ankle
    (B) knee
    (C) mid-thigh
    (D) buttock and groin
    (E) abdomen
A
  1. (B)
55
Q
  1. Which of the following is the most widely
    accepted cause of trigeminal neuralgia?
    (A) Demyelinating conditions, as trigeminal
    neuralgia is most common in patients
    with multiple sclerosis
    (B) Direct trauma of the trigeminal ganglion
    at the level of the foramen ovale, before
    branching into its three branches
    (C) Arterial cross-compression of the
    trigeminal nerve in the posterior fossa
    (D) Tumors of the posterior fossae
    (E) Poor vascular supply to the affected
    trigeminal branch
A
  1. (C) It is accepted that the most common cause
    of trigeminal neuralgia is arterial crosscompression
    of the trigeminal nerve in the posterior
    fossa, as suggested by Jannetta in 1982.
    Electron microscopy of trigeminal nerve biopsies
    taken from patients with trigeminal neuralgia
    has shown areas of axonal swelling and demyelination
    adjacent to the area of arterial compression.
    Although trigeminal neuralgia is more
    common in patients with multiple sclerosis,
    only a small portion of patients with trigeminal neuralgia suffer from multiple sclerosis and
    does not explain the majority of the cases.
56
Q
  1. Which of the following is true regarding medical
    management for the treatment of trigeminal
    neuralgia?
    (A) Anticonvulsant medications are usually
    considered as the second line of treatment
    (B) Beneficial effects of carbamazepine are
    better in elderly patients
    (C) Risk of side effects of carbamazepine
    increase with age
    (D) Carbamazepine has proven to be the
    most effective treatment for trigeminal
    neuralgia, independently of the sideeffect
    profile
    (E) Because of the unlikelihood of serious
    side effects with surgery, all patients
    should consider this option first
A
  1. (C) Carbamazepine is likely to be beneficial in
    up to 70% of the patients. Incidence of side effects
    is often higher in elderly patients especially if the
    drug escalation is too fast. Allergic rash is seen in
    up to 10% of the patients and high concentration
    of the drug may be associated with fluid retention
    promoting cardiac problems. Carbamazepine is a
    potent hepatic enzyme inducer which can potentially
    lead to undesirable drug-to-drug interactions.
    Although microsurgical exploration of the
    posterior fossa is the highly successful, it is a
    major surgery with 0.5% risk of mortality and
    major morbidity. The effectiveness of pimozide
    for trigeminal neuralgia is better than carbamazepine,
    but the high frequency of side effects
    limits its clinical use.
57
Q
  1. The gasserian ganglion
    (A) receives exclusively proprioceptive information
    from the muscles of mastication
    (B) the mandibular branch is located medial
    to the ophthalmic branch
    (C) the two medial branches are sensory
    while the lateral branch is partially
    motor
    (D) the ganglion lies out of the cranium, in
    the Meckel cave
    (E) the foramen rotundum is used as landmark
    for the blockage of the trigeminal
    ganglion
A
  1. (C) The trigeminal ganglion receives sensation
    from the oral mucosa, scalp, nasal areas, face,
    and teeth. Proprioceptive information is transmitted
    into the ganglion from the mastication
    and extraocular muscles. The peripheral
    branches of the ganglion are the ophthalmic,
    the maxillary, and the mandibular, which are
    organized somatotropically, with the ophthalmic
    branch located dorsally, the maxillary
    branch is intermediate, and the mandibular
    nerve is located ventrally. The gasserian ganglion
    lies within the cranium, in the middle
    cranial fossa. The posterior border of the ganglion
    includes the dura of the Meckel cave. The
    landmark to perform the trigeminal ganglion
    block is the foramen ovale and not the foramen
    rotundum.
58
Q
  1. Which of the following is true regarding the
    diagnosis of trigeminal neuralgia?
    (A) The diagnosis must be confirmed with
    magnetic resonance imaging (MRI) to
    detect vascular trigeminal nerve compression
    (B) Sensory evoked potentials is the most
    sensitive test to perform the diagnosis
    (C) The diagnosis is clinical and tests are
    only necessary to rule out associated
    conditions
    (D) To accurately diagnose the condition, it
    is necessary to correlate clinical findings
    with MRI and sensory evoke potential
    tests
    (E) None of the above
A
  1. (C) The diagnosis of trigeminal neuralgia is
    eminently clinical and further tests are necessary
    only to rule out associated conditions.
    When the condition is found, MRI and evoked
    potential testing are strongly recommended to
    rule out secondary causes. Clinically the onset of
    trigeminal neuralgia is around the age of 50 years,
    more common in females, almost exclusively
    unilateral with a paroxysmal nature.
59
Q
390. Giant cell arteritis is characterized by which of
the following?
(A) Affects almost exclusively Asian
population
(B) As other forms of vasculitis, giant cell
arteritis commonly involves skin,
kidneys, and lungs
(C) Males are more commonly affected
(D) It is more common in older patients,
with a peak incidence between 60 to
75 years of age
(E) Visual loss is the presenting symptom in
over 50% of the patients
A
  1. (D) The giant cell arteritis affects almost exclusively
    the white population although it can occur in worldwide. Unlike other forms of vasculitis
    it rarely affects skin, kidneys, or lungs.
    Females are affected 3 times more often than
    males. Visual loss is now considered to affect
    between 6% to 10% of patients in most series.
60
Q
  1. According to the International Headache
    Society Diagnostic Criteria, analgesic rebound
    headache is
    (A) headache that resolves or reverts within
    2 weeks after discontinuation of the suspected
    medication
    (B) headache that worsens after intake of
    analgesics and reduces in intensity and
    frequency with reduction in the analgesic
    dose
    (C) the intensity of the headache decreases
    in intensity proportionally to the
    decrease in the dose of analgesic
    (D) headache greater than 15 days per
    month that has developed or markedly
    worsened during medication overuse
    (E) headache that increases in intensity with
    the use of morphine, most likely
    because of the cerebral vasodilation
    mediated by histamine release
A
  1. (D) Analgesic rebound headache resolves or
    reverts to its previous pattern within 2 months
    of discontinuing of the overused medication.
61
Q
  1. Cluster headaches are characterized by
    (A) lancinating unilateral headache that is
    commonly triggered by stress factors
    (B) the pain is strictly unilateral and autonomic
    symptoms occur ipsilateral to the
    pain
    (C) the onset is slow with progressive worsening
    of the pain over several hours
    with an attack usually lasting 3 to
    4 days
    (D) melatonin is commonly indicated as
    therapy for the acute attack
    (E) cluster headaches are more common in
    elderly patients
A
  1. (B) The first statement better describes trigeminal
    neuralgia. Cluster headache affects more
    males than females with a 5:1 ratio and can
    begin at any age. Attacks are severe, stabbing,
    screwing, unilateral pain, occasionally preceded
    by premonitory symptoms, with sudden
    onset, and rapid crescendo. Therapeutic interventions
    for the acute attack include oxygen,
    triptans, dihydroergotamine, ketorolac, chlorpromazine,
    or intranasal lidocaine, cocaine, or
    capsaicin. Melatonin has been found to be
    moderately effective as a preventive treatment
    in episodic and chronic cluster headache
62
Q
  1. Which of the following describes the pathophysiologic
    changes seen in migraine?
    (A) Inflammation of hypothalamic structures
    leads to low threshold stimulation
    of vascular and meningeal tissues
    (B) Central sensitization mediated by attribution
    to activation of β-fibers in the
    trigeminal system, mediates extracranial
    hypersensitivity
    (C) Large cerebral vessels, pial vessels, large
    sinuses, and the dura, are innervated by
    fibers originating from the sphenopalatine
    ganglion
    (D) Activation and threshold reduction of
    the trigeminocervical complex by its
    most caudal cells
    (E) In acute attacks, a marked reduction in
    vasoactive substances, including substance
    P, calcitonin gene related peptide (CGRP),
    and nitric oxide is commonly seen
A
  1. (D) Sterile neurogenic inflammation is often
    seen after stimulation of the trigeminal ganglion,
    which innervates large cerebral vessels,
    pial vessels, large sinuses, and the dura via
    unmyelinated C fibers. In acute attacks of
    migraine, substance P, CGRP, and nitric oxide
    mediate the neurogenic inflammation
63
Q
394. Which of the following is correct regarding
headache?
(A) Migraine is the most common form of
headache
(B) Tension-type headache (TTH) is commonly
aggravated by physical exercise
(C) The presence of nausea, vomiting,
photophobia, or phonophobia excludes
the diagnosis of TTH
(D) The most common form of migraine is
associated with aura
(E) Comorbid conditions associated with
chronic migraine include depression,
anxiety, and panic disorders
A
  1. (E) TTH is the most common type of headache.
    Aura is present in only 20% of patients suffering
    from migraine. Although chronic daily
    headache diagnostic criteria for probable TTH
    requires no nausea or vomiting as one of the
    criteria or absence of photophobia, or phonophobia,
    nausea may be seen in 4.2% of patients
    with TTH, while phonophobia is reported in
    10.6% of them.
64
Q
  1. Hundred precent oxygen inhalation is a safe
    and effective method for acute treatment of
    (A) chronic daily headache
    (B) TTH
    (C) migraine with aura
    (D) cluster headache
    (E) glossopharyngeal neuralgia
A
  1. (D) Inhalation of 100% oxygen at 7 to 12 L/min
    is effective in treating the majority of cluster
    headache sufferers when used continuously
    for 15 to 20 minutes. Generally oxygen inhalation
    is not considered to be effective in any
    other form of primary neurovascular headache.
65
Q
  1. The Ramsay Hunt syndrome is caused by the
    infection of the varicella-zoster virus of the
    (A) sphenopalatine ganglion
    (B) gasserian ganglion
    (C) geniculate ganglion
    (D) glossopharyngeal ganglion
    (E) stellate ganglion
A
  1. (C)
66
Q
  1. Which of the following characterizes the spontaneous
    intracranial hypotension (SIH)?
    (A) Is the same entity as post–dural puncture
    headache (PDPH)
    (B) Headache is consistently unilateral
    (C) Orthostatic headache is pathognomonic
    (D) Patients complain of bitemporal headache
    (E) To confirm the diagnosis, it is required
    that cerebrospinal fluid (CSF) opening
    pressures be below 60 mm H2O
A
  1. (C) PDPH and SIH are two distinct clinical entities
    with similar presentation. The headache is
    always bilateral, located in the occipital and/or
    frontal area. Although low CSF pressure is
    often noted, it is not necessary to confirm the
    diagnosis.
67
Q
  1. A 20-year-old male presents to the clinic with
    complaints of moderate headaches located
    bilateral in the forehead, parietal, and occipital
    areas. The pain is dull and continuous and not
    associated with nausea, vomiting, photophobia,
    and phonophobia. The patient recalls that
    the symptoms started 1 year ago and have been
    constant since they started. No abnormalities
    where observed on physical examination, sinus
    computed tomography (CT), or brain MRI. The
    patient has occasionally tried over-the-counter
    analgesics with no relief. Which of the following
    is the most likely diagnosis?
    (A) Status migrainosus
    (B) Rebound headache
    (C) New daily persistent headache
    (D) Cluster headache
    (E) Classical migraine
A
  1. (C) New daily persistent headache is a chronic,
    unremitting headache of sudden onset, daily
    pattern. The duration of the headache should be
    at least 3 months. Some important features
    include its moderate severity, bilateral location,
    and lack of nausea, vomiting (N/V), photophobia,
    and phonophobia (P/P). On the other hand,
    status migrainosus is a severe debilitating
    migraine, associated with N/V, P/P, and with
    duration longer than 72 hours but that typically
    do not exceed 2 weeks. The other diagnoses are
    not consistent with the symptoms.
68
Q
399. Which of the following is a theory that may
explain the presence of aura?
(A) Cortical spreading depression
(B) The vascular theory
(C) Hormonal fluctuation
(D) Estrogen withdrawal
(E) Cerebral idiopathic hypertension
A
  1. (A) The previously known classic migraine
    (migraine with aura) is preceded by visual aura
    that starts 20 to 40 minutes before the migraine
    and is characterized by spreading scintillations
    reflecting a slow propagation of neuronal and
    glial excitation emanating from one occipital
    lobe. Cortical spreading depression (CSD) presents
    with dramatic shifts in cortical steady
    potential (DC), temporary increases in extracellular
    ions and excitatory neurotransmitters
    (glutamate), and transient raise, followed by
    sustained decrease in cortical blood flow. The
    vascular theory proposed that migraine with
    aura is caused by intracranial cerebral vasoconstriction
    and the headache by reactive
    vasodilation. Despite that, the theory can not
    explain the prodromal symptoms or why some
    antimigraine medications are not effective.
    Hormonal fluctuations and estrogen withdrawal
    may explain the higher incidence of
    migraine in female patients during their reproductive
    years, but are not related to the presence
    of aura. Cerebral idiopathic hypertension
    is a form of headache of unknown etiology
69
Q
  1. Chronic low back pain and neck pain persists
    1 year or longer in what percentage of patients?
    (A) 5% to 10%
    (B) 15% to 20%
    (C) 20% to 25%
    (D) 25% to 60%
    (E) 60% to 75%
A
  1. (D) The published literature commonly states
    that 80% to 90% of low back pain resolves in
    about 6 weeks, irrespective of the administration
    or type of treatment, with only 5% to 10%
    of patients developing persistent back pain. Contrary to this assumption, actual analysis of
    research evidence shows that chronic low back and neck pain persist 1 year or longer in 25% to 60% of adult and/or elderly patients.
70
Q
401. The prevalence of zygapophysial (facet) joint
involvement in low back pain is
(A) 5% to 10%
(B) 10% to 15%
(C) 15% to 45%
(D) 50% to 60%
(E) 65% to 70%
A
  1. (C) Based on evaluations utilizing controlled diagnostic
    blocks, the prevalence of zygapophysial or
    facet joint involvement has been estimated to be
    between 15% and 45% in heterogeneous groups of
    patients with chronic low back pain.
71
Q
402. A 58-year-old with metastatic lung cancer suddenly
complains of severe back pain. Symptoms
of early spinal cord compression include all of the
following, EXCEPT
(A) rapid onset
(B) symmetric and profound weakness
(C) spasticity
(D) increased deep tendon reflexes
(E) urinary retention and constipation
A
  1. (E) The clinical picture of metastatic epidural
    spinal cord compression is uniformly reported
    as pain, weakness, sensory loss, and autonomic
    dysfunction. Metastatic epidural spinal cord
    compression initially presents with severe back
    pain in 95% of cases. After weeks of progressive
    pain, the patient may develop weakness, sensory
    loss, autonomic dysfunction, and reflex
    abnormalities. Bladder and bowel dysfunction
    are rarely presenting symptoms, but may
    appear after sensory symptoms have occurred.
    The exception to this generalization develops
    with compression of the conus medullaris,
    which presents as acute urinary retention and
    constipation without preceding motor or sensory
    symptoms
72
Q
  1. Specific indications for discography include all
    of the following, EXCEPT
    (A) further evaluation of abnormal discs to
    assess the extent of abnormality
    (B) patients with persistent, severe symptoms
    in whom other diagnostic tests
    have revealed clear confirmation of a
    suspected disc as the source of pain
    (C) assessment of patients who have failed
    to respond to surgical procedures to
    determine if there is possible recurrent
    disc herniation
    (D) assessment of discs before fusion to
    determine if the discs within the proposed
    fusion segment are symptomatic
    (E) assessment of minimally invasive surgical
    candidates to confirm a contained
    disc herniation or to investigate contrast
    distribution pattern before intradiscal
    procedures
A
  1. (B) Patients with severe, persistent symptoms
    (discogenic in origin) that have been confirmed
    by other diagnostic evaluations do not need to
    undergo further evaluation by discography.
    Specific uses for discography include, but are
    not limited to: further evaluation of demonstrably
    abnormal discs to help assess the extent of
    abnormality or correlation of the abnormality
    with clinical symptoms (in case of recurrent pain
    from a previously operated disc and a lateral
    disc herniation); patients with persistent, severe
    symptoms in whom other diagnostic tests have
    failed to reveal clear confirmation of a suspected
    disc as the source of pain; assessment of
    patients who have failed to respond to surgical
    procedures to determine if there is painful
    pseudoarthrosis or a symptomatic disc in a posteriorly
    fused segment, or to evaluate possible
    recurrent disc herniation; assessment of discs
    before fusion to determine if the discs within the
    proposed fusion segment are symptomatic and
    to determine if discs adjacent to this segment are normal; and assessment of minimally invasive
    surgical candidates to confirm a contained disc
    herniation or to investigate contrast distribution
    pattern before intradiscal procedures.
73
Q
  1. The following signs and symptoms are consistently
    found with cervical radiculopathy, EXCEPT
    (A) gait disturbances
    (B) normal muscle tone
    (C) negative Babinski test
    (D) weak tendon reflexes
    (E) positive axial compression test (Spurling
    maneuver)
A
  1. (A) Gait disturbances are a feature of cervical
    myelopathy, not radiculopathy. Other signs and
    symptoms of cervical radiculopathy include
    upper extremity sensory disturbances and muscle
    weakness.
74
Q
  1. All of the following are reasons associated with
    smoking as a risk factor for low back pain,
    EXCEPT
    (A) mineral content of the lumbar vertebrae
    is decreased
    (B) fibrinolytic disc activity is altered
    (C) blood flow and nutrition to the disc are
    diminished
    (D) disc pH is higher
    (E) increased degenerative changes of the
    lumbar spine
A
  1. (D) Experimental studies have given support to
    the hypothesis that blood flow and nutrition to
    the disc are diminished in smokers, the pH of
    the disc is lowered, disc mineral content is
    lower, fibrinolytic activity is changed, and there
    are increased degenerative changes seen in the
    lumbar spine.
75
Q
  1. All of the following treatments have strong evidence
    to back their use when treating acute
    low back pain, EXCEPT
    (A) muscle relaxants effectively reduce low
    back pain
    (B) bed rest is effective for treating low back
    pain
    (C) continuing normal activity gives equivalent
    or faster recovery from acute low
    back pain
    (D) NSAIDs prescribed at regular intervals
    are an effective treatment for acute low
    back pain
    (E) different types of NSAIDs are equally
    effective at treating low back pain
A
  1. (B) There is strong evidence from randomized
    controlled trials that bed rest is not effective
    for treating acute low back pain.
76
Q
  1. Age-related changes in the intervertebral discs
    include all of the following, EXCEPT
    (A) the dimensions of the lumbar intervertebral
    discs decrease with age
    (B) collagen lamellae of the annulus fibrosis
    increases in thickness
    (C) distinction between the nucleus pulposus
    and annulus fibrosis becomes less
    apparent
    (D) the nucleus pulposus is less able to
    transmit weight directly
    (E) 80% of nucleus pulposus cells in the elderly
    exhibit necrosis
A
  1. (A) Narrowing of the intervertebral discs has
    long been considered one of the signs of pathologic
    aging of the lumbar spine, but recent data
    has shown that notion to be untrue. Large-scale
    postmortem analysis have shown lumbar disc
    height and diameter to actually increase with
    age. The anterior-posterior diameter increases
    by about 10% in females and 2% in males. Disc
    height has been shown to increase by about
    10% in most lumbar discs.
77
Q
408. Radiculopathy is a neurologic condition associated
with all of the following characteristics,
EXCEPT
(A) numbness
(B) weakness
(C) pain
(D) compression of axons
(E) ischemia of axons
A
  1. (C) Radiculopathy is a condition in which conduction
    within the axons of a spinal nerve or its
    roots are blocked. It can result in numbness
    and weakness secondary to conduction block
    in sensory and motor neurons respectively.
    Conduction blockade can be caused by compression
    or ischemia. It is important to make
    the distinction that radiculopathy does not
    cause pain. It may, however, be associated with
    pain.
78
Q
  1. Adverse effects of epidurally administered
    steroids include all of the following, EXCEPT
    (A) Cushing syndrome
    (B) osteoporosis
    (C) avascular bone necrosis
    (D) hypoglycemia
    (E) suppression of the hypothalamus-pituitary
    axis
A
  1. (D) The major theoretical complications of corticosteroid
    administration include suppression
    of pituitary-adrenal axis, hypercorticism,
    Cushing syndrome, osteoporosis, avascular
    necrosis of bone, steroid myopathy, epidural lipomatosis, weight gain, fluid retention, and
    hyperglycemia.
79
Q
  1. Relative contraindications to epidural steroid
    injections include
    (A) preexisting neurologic disorder (ie, multiple
    sclerosis)
    (B) sepsis
    (C) therapeutic anticoagulation
    (D) localized infection at injection site
    (E) patient refusal
A
  1. (A) Absolute contraindications to epidural
    steroid injections include sepsis, infection at
    injection site, therapeutic anticoagulation, and
    patient refusal. Relative contraindications
    include preexisting neurologic conditions, prophylactic
    low-dose heparin, thrombocytopenia,
    and uncooperative patients.
80
Q
  1. L4-L5 disk herniation with L5 nerve root
    involvement includes
    (A) numbness over the medial thigh and knee
    (B) weakness with dorsiflexion of great toe
    and foot
    (C) difficulty walking on toes
    (D) pain in lateral heel
    (E) quadriceps weakness
A
  1. (B) L4-L5 disc herniation with L5 nerve root
    involvement involves: pain over the sacroiliac
    joint, hip, lateral thigh, and leg; numbness
    over the lateral leg and first three toes; weakness
    with dorsiflexion of great toe and foot;
    difficulty walking on heels; possible foot
    drop; and internal hamstring reflex diminished
    or absent. Numbness over the medial
    thigh and knee, and quadriceps weakness are
    indicative of L3-L4 disc herniation with L4
    nerve root involvement. Difficulty walking
    on toes and lateral heel pain are common
    with L5-S1 disk herniation involving the S1
    nerve root.
81
Q
412. In patients with chronic low back pain, the
prevalence of sacroiliac joint pain is
(A) 10%
(B) 15%
(C) 20%
(D) 25%
(E) 30%
A
  1. (B) Fifteen percent of patients with chronic low

back pain have sacroiliac joint pain.

82
Q
  1. Spondylolysis
  2. Spondylolisthesis
  3. Kissing spines
  4. Radiculopathy
  5. Radicular pain
    (A) Neurologic condition in which conduction
    is blocked to the axons of a spinal
    nerve or its roots. It results in numbness
    and weakness
    (B) An acquired defect caused by fatigue
    fracture of the pars interarticularis
    (C) Pain that arises as a result of irritation of
    a spinal nerve or its roots
    (D) Displacement of a vertebrae or the
    vertebral column in relationship to the
    vertebrae below it
    (E) Periostitis of spinous processes or
    inflammation of the affected ligament
A

413 to 417. 413 (B); 414 (D); 415 (E); 416 (A); 417 (C)
Spondylolysis is an acquired defect that results
from a fatigue fracture of the pars interarticularis
(the part of the lamina that intervenes
between the superior and inferior articular
processes on each side). Spondylolisthesis is
the displacement of a vertebrae or the vertebral
column in relationship to the vertebrae below.
Kissing spines (also known as Baastrup disease)
affects the lumbar spinous processes.
Excessive lumbar lordosis or extension injuries
to the lumbar spine cause adjacent spinous
processes to clash and compress the intervening
interspinous ligament. This results in a
periostitis of the spinous process or inflammation
of the affected ligament. Radiculopathy is
a neurologic condition in which conduction
blocks the axons of a spinal nerve or its roots
that results in numbness and weakness.
Radicular pain is pain that arises as a result of
irritation of a spinal nerve or its roots.

83
Q
  1. Evidence regarding the value of epidural injections
    for the management of chronic spinal
    pain demonstrates the following:
    (A) Limited with interlaminar lumbar
    epidural steroid injections for short-term
    relief of lumbar radicular pain
    (B) Strong with interlaminar lumbar
    epidural steroid injections for long-term
    relief of lumbar radicular pain
    (C) Moderate for lumbar transforaminal
    epidural steroid injections for short-term
    relief of lumbar radicular pain
    (D) Strong for lumbar transforaminal
    epidural steroid injections for long-term
    relief of lumbar radicular pain
    (E) Strong for caudal epidural steroid
    injections for short-term relief of lumbar
    radiculopathy and post–lumbar
    laminectomy syndrome
A
  1. (E) In managing lumbar radicular pain with
    interlaminar lumbar epidural steroid injections,
    the evidence is strong for short-term relief and
    limited for long-term relief. In managing cervical
    radiculopathy with cervical interlaminar
    epidural steroid injections, the evidence is moderate.
    The evidence for lumbar transforaminal
    epidural steroid injections in managing lumbar
    radicular pain is strong for short-term and moderate
    for long-term relief. The evidence for cervical
    transforaminal epidural steroid injections
    in managing cervical nerve root pain is moderate.
    The evidence is moderate in managing lumbar
    radicular pain in post–lumbar laminectomy syndrome.
    The evidence for caudal epidural steroid
    injections is strong for short-term relief and moderate
    for long-term relief, in managing chronic
    pain of lumbar radiculopathy and post–lumbar
    laminectomy syndrome.
84
Q
419. All of the following statements regarding intervertebral
disc innervation are true, EXCEPT
(A) nerve plexuses that innervate the intervertebral
discs are derived from dorsal rami
(B) in normal lumbar intervertebral discs,
nerve fibers are only found in the outer
third of the annulus fibrosis
(C) discs painful on discography and
removed with operation have nerve
growth deep into the annulus and into
the nucleus pulposus
(D) disc fissuring is a trigger for neoinnervation
of a disc
(E) the anterior and posterior nerve
plexuses accompany the anterior and
posterior longitudinal ligaments
A
  1. (A) The sources of the nerve endings in the
    lumbar discs are two extensive microscopic
    plexuses of nerves that accompany the anterior
    and posterior longitudinal ligaments. The nerve
    plexuses that innervate intervertebral discs are
    derived from the lumbar sympathetic trunks.
    The dorsal rami supply innervation to the muscles
    of the back and zygapophysial joints. In
    normal lumbar intervertebral discs, nerve fibers
    are only found in the outer third of the annulus
    fibrosis. Discs painful on discography and
    removed with operation have nerve growth
    deep into the annulus and into the nucleus pulposus.
    Disc fissuring is a trigger for neoinnervation
    and neovascularization of a disc.
85
Q
  1. Three days after a lumbar epidural steroid injection
    was given, a 57-year-old male complains
    of fever and severe back pain over the site where
    the injection was given. Two days later, the back
    pain has progressively worsened, and a severe
    radiating pain goes down the right leg and knee.
    Which of the following is the most likely complication
    of the epidural steroid injection?
    (A) Epidural abscess
    (B) Epidural hematoma
    (C) Arachnoiditis
    (D) Anterior spinal artery syndrome
    (E) Cauda equina syndrome
A
  1. (A) Development of an epidural abscess is a
    very rare complication of epidural steroid injections.
    It needs to be recognized and treated
    quickly to avoid irreversible injury. Symptoms
    of an epidural abscess include severe back pain
    that is followed by radicular pain 3 days later.
    The initial back pain may not become evident
    for several days after the injection.
86
Q
421. X-ray imaging is recommended for which of
the following cause of low back pain?
(A) Disc bulging
(B) Cauda equina syndrome
(C) Spondylolisthesis
(D) Lateral disc herniation
(E) Spinal cord tumors
A
  1. (C) Plain x-rays are recommended for possible
    fractures, arthropathy, spondylolisthesis, tumors,
    infections, stenosis, and congential deformities.
    CT images are recommended for bone/joint
    pathologies, lateral disc herniations, stenosis
    (ie, spinal canal, neuroforaminal, lateral recess),
    and for those in which an MRI is contraindicated.
    MRI is recommended for disc herniations,
    spinal stenosis, osteomyelitis, tumors (ie, spinal
    cord, nerve roots, nerve sheath, paraspinal soft
    tissue), and cauda equine syndrome.
87
Q
422. Which of the following nerve root and muscle
motion combinations is correct?
(A) L2—leg extension
(B) L3—heel walking
(C) L4—toe walking
(D) L5—first toe dorsiflexion
(E) S1—hip flexion
A
  1. (D) The L2 nerve root is involved with hip flexion,
    L3 with leg extension, L4 with heel walking,
    L5 with first toe dorsiflexion (and heel
    walking), and S1 with toe walking
88
Q
  1. Which of the following is the most frequent
    complication of a laminotomy with discectomy?
    (A) Recurrent disc herniation
    (B) Infection
    (C) Dural tear
    (D) Neural injury
    (E) Failed back surgery syndrome (FBSS)
A
  1. (C) Laminotomy with discectomy has a low infection
    rate, statistically. The most frequent complication
    is a dural tear. Neural injury may occur as
    a result of a dural tear and may cause long-term
    pain and neurologic deficit. Recurrence of the
    herniation occurs in approximately 5% of cases.
    Infection and neural injury occurs in less than
    0.5% of cases.
89
Q
424. Which of the following includes conservative
treatment for FBSS?
(A) Discectomy
(B) Chemonucleolysis
(C) Rehabilitation
(D) Laminectomy
(E) Fusion
A
  1. (C) Conservative treatment is usually the first
    treatment of choice for patients presenting with
    FBSS. It consists of medical management of
    contributing factors (ie, depression, obesity,
    smoking), rehabilitation, and behavior modification
    (ie, alcohol or drug dependency).
90
Q
  1. Favorable prognostic indicators for patients
    undergoing repeated lumbosacral surgery include
    all of the following, EXCEPT
    (A) female sex
    (B) satisfactory outcome from prior surgeries
    (C) operative findings of disk herniation
    (D) epidural scarring requiring lysis of
    adhesions
    (E) radicular pain
A
  1. (D) Many prognostic indicators have been
    implicated in patients undergoing repeat lumbosacral
    spine surgery. They may or may not be
    significant for each patient and should be taken
    into context for the particular patient. Women
    have been found to have better outcomes than
    men. Patients with a history of favorable outcomes
    from prior surgeries tend to have better
    outcomes as well. A history of few previous
    surgeries, operative/myelographic findings of
    disc herniation, and a history of working
    immediately prior to surgery are all favorable
    prognostic indicators. Less favorable prognostic
    indicators include epidural scarring that
    requires lysis of adhesions and pseudoarthrosis
    of a prior fusion.
91
Q
426. Waddell signs were developed to help identify
nonorganic causes of low back pain. They
include all of the following, EXCEPT
(A) tenderness
(B) stimulation
(C) distraction testing
(D) regional disturbance
(E) underreaction
A
  1. (E) Waddell signs are used to help diagnose
    nonorganic low back pain complaints. Each of
    the five findings is considered positive if present.
    Three positive findings are considered highly
    suggestive of a nonorganic source of pain:
  2. Tenderness: does not follow dermatomal or
    referral patterns and is hard to localize.
  3. Stimulation testing: stimulating distant
    sites should not cause discomfort.
  4. Distraction testing: findings when testing the
    same site are inconsistent when the patient’s
    attention is distracted.
  5. Regional disturbance: motor and sensory
    testing yield nonanatomic findings.
  6. Overreaction: inappropriate verbal remarks
    or facial expressions, withdrawal from touch,
92
Q
  1. A25-year-old male presents with progressively
    worsening neck and back pain and stiffness
    over 4 months that improves with light exercise
    and warm showers. Which of the following is
    the most likely diagnosis?
    (A) Rheumatoid arthritis
    (B) Ankylosing spondylitis
    (C) Psoriatic arthritis
    (D) Klippel-Feil syndrome
    (E) Reiter syndrome
A
  1. (B) Ankylosing spondylitis is characterized by
    pain and stiffness in young males (typically
    ages 17-35 years) more often than females. It is
    worse in the morning and improves with mild
    exercise. The pain will typically last for at least
    3 months and be diffuse in nature affecting the
    low back and spine. Rheumatoid arthritis is an
    inflammatory polyarthritis that affects middleaged
    women more often than men. It typically
    presents with morning stiffness that improves
    as the day progresses, and the spine is not
    affected until late in the disease. Psoriatic
    arthritis is characterized by inflammation of
    the skin and joints that typically presents in
    the fourth and fifth decades of life. Klippel-Feil
    syndrome is a congenital disorder that is characterized
    by abnormal fusion of two or more
    bones in the cervical spine. Reiter syndrome is
    a reactive arthritis that is characterized by a
    triad of symptoms: nongonococcal urethritis,
    conjunctivitis, and arthritis.
93
Q
428. Which of the following is a major criteria for
cervicogenic headache?
(A) Bilateral head or face pain without
sideshift
(B) Pain is superficial and throbbing
(C) Restricted neck range of motion
(D) Pain relief with digital pressure to
cervical vertebrae
(E) Lack of relief from anesthetic blockade
A
  1. (C) The three major criteria for cervicogenic
    headache include (1) signs and symptoms of
    neck involvement (precipitation of head pain
    by: neck movement and/or sustained awkward
    head positioning, by external pressure over the
    upper cervical or occipital region on the symptomatic
    side; restriction of the range of motion in
    the neck; ipsilateral neck, shoulder, or arm pain
    of a rather vague nonradicular nature or, occasionally,
    arm pain of a radicular nature); (2) confirmatory
    evidence by diagnostic anesthetic
    blockades; and (3) unilaterality of the head pain
    without sideshift. Head pain characteristics
    include moderate-severe, nonthrobbing, and
    nonlancinating pain, usually starting in the neck,
    episodes of varying duration, or fluctuating,
    continuous pain. Other characteristics of some
    importance: only marginal effect or lack of effect
    of indomethacin, only marginal effect or lack of
    effect of ergotamine and sumatriptan, female
    sex, not infrequent occurrence of head, or indirect
    neck trauma by history, usually of more
    than only medium severity.
94
Q
429. Neurogenic claudication can be distinguished
from vascular claudication by which of the
following?
(A) Leg tightness
(B) Pain alleviated with standing
(C) Pain exacerbated with lumbar flexion
(D) No change in pain with exercise
(E) Pain exacerbated with lying supine
A
  1. (D) Neurogenic claudication pain is secondary
    to nerve root compression rather than lack of
    blood supply that is seen with vascular claudication.
    The pain is exacerbated by standing
    erect and downhill walking. Improvement
    comes with lying supine more than lying in
    the prone position, sitting, squatting, and
    lumbar flexion. Neurogenic claudication is not
    made worse with biking, uphill walking, and
    lumbar flexion, unlike vascular claudication.
    It is not alleviated with standing.
95
Q
  1. Neck pain has been suggested to have a multifactorial
    origin. Which of the following statements
    regarding neck pain is true?
    (A) Workplace interventions are not effective
    at reducing neck pain
    (B) Normal degenerative changes in the
    cervical spine are a risk factor for pain
    (C) Physical activity does not protect
    against neck pain
    (D) Precision work does not increase the
    risk of neck pain
    (E) Social support in the workplace does
    not affect neck pain
A
  1. (A) Neck pain has been suggested to have multifactorial
    etiologies. Risk factors for neck pain
    that cannot be modified include age, sex, and
    genetics. There is no evidence that normal cervical
    spine degenerative changes are a risk
    factor for neck pain. Modifiable risk factors for
    neck pain include smoking and exposure to
    environmental tobacco. Participation in physical
    activity seems to offer a protective effect.
    High quantitative job demands, low social support
    at the workplace, inactive work position,
    repetitive work, and meticulous work increases
    the risk of neck pain. There is a lack of evidence
    that workplace interventions are successful
    in decreasing neck pain in employees.
96
Q
  1. In patients with neck pain, what is more
    predictive at excluding a structural lesion or
    neurologic compression than at diagnosing any
    specific etiologic condition?
    (A) MRI
    (B) Discography
    (C) Blood tests
    (D) Physical examination
    (E) Electrophysiology
A
  1. (D) In patients with neck pain, the physical
    examination is more predictive at excluding a
    structural lesion or neurologic compression
    than at diagnosing any specific etiologic condition
    in patients with neck pain. Other assessment
    tools (ie, electrophysiology, imaging,
    injections, discography, functional tests, and
    blood tests) lack validity and utility.
97
Q
  1. All of the following characteristics are associated
    with a poor prognosis for neck pain, EXCEPT
    (A) prior neck pain
    (B) pain resulting from an accident
    (C) passive coping techniques
    (D) middle age
    (E) compensation
A
  1. (B) Most people with neck pain do not experience
    a complete resolution of symptoms.
    Between 50% and 85% of those who experience
    neck pain at some initial point will report neck
    pain again 1 to 5 years later. These numbers
    appear to be similar in the general population, in
    workers, and after motor vehicle crashes. The
    prognosis for neck pain also appears to be multifactorial.
    Younger age was associated with a
    better prognosis, whereas poor health and prior
    neck pain episodes were associated with a
    poorer prognosis. Poorer prognosis was also
    associated with poor psychologic health, worrying,
    and becoming angry or frustrated in
    response to neck pain. Greater optimism, a
    coping style that involved self-assurance, and
    having less need to socialize, were all associated
    with better prognosis. Specific workplace or
    physical job demands were not linked with
    recovery from neck pain. Workers who engaged
    in general exercise and sporting activities were
    more likely to experience improvement in neck
    pain. Postinjury psychologic distress and passive
    types of coping were prognostic of poorer
    recovery in WAD. There is evidence that compensation
    and legal factors are also prognostic
    for poorer recovery from WAD.
98
Q
433. Which of the following is the most common
complication of fluoroscopically guided interlaminar
cervical epidural injections?
(A) Nonpositional headache
(B) Vasovagal reactions
(C) Increased neck pain
(D) Fever
(E) Dural puncture
A
  1. (C) The reported complications of fluoroscopically
    guided interlaminar cervical epidural
    injections are increased neck pain (6.7%), nonpositional
    headaches (4.6%), insomnia the night
    of the injection (1.7%), vasovagal reaction reactions
    (1.7%), facial flushing (1.5%), fever on the
    night of the procedure (0.3%), and dural puncture
    (0.3%). The incidence of all complications
    per injection is 16.8%.
99
Q
  1. A 54-year-old female complains suddenly of
    inability to move her legs after a transforaminal
    epidural steroid injection. On further examination,
    she is found to have intact light touch
    sensation, sphincter disturbance, and loss of
    pain and temperature sensation. What is the
    most likely diagnosis?
    (A) Cauda equina syndrome
    (B) Epidural hematoma
    (C) Epidural abscess
    (D) Transient paraplegia
    (E) Anterior spinal artery syndrome
A
  1. (E) Anterior spinal artery syndrome classically
    presents in older patients with abrupt motor
    loss, sphincter disturbance, and nonconcordant
    sensory examination with preservation of sensation
    to light touch but loss of pain and temperature.
    It may also occur during aortic
    procedures. When anterior spinal artery syndromes
    occur during or after transforaminal
    epidural steroid injection, the patient may have
    abrupt back or abdominal pain after injection.
    An MRI will demonstrate a T2 signal change
    consistent with cord ischemia/infarct. Anterior
    spinal artery ischemia may be caused by arteriosclerosis, tumors, thrombosis, hypotension, air or fat embolism, toxins, or other causes. Particulate (steroid) substances, arterial injury, or vascular spasm are other potential causes and have been implicated as significant possibilities for the occurrence of ischemic events after transforaminal
    epidural steroid injections.
100
Q
  1. A57-year-old diabetic male presents with a new
    onset of neck pain over the past several hours;
    the pain is beginning to move down each arm
    equally. Two days ago he had a cervical epidural
    injection which he receives periodically for a herniated
    disc. On physical examination, his temperature
    is 102.4°F, his cervical spine is exquisitely
    tender to palpation and he complains of radicular
    pain down both arms. The most likely organism
    causing this presentation is
    (A) Pseudomonas
    (B) Escherichia coli
    (C) Streptococcus pneumoniae
    (D) Hemophilus influenza
    (E) Staphylococcus aureus
A
  1. (E) A spinal epidural abscess must be recognized
    promptly and treated quickly, otherwise
    extreme morbidity can result. It may be separate
    or associated with vertebral osteomyelitis.
    Diabetic, alcoholic, IV drug using patients, and
    immunocompromised patients are all at
    increased risk. Staphylococcus aureus is the most
    common organism involved. An affected patient
    usually presents with neck pain that rapidly
    progresses to radicular symptoms. Quadriplegia
    can result if left untreated. Treatment involves
    surgical removal and antibiotic management.
101
Q
  1. The following statements are true regarding
    the pathologic mechanism in HIV-related neuropathy,
    EXCEPT
    (A) HIV is found within endoneurial
    macrophages
    (B) HIV is found within Schwann cells
    (C) antisulfatide antibodies are one of the
    humoral factors responsible for
    demyelinating diseases in AIDS patients
    (D) secretion of cytokines by the HIVinfected
    glial cells may generate tissuespecific
    autoimmune attack
    (E) the pathologic mechanisms in HIV-related
    neuropathies are not well understood
A
  1. (B) The pathophysiology of HIV-related neuropathies
    is still not well understood. The current
    understanding is that it is not related to the direct
    effect of the virus itself. HIV is not found within
    ganglionic neurons of Schwann cells, but only in
    endoneurial macrophages, which may generate
    a tissue-specific autoimmune response by secretion
    of cytokines, which, in turn, promotes trafficking
    of activated T cells and macrophages
    within the endoneurial parenchyma.
102
Q
  1. Pain syndromes of neuropathic nature occur
    in approximately 40% of AIDS patients with
    pain. Several types of peripheral neuropathies
    have been described in patients with HIV and
    AIDS. The most common painful neuropathy
    encountered in patients with HIV and AIDS is
    (A) mononeuritis multiplex
    (B) polyradiculopathy
    (C) cauda equina syndrome
    (D) painful toxic neuropathy
    (E) predominantly sensory neuropathy of
    AIDS
A
  1. (E) The predominantly sensory neuropathy of
    AIDS affects up to 30% of people with HIV
    infection and AIDS and is the most commonly
    encountered.
103
Q
  1. The most important pathophysiologic event
    in sickle cell anemia, which explains most of
    its clinical manifestations, is vascular occlusion.
    The following are the pathophysiologic
    processes that lead to vascular occlusion in
    patients with sickle cell disease (SCD), EXCEPT
    (A) erythrocyte dehydration
    (B) distortion of the shape of erythrocytes
    (C) polymerization of the sickle cell hemoglobin
    on deoxygenation
    (D) decreased deformability of erythrocytes
    (E) decreased stickiness of erythrocytes
A
  1. (E) The primary process that leads to vascular
    occlusion is the polymerization of sickle cell
    hemoglobin on deoxygenation, which in turn
    results in distortion of the shape of red blood
    cells (RBCs), cellular dehydration, decreased
    deformability, and increased stickiness of RBCs,
    which promotes their adhesion to and activation
    of the vascular endothelium.
104
Q
  1. Aphysician has to exercise extra caution when
    attributing SCD patient’s complaints of pain
    to behavioral deviations, such as drug-seeking
    behavior, because
    (A) patients in real pain, such as sickle cell
    pain, do not develop addiction to opioids
    (B) most patients with SCD have substance
    abuse and addiction, as they are
    exposed to opioids early in life
    (C) there is a higher incidence of controlledsubstance
    diversion in SCD patients
    (D) sickle cell pain could be the prodrome
    of a serious and potentially fatal complication
    of SCD
    (E) severe pain, such as sickle cell pain,
    should only be managed by an experienced
    physician subspecializing in pain
    management
A
  1. (D) SCD is unlike other pain syndromes where
    the provider can make decisions on treatment
    based solely on the pain and its associated
    behavior. A primary care physician, for example,
    taking care of a middle-aged patient with
    job-related low back pain may decide to expel the patient from his or her care if the patient in
    question demonstrates suspicious drug-seeking
    behavior. Doing the same with patients who
    have SCD could be counterproductive. There
    are anecdotes of patients with SCD who were
    dismissed from certain programs only to be
    found dead at home within 24 hours after dismissal
    or to be admitted to other hospitals with
    serious complications. Sickle cell pain could be
    the prodrome of a serious and potentially fatal
    complication of SCD in some patients.
105
Q
  1. What makes the pain of SCD unique in its
    acuteness and severity?
    (A) SCD patients tend to have a decreased
    threshold to pain because of prolonged
    and early exposure to severe pain in life
    (B) SCD patients have increased tolerance to
    opioids and opioid-related hyperalgesia
    (C) SCD pain pathophysiology involves a
    combination of ischemic tissue damage
    and secondary inflammatory response
    (D) Repetitive SCD crises lead to ischemic
    damage of the CNS and subsequent
    central sensitization to pain
    (E) SCD patients tend to anticipate and
    respond with a spectacular behavioral
    manifestation to pain, because of its
    cyclic feature
A
  1. (C) Tissue ischemia caused by vascular occlusion
    resulting from in situ sickling causes
    infarctive tissue damage, which in turn initiates
    a secondary inflammatory response. The secondary
    response may enhance sympathetic
    activity by means of interactions with neuroendocrine
    pathways and trigger release of
    norepinephrine. In the setting of tissue injury,
    this release causes more tissue ischemia, creating
    a vicious cycle. It is the combination of
    ischemic tissue damage and secondary inflammatory
    response that makes the pain of SCD
    unique in its acuteness and severity.
106
Q
  1. At initial presentation, objective signs of a
    painful SCD crisis, such as fever, leukocytosis,
    joint effusions, and tenderness, occur in
    (A) less than 10% of patients
    (B) about 25% of patients
    (C) about 50% of patients
    (D) about 75% of patients
    (E) more than 90% of patients
A
  1. (C) Objective signs of a painful crisis, such as
    fever, leukocytosis, joint effusions, and tenderness,
    occur in about 50% of patients at initial
    presentation.
107
Q
  1. What percentage of hospital admissions in adult
    SCD patients result from acute sickle cell pain?
    (A) Less than 10%
    (B) About 25%
    (C) About 50%
    (D) About 75%
    (E) More than 90%
A
  1. (E) Pain is the hallmark of SCD, and the acute
    sickle cell painful episode (painful crisis) is the
    most common cause of more than 90% of hospital
    admissions among adult patients who
    have SCD.
108
Q
443. Which of the following is true regarding treatment
of sickle cell pain with NSAIDs?
(A) They should be completely avoided
because of potential side effects
(B) They should not be administered
continuously for more than 5 days
(C) They should be administered only in
combination with opioids
(D) They should not be administered
continuously for more than 1 month
(E) Potential morbidity from their side
effects in SCD patients is the same as in
the general population
A
  1. (B) NSAIDs have potentially serious, systemic
    adverse effects. They include gastropathy,
    nephropathy, and hemostatic defects. It is
    advisable not to administer them continuously
    for more than 5 days to patients with SCD.
109
Q
  1. Pharmacologic management of SCD pain
    includes three major classes of compounds:
    nonopioids, opioids, and adjuvants. Nonopioids
    include acetaminophen, NSAIDs, topical agents,
    tramadol, and corticosteroids. The following is
    true about Tramadol, EXCEPT
    (A) it inhibits neuronal reuptake of
    serotonin and norepinephrine
    (B) it acts as a weak μ-receptor agonist
    (C) it does not have a “ceiling” effect
    because of its safe side-effect profile
    (D) it is not associated with an addiction
    potential
    (E) it is a centrally acting analgesic
A
  1. (D) Tramadol is a synthetic, centrally acting
    analgesic that is not chemically related to opioids.
    It acts as a weak agonist with preferential affinity
    to the μ-receptors. Moreover, it inhibits
    neuronal reuptake of both serotonin and norepinephrine
    and stimulates the release of serotonin.
    Thus, it has functional properties of an opioid and an antidepressant. This drug
    received an initial enthusiastic reception based
    on the perception that it was not associated with
    clinically significant respiratory depression or
    addiction potential. However, this enthusiasm
    waned after reports indicated that seizures may
    be an adverse effect and that abuse potential is increasing.
110
Q
  1. All of the following are true about chronic pain
    in the spinal cord injury (SCI) patient, EXCEPT
    (A) approximately two-thirds of all SCI
    patients suffer from chronic pain
    (B) approximately one-third of SCI patients
    with pain have severe pain
    (C) pain in SCI patients may lead to severe
    depression and even suicide
    (D) because of the overwhelmingly significant
    impairment of other important
    functions, pain is only a minor consideration
    in an SCI patient
    (E) pain in SCI interferes with rehabilitation
    and activities of daily living (ADLs)
A

(D) Chronic pain is a major complication of
SCI. Epidemiologic studies indicate that
approximately two-thirds of all SCI patients
suffer from chronic pain out of which one-third
have severe pain. Pain interferes with rehabilitation,
daily activities, quality of life, and may
have significant influence on mood leading to
depression and even suicide.

111
Q
446. In an SCI patient, chronic pain secondary to
overuse is common in
(A) neck
(B) lower back
(C) shoulders and arms
(D) hips and thighs
(E) knees and feet
A
  1. (C) Musculoskeletal pain is common in both
    the acute and chronic phase of SCI. Chronic
    pain secondary to overuse is common in shoulders
    and arm, and vertebral column pain may
    occur because of the secondary changes following
    fractures and fixation, mechanical instability,
    and osteoporosis.
112
Q
447. Autonomic dysreflexia usually occurs after an
SCI at
(A) any level
(B) above C4
(C) above C7
(D) above T6
(E) above L1
A
  1. (D) Autonomic dysreflexia may complicate SCI
    patients with a lesion above the splanchnic outflow
    (sixth thoracic level).
113
Q
448. The following is true regarding the visceral pain
in an SCI patient, EXCEPT
(A) it is unlikely that visceral pain may
occur in the absence of any abdominal
organ dysfunction
(B) the pattern of visceral pain is not
affected in an SCI patient, because it is
transmitted through the sympathetic
system, which usually bypasses the site
of injury
(C) autonomic dysreflexia cannot be
triggered by visceral pain
(D) visceral pain is always present in an SCI
patient as part of the central pain
syndrome
(E) increases in spasticity or autonomic
reactions may be the only indications of
abdominal organ dysfunction
A
  1. (E) Visceral pain usually presents as dull or
    cramping abdominal uncomfortable and
    painful sensations, which may be associated
    with nausea and autonomic reactions. It is
    likely that visceral pain may occur in the
    absence of any abdominal organ dysfunction,
    and may in some cases represent a neuropathic
    type of pain. SCI patients may not have the
    typical signs of abdominal illness, and they
    should be carefully examined whenever any
    new pain or changes in existing pain occur.
    Increases in spasticity, pain at any location, or
    autonomic reactions may be the only indications
    of abdominal organ dysfunction.
114
Q
  1. Neuropathic pain in SCI is divided into abovelevel,
    at-level, and below-level types. Depending
    on the type of pain, nerve root injury (peripheral
    component), and/or SCI (central component)
    may contribute to the pain. Which of the following
    is true?
    (A) Below-level pain has only peripheral
    component
    (B) At-level pain has only peripheral component
    (C) Below-level pain is usually caused by
    compressive mononeuropathy
    (D) Below-level pain is usually caused by
    CRPS
    (E) At-level pain may have both peripheral
    and central components
A
  1. (E) Above-level neuropathic pain includes pain
    caused by compressive mononeuropathies
    (particularly carpal tunnel syndrome) and
    CRPS. While below-level pain is considered to
    be a central pain caused by the spinal cord trauma, at-level pain may have both peripheral
    (nerve root) and central (spinal cord) components
    that are difficult to separate.
115
Q
  1. One of the characteristics of stimulus-evoked
    neuropathic pain in SCI can be temporal summation
    of pain. Temporal summation of pain is
    defined as
    (A) elicitation of pain by nonnoxious
    stimulation
    (B) pain continuing after stimulation has
    ceased
    (C) an increased pain response to a noxious
    stimulus
    (D) abnormal increase in pain with each
    repetitive stimulation
    (E) pain felt in a place apart from the
    stimulated area
A
  1. (D) Answers (A), (B), (C), and (D) define allodynia,
    aftersensation, hyperalgesia, and referred
    pain, respectively.
116
Q
  1. An axonal injury triggers a Wallerian degeneration,
    which is defined as
    (A) degeneration of the portion of the axon
    separated from the neuronal body by
    the injury
    (B) degeneration of the injured neuron distal
    and proximal to the level of injury
    (C) atrophy of the motor unit supplied by
    the injured neuron
    (D) dying of the body of the neuron, which
    lost its axon
    (E) degeneration of the secondary afferent
    neuron because of the absence of the
    input from the injured primary afferent
    neuron
A
  1. (A)
117
Q
  1. Which of the following is true about the central
    cord syndrome?
    (A) It is the injury of the mid-portion of the
    spinal cord, usually around T6 level
    (B) Upper extremities are affected more
    than lower
    (C) It is very uncommon
    (D) Patient usually presents with absent
    perianal sensation
    (E) It is usually associated with complete SCI
A
  1. (B) Central cord syndrome is one of the incomplete
    SCI syndromes. It is the most common
    pattern of injury, representing central gray
    matter destruction with preservation of only
    the peripheral spinal cord structures, the sacral
    spinothalamic and corticospinal tracts. The
    patient usually presents as a quadriplegic with
    perianal sensation and has an early return of
    bowel and bladder control. Any return of
    motor function usually begins with the sacral
    elements (toe flexors, then the extensors), followed
    by the lumbar elements of the ankle,
    knee, and hip. Upper extremity functional
    return is generally minimal and is limited by
    the degree of central gray matter destruction.
118
Q
  1. A common sensory symptom in patients with
    CRPS is hyperpathia which may be defined as
    (A) normally innocuous stimuli are perceived
    as painful
    (B) exaggerated pain perception after a noxious
    stimulus at the site of injury
    (C) exaggerated pain perception after a noxious
    stimulus in the area surrounding
    the primary affected skin
    (D) exaggerated delayed painful perception
    after a noxious stimulus
    (E) unpleasant abnormal sensation, whether
    spontaneous or evoked
A
  1. (D) Sensory symptoms and signs in CRPS
    include spontaneous pain, hyperpathia, allodynia,
    and hyperalgesia. Answer (A) is the definition
    of allodynia, answer (B) defines primary
    hyperalgesia, and answer (C) is secondary
    hyperalgesia. Answer (E) is the definition of
    dysesthesia. Dysesthesia maybe spontaneous.
119
Q
  1. Common findings in patients with CRPS I
    include
    (A) symmetrical distal extremity pain
    (B) pain intensity that is usually proportional
    to the intensity of the initiating
    event
    (C) nearly all patients with CRPS I having
    sweating abnormalities
    (D) sensory abnormalities that are most
    often proximal
    (E) consistency between the inciting lesion
    and the spatial distribution of the pain
A
  1. (C) CRPS I is a painful condition following an
    injury, which may not even be a neuropathic
    pain, as not obvious lesion is present. Patients
    with this condition develop asymmetrical distal
    extremity pain, which is disproportionate to the
    intensity of the initiating event. Sensory abnormalities
    appear early in the course of the disease
    and are more pronounced distally. No clear relationship
    between the injury and the area of pain
    distribution exist. Sweating abnormalities,
    whether hypohidrosis or hyperhidrosis are
    present in nearly all patients with CRPS I.
120
Q
455. Late changes observed in patients with CRPS I
include
(A) sensory abnormalities
(B) warm extremities
(C) distal swelling
(D) trophic changes
(E) increased dermal blood flow
A
  1. (D) Trophic changes, particularly abnormal
    hair growth, fibrosis, decreased dermal blood
    flow, thin glossy skin, and osteoporosis are more common in long standing cases, while
    the others described present in acute phases of
    the disease.
121
Q
  1. Which of the following is true in relation to
    CRPS?
    (A) Males are more commonly affected than
    females
    (B) CRPS II is more common than CRPS I
    (C) Three bone scan showing unilateral
    periarticular uptake is mandatory to
    confirm CRPS diagnosis
    (D) The diagnosis of CRPS is mainly clinical
    (E) The mean age group is between 15 and
    25 years
A
  1. (D) The diagnosis of CRPS I and II follows the
    IASP clinical criteria. Bone scintigraphy may be
    a valuable tool to rule out other conditions.
    CRPS I is more common than CRPS II and the
    female to male ratio is from 2:1 to 4:1.
122
Q
  1. Which of the following is the diagnostic criteria
    that differentiates CRPS II from CRPS I?
    (A) Triple-phase bone scan showing diffuse
    spotty osteoporosis
    (B) Weakness of all muscles of the affected
    extremity
    (C) Sweating abnormalities
    (D) Lesion of a peripheral nerve structure is
    mandatory
    (E) Paresis
A
  1. (D) The symptoms of CRPS II are similar to
    those of CRPS I, except that in CRPS II, there
    must be a lesion of a peripheral nerve structure
    and subsequent focal deficit are mandatory for
    the diagnosis.
123
Q
  1. Patients with CRPS exhibit significant psychologic
    findings, including
    (A) the most common psychiatric comorbidities
    are anxiety and depression
    (B) increased incidence of somatization in
    patients with CRPS than to patients
    with chronic low back pain
    (C) psychiatric problems are the cause of
    CRPS
    (D) CRPS is a psychogenic condition
    (E) maladaptive behaviors in CRPS patients
    indicate the presence of psychopathology
A
  1. (A) The majority of patients with CRPS present
    with significant psychologic distress, being the
    most frequent anxiety and depression. Current
    evidence is against the theory that CRPS is a
    psychogenic condition. The pain in CRPS is the
    cause of psychiatric problems and not vice
    versa. When compared to patients with low
    back pain, CRPS patients showed a higher frequency
    of somatization, but other psychologic
    parameters were similar.
124
Q
  1. The Lewis triple response consists of the following,
    EXCEPT
    (A) activation and sensitization of cutaneous
    C fibers elicit local edema
    (B) reddening of the skin at the site of the
    stimulus
    (C) spreading flare
    (D) local peripheral vasoconstriction mediated
    by the release of substance P
A
  1. (D) Activation and sensitization of cutaneous C
    fibers elicit a response consisting of a wheal
    (local edema), a reddening of the skin at the site
    of stimulus, and a spreading flare, were the
    responses described by Lewis (1927). [Lewis T.
    The blood vessels of the human s
125
Q
  1. The diagnosis of myofascial pain syndrome is
    confirmed when
    (A) the myofascial trigger point is identified
    by palpation
    (B) a patient has a widespread muscle
    condition
    (C) a patient is diagnosed with fibromyalgia
    first
    (D) regional muscle spasms are noted
    (E) none of the above
A
  1. (A) Muscle pain tends to be dull, poorly localized,
    and deep in contrast to the precise location
    of cutaneous pain. The diagnosis of
    myofascial pain syndrome is confirmed when
    the myofascial trigger point is identified by
    palpation. An active myofascial trigger point is
    defined as a focus of hyperirritability in a
    muscle or its fascia that causes the patient pain.
    B. Myofascial pain syndrome is usually
    thought of as a regional pain syndrome in
    contrast to fibromyalgia as a widespread
    syndrome; however, as many as 45% of
    patients with chronic myofascial pain syndrome
    have generalized pain in three or four
    quadrants. Hence, regional pain syndromes should raise a suspicion of myofascial pain
    syndrome, but patients with widespread
    musculoskeletal pain can also have
    myofascial pain syndrome.
    C. The American Pain Society showed general
    agreement with the concept that myofascial
    pain syndrome exists as an entity distinct
    from fibromyalgia.
    D. Systemic palpation differentiates between
    myofascial
126
Q
  1. Four experienced physicians examine a patient.
    They all identify the same precise localization
    of trigger points within a muscle. The minimum
    criteria that must be satisfied in order to
    distinguish a myofascial trigger point from any
    other tender area in muscle are
    (A) a taut band and a tender point in that
    taut band
    (B) a local twitch response
    (C) referred pain
    (D) reproduction of the person’s symptomatic
    pain
    (E) A and C
A
  1. (A) The minimum criteria that must be satisfied
    in order to distinguish a myofascial trigger
    point from any other tender area in muscle are
    a taut band and a tender point in that taut
    band. The existence of a local twitch response,
    referred pain, or reproduction of the person’s
    symptomatic pain increases the certainty and
    specificity of the diagnosis.
127
Q
  1. A23-year-old female is found to have a trigger
    point in the left trapezius muscle. With regard
    to electrical characteristics of the trigger point,
    which of the following is false?
    (A) A characteristic electrical discharge
    emanates from the trigger point
    (B) Spontaneous EMG activity typical of
    end-plate noise occurring in myofascial
    trigger points has been further confirmed
    in a study of young subjects with
    chronic shoulder and arm pain
    (C) The sympathetic nervous system does
    not have a modulating effect on the
    motor activity of the trigger point
    (D) End-plate noise without spikes was
    found at trigger point sites to a significantly
    greater degree than at end-plate
    zones outside of trigger points, and not
    at all in taut band sites outside of an
    end-plate zone
    (E) All the above statements are true
A
  1. (C)
    D. This statement is true. End-plate noise is
    characteristic of, but not restricted to, the
    region of the myofascial trigger point.
    Hence, an objective EMG signature of the
    trigger point is now available for diagnostic
    and research purposes.
128
Q
  1. Manual therapy is one of the four basic treatment
    options used for inactivating trigger
    points. Some practitioners also incorporate the
    stretch and spray technique. It is therapeutic
    (A) because like with other soothing sprays,
    the placebo effect is extremely high
    (B) because the vapocoolant spray stimulates
    thermal and tactile A-β skin receptors,
    thereby inhibiting C fiber and A-δ
    fiber afferent nociceptive pathways and
    muscle spasms, myofascial trigger
    points, and pain when stretching
    (C) because the vapocoolant is focused
    specifically on the trigger point
    (D) because therapists don’t have to be as
    liberal when passively stretching
    patients
    (E) B and D
A
  1. (B)
    A. There is no reason to believe that the
    placebo effect is unusually high with this
    or other soothing sprays.
    B. The stretch and spray technique combines
    the use of a vapocoolant spray with passive
    stretching of the muscle. Application of
    vapocoolant spray stimulates temperature
    and touch A-β skin receptors, thereby inhibiting
    C fiber and A-δ fiber afferent nociceptive
    pathways and muscle spasms, myofascial
    trigger points, and pain when stretching
    C. The patient is positioned comfortably and
    the muscle involved is sprayed with a
    vapocoolant spray, and then the muscle is
    stretched passively. With the muscle in the
    stretched position, the spray is applied again
    over the skin overlying the entire muscle,
    starting at the trigger zone and proceeding in the direction of, and including, the
    referred pain zone. After, the area is heated
    with a moist warm pack for 5 to 10 minutes.
    The patient is encouraged to perform full
    range of motion exercises with the body
    part. This technique can be used in physical
    therapy as a separate modality or following
    myofascial trigger point injections.
    D. It is expected that therapists can be more
    liberal once the spray has been applied
    owing to less patient discomfort.
129
Q
464. Which of the following is not an effective
myofascial release technique?
(A) Strumming
(B) Perpendicular and oscillating
mobilizations
(C) Therapeutic ultrasound
(D) Connective tissue massage
(E) Pétrissage
A
  1. (C) Myofascial release techniques and sustained
    pressure may soften and relax contracted
    and hardened muscles. The principle of
    the least possible force is applied, instead of
    applying high stress to the muscle. Effective
    myofascial release techniques include strumming.
    Strumming is when a finger runs across
    a taut band at the level of the trigger point over
    the nodules from one side of the muscle to the
    other. The operator’s fingers pull perpendicularly
    across the muscle rather than along the
    length of the fibers. When the nodule of the
    trigger point is encountered, light pressure is
    maintained until the operator senses tissue
    release. Other techniques include perpendicular
    and oscillating mobilizations, tissue rolling,
    connective tissue massage, and deep muscle
    massage consisting of effleurage (stroking massage
    technique) and pétrissage (kneading massage
    technique).
    After superficially passing over the muscles
    and adjacent muscles, massage therapy
    can be applied directly to the taut band and
    trigger points. Exercise and massage helped to
    reduce the number and intensity of trigger
    points, but the addition of therapeutic ultrasound
    did not improve the outcome.
130
Q
465. Trigger points can theoretically be related to
underlying articular dysfunction. Joint and
muscular dysfunction is closely related and
should be considered as a single functional
unit. It has been noted that
(A) restrictions in joint capsules inhibit
function for those muscles overlying
the particular joint, but muscle
dysfunction does not result in joint
capsule restrictions
(B) restrictions in joint capsules do not
inhibit adjacent muscles, nor does
muscular dysfunction result in joint
capsule restrictions
(C) restrictions in joint capsules inhibit muscle
function for those muscles overlying
the particular joint. Conversely, muscle
dysfunction results in joint capsule
restrictions
(D) restrictions in joint capsules do not
severely limit the overlying muscles;
however, muscle dysfunction does
regulate joint capsule range of motion
(E) muscle dysfunction does not result in
joint capsule range of motion
A
  1. (C) When treating myofascial pain, the physician
    must evaluate and, if indicated, treat both soft
    tissue and joint dysfunction. Restrictions in joint
    capsules hinder the function of the muscles that
    overlie the joint, while muscle irregularities result
    in joint capsule restrictions. Zygapophyseal joints
    may have pain referral patterns that are analogous
    to myofascial trigger points. The limited
    range of motion and weakness that results from this somatic dysfunction that affects muscles and joints can be reversed easily by manual therapy.
131
Q
  1. An administrative assistant presents with a
    number of upper back trigger points. She is scheduled
    for dry needling. What is the most common
    indication for this therapeutic modality?
    (A) Relief of an acute myofascial pain syndrome
    (B) To identify a myofascial trigger point as
    the cause of a particular pain
    (C) To eliminate a trigger point permanently
    (D) Inactivation of myofascial trigger points
    to facilitate physical therapy
    (E) None of the above
A
  1. (D) Answers A., B., and D. are the therapeutic,
    diagnostic, and adjunctive indications for
    myofascial trigger point needling, respectively,
    with D. being the most common appropriate
    use of this technique. Rarely is dry needling
    done to eliminate a trigger point permanently,
    although, this can happen when the myofascial
    pain syndrome is acute.
    Inactivation of the myofascial trigger
    point appears to be the result of the mechanical
    action of the needle in the trigger point
    itself, because it also occurs when no medication
    is used. However, using local anesthetics
    is more comfortable for the patients and
    results in longer lasting pain reduction.
    After pinpointing and manually stabilizing
    the trigger point in the taut band with the fingers,
    the needle is quickly passed through the skin and
    into the trigger zone. Alocal twitch response or
    a report of referred pain indicates that the trigger
    zone has been entered. One-tenth to 0.2 mL
    of local anesthetic can be injected. The needle
    is pulled back to just below the skin, the angle is
    changed, and it is once again inserted through
    the muscle to another trigger zone. In this way a
    funnel-shaped volume of muscle can be evaluated
    without withdrawing the needle through the
    skin. The trigger zone is explored this way until
    no further local twitch responses are obtained. By
    this time, the taut band is usually gone and the
    spontaneous pain of the trigger point has subsided.
    Experienced patients know when trigger
    points have been inactivated.
132
Q
  1. Randomized, double-blind, controlled studies
    have shown that adding which of the following
    medications to local anesthetics increases the
    pain relief obtained from myofascial trigger
    point injections?
    (A) Steroids
    (B) Ketorolac
    (C) Vitamin B12
    (D) Diphenhydramine
    (E) None of the above
A
  1. (E)
    A., B., and C. While direct needling of myofascial
    trigger points appears to be an effective
    treatment, there is insufficient evidence that
    needling therapies have efficacy beyond
    placebo. These researchers also found no
    evidence to suggest that the injection of one
    material was more effective than another.
    They found no advantage to adding steroids,
    ketorolac, or vitamin B12 to local anesthetic.
    Steroids actually have the disadvantage
    that they are locally myotoxic and that repeated administration can produce all the
    unwanted side effects associated with
    steroids. For those who are allergic to local
    anesthetics, saline or dry needling can be
    used.
    D. No studies have been done to confirm or
    refute that diphenhydramine increases the
    efficacy of myofascial trigger points.
133
Q
468. Which of the following is not a complication of
trigger point injections?
(A) Local hemorrhage into muscle
(B) Infection
(C) Transient nerve block
(D) Syncope
(E) Torticollis
A
  1. (E) Complications of trigger point injections:
    • Local bleeding into muscle
    • Local swelling
    • Painful contraction of a taut band from
    inadequate myofascial trigger point inactivation
    (missing the trigger point)
    • Infection
    • Perforation of a viscous body, most commonly
    the lung
    • Nerve injury from direct trauma by the needle
    • Transient nerve block
    • Syncope
    • Allergic reaction from the anesthetic
    Torticollis is a contraction, often spasmodic,
    of the muscles of the neck, chiefly those
    supplied by the spinal accessory nerve; the
    head is drawn to one side and usually rotated
    so that the chin points to the other side. While
    missing the trigger point during needling can
    cause a painful contraction of a taut band, torticollis
    has not been noted.
134
Q
  1. Two days after a trigger point injection, a patient
    presents to your office irate. He claims that the
    trigger point injection has not helped him whatsoever.
    Which one of the following is not a likely
    reason why you are having this problem?
    (A) You missed the trigger point during
    needling
    (B) The patient is not motivated to improve
    (C) You injected the secondary or satellite
    trigger point and not the primary
    trigger point
    (D) Insufficient muscle stretching in the
    clinic after the injection
    (E) Not enough stretching by the patient at
    home
A
  1. (B) Answers (A), (C), (D), and (E) are all causes
    of trigger point failure.
    C. Myofascial adhesions can possibly develop
    with secondary or “satellite” trigger points in
    nearby muscles. Trigger points appearing in
    muscles that are part of a functional unit
    must be treated together. Muscles that work
    together as agonists or in opposition as
    antagonists, constitute a functional muscle
    unit. For example the trapezius and levator
    scapula muscles work together as agonists
    in elevation of the shoulder, but are antagonists
    in rotation of the scapula, the trapezius
    rotating the glenoid fossa upward and the
    levator scapula rotating it downward.
135
Q
  1. Mechanical precipitating factors can cause
    unrelenting musculoskeletal pain. The major
    mechanical factors that practitioners must consider
    in treating myofascial pain syndrome
    include anatomic variations, poor posture, and
    work-related stress. Of the anatomic variations,
    which of the following are the most common?
    (A) Leg length discrepancy and small
    hemipelvis
    (B) Short femur syndrome
    (C) Long great toe syndrome
    (D) Kyphosis
    (E) All of the above
A
  1. (A) The most common anatomic variations that
    constitute mechanical factors precipitating
    myofascial pain are: leg length discrepancy and
    small hemipelvis, short upper arm syndrome,
    and the long second metatarsal syndrome.
    The leg length disparity syndrome produces
    a pelvic tilt that results in a cascade of
    chronic contraction and activation of a chain of
    muscles in an attempt to straighten the head
    and level the eyes. The quadratus lumborum
    and paraspinal muscles contract to correct the
    deviation of the spine caused by the pelvic tilt.
    Unwarranted loading perpetuates myofascial
    trigger points and may result in low back,
    head, neck, and shoulder pain. Trigger points
    in these persistently shortened and constantly
    contracted muscles are not easily inactivated
    until the muscles are unloaded. The quadratus
    lumborum is less likely to develop trigger
    points during the teenage years, and typically,
    unilateral low back pain is located on the side
    of the shorter leg because of early shrinking of
    the ipsilateral annulus fibrosis. In adults, it
    occurs on the side of the longer leg, caused by
    later spondylitic changes and quadratus lumborum
    shortening. A true leg length incongruity
    can be corrected by placing a heel lift on
    the shorter leg. The asymmetry caused by a
    small hemipelvis is corrected by placing an
    ischial or “butt” lift under the ischial tuberosity.
    Short upper arms cause forward shoulder
    roll, pectoral muscle shortening, and abnormal
    loading of the neck and trunk muscles as
    the individual attempts to find a comfortable
    position when seated.
    Along second metatarsal bone obscures the
    stable tripod support of the foot produced by
    the first and second metatarsal bones anteriorly
    and the heel posteriorly. In contrast, in this condition,
    weight is carried on a knife-edge from
    the second metatarsal head to the heel, overstressing
    the peroneus longus that attaches to
    the first metatarsal bone. Diagnostic callus formation
    takes place in the abnormally stressed
    areas: under the second metatarsal head, and
    on the medial aspect of the foot at the great toe
    and first metatarsal head. Correction is accomplished
    with support under the head of the first
    metatarsal.
136
Q
471. What nutritional or hormonal factors have
repeatedly been found to be low in persons
with persistent myofascial pain?
(A) Iron
(B) Folic acid
(C) Vitamin B12
(D) Thyroid hormone
(E) All of the above
A
  1. (E)
    A. In women with chronic coldness and
    myofascial pain, ferritin has been found to
    be below 65%, largely because of an iron
    intake that is insufficient to replace menstrual
    loss. GI blood loss caused by antiinflammatories
    and parasitic diseases can
    also cause ferritin to be low. Ferritin represents
    the tissue bound nonessential iron stores
    in the body that supply the essential iron
    for oxygen transport and iron-dependent
    enzymes. Fifteen to 20 ng/mL is low and
    anemia is common at levels of 10 ng/mL or
    less. The association between depleted iron
    and chronic myofascial pain hints that ironrequiring
    enzymatic reactions may be limited
    in these people, which may produce an
    energy crisis in muscle when it is overloaded
    and thereby produce metabolic stress.
    Myofascial trigger points will not easily
    resolve in such instances, and iron supplementation
    in patients with chronic myofascial
    pain syndrome and serum ferritin levels
    below 30 mg/mL prevents or corrects these
    symptoms.
    B. and C. Folic acid and vitamin B12 function
    not only in erythropoiesis but also in central
    and peripheral nerve formation. Preliminary
    studies have shown that 16% of patients
    with chronic myofascial pain syndrome
    either were deficient in vitamin B12 or had
    insufficient levels of vitamin B12, and that
    10% had low serum folate levels.
    D. Hypothyroidism can be suspected in
    chronic myofascial pain syndrome when
    coldness, dry skin/hair, constipation, and
    fatigue are also present. One study, found
    that under these circumstances (chronic) it
    occurred in 10% of patients. The myofascial
    trigger points tend to be more extensive in
    hypothyrotic patients. Hormone replacement
    may resolve many myofascial complaints
    and perpetuate a more permanent
    healthy state by allowing the implementation
    of physical therapy and trigger point
    inactivation.
    Also common in chronic musculoskeletal
    pain: low vitamin D, recurrent candida yeast
    infections, elevated uric acid levels, parasitic
    infections (especially amebiasis), Lyme disease,
    osteoarthritis, rheumatoid arthritis, Sjögren
    syndrome, carpal tunnel syndrome, and peripheral
    neuropathy secondary to DM.
    The postlaminectomy syndrome is frequently
    caused by myofascial trigger points.
137
Q
472. Biologic aberrations seen in most patients with
fibromyalgia include all of the following, EXCEPT
(A) lowered pain thresholds to pressure
induced pain
(B) disordered sleep as evidenced by
polysomnography
(C) increased spinal fluid levels of
substance P
(D) decreased spinal fluid levels of nerve
growth factor (NGF)
(E) no physiological or biochemical
evidence for central sensitization
A
  1. (D) Biologic abnormalities that are detected in
    most fibromyalgia patients include
    • Dysfunctional sleep by polysomnography
    • Physiological or biochemical evidence for
    central sensitization
    • Temporal summation or second pain
    • Lowered thresholds to pressure-induced
    pain detected by brain imaging
    • Low levels of the biogenic amines to drive
    descending inhibition of nociception
    • Elevated spinal fluid levels of substance P
    • In primary fibromyalgia only, elevated
    spinal fluid levels of NGF
138
Q
  1. Of the following criteria, which are absolutely
    necessary for the classification of fibromyalgia
    syndrome?
    (A) Widespread pain for at least 3 months
    (B) Pain sensitivity to 4 kg of digital
    pressure at a minimum of 11 of 18
    anatomically defined tender points
    (C) Diagnosis after the age of 18 years
    (D) A and B
    (E) A, B, and C
A
  1. (D) The American College of Rheumatology
    sanctioned a study that led to the criteria for
    diagnosing fibromyalgia: a history of widespread
    pain for at least 3 months and pain sensitivity
    to 4 kg of digital pressure at 11 or more
    of 18 anatomically defined tender points. The
    criteria displayed sensitivity and specificity of
    88.4% and 81.1%, respectively, for patients
    with fibromyalgia against normal control
    and disease control subjects with other painful
    conditions.
139
Q
  1. Which of the following is not one of the 18 potential
    locations for tender points in fibromyalgia?
    (A) Occiput, at the suboccipital muscle
    insertion
    (B) Low cervical, at the anterior aspects of
    the intertransverse spaces at C5-C7
    (C) Lumbar paraspinal musculature, from
    the level of L3 to L5
    (D) Lateral epicondyle, extensor muscle,
    2 cm distal to the epicondyle
    (E) Knees, at the medial fat pad proximal to
    the joint line and condyle
A
  1. (C) In addition to the four areas mentioned
    in the question stem, five other locations
    exist:
  2. Trapezius, at the middle of the upper muscle
    border
  3. Supraspinatus, near the origins, above the
    spine of the scapula
  4. Second rib, upper surface just lateral to the
    second costochondral junction
  5. Gluteal, in upper outer quadrants of buttocks
    in anterior fold of muscle
  6. Greater trochanter, posterior to the trochanteric
    prominence
140
Q
  1. A 60-year-old female recently diagnosed with
    fibromyalgia has been exhibiting signs of anxiety
    and depression and is now in treatment.
    This situation
    (A) makes perfect sense because fibromyalgia
    is a psychogenic disorder
    (B) relates to the fact that a subgroup of
    fibromyalgia patients concurrently have
    depression and anxiety, although an
    affective disorder is unlikely to cause
    fibromyalgia
    (C) demonstrates a patient who is less likely
    to exhibit signs of depression than one
    who has not sought out medical care
    (D) exhibits a patient who was probably
    abused as a child
    (E) none of the above
A
  1. (B)
    A. and B. Fibromyalgia is no longer considered
    a psychogenic disorder, however,
    there is a subgroup of fibromyalgia patients
    with associated depression or anxiety.
    C. It is believed that fibromyalgia patients
    who have had medical treatment are more
    apt to exhibit symptoms of depression than
    those in the community who have not.
    D. Sexual abuse in childhood is no longer
    considered a legitimate hypothesis for the
    origination of fibromyalgia.
141
Q
476. Which of the following does not occur often
as a clinical manifestation of the fibromyalgia
syndrome?
(A) Irritable bladder syndrome
(B) Irritable bowel syndrome
(C) Urinary urgency
(D) Dizziness and light-headedness
(E) A and D
A
  1. (E) The female urethral syndrome or irritable
    bladder syndrome constitutes urinary frequency,
    dysuria, suprapubic discomfort, and
    urethral pain despite sterile urine.
142
Q
  1. A 50-year-old female with fibromyalgia complains
    of trouble sleeping. You are not surprised
    as it is well-known that these patients
    (A) awaken in the morning feeling stiff,
    cognitively sluggish, and unrefreshed
    by their sleep
    (B) commonly awaken feeling distressingly
    alert after only a few hours of sleep
    (mid insomnia) and then are unable to
    sleep soundly again until near morning
    (terminal insomnia)
    (C) don’t have trouble napping during the
    day
    (D) A and B
    (E) A, B, and C
A
  1. (E)
    A., B., and C. Most patients (90%) with
    fibromyalgia have trouble sleeping. Some
    have difficulty getting to sleep (initial insomnia),
    while the majority awaken feeling alarmingly
    alert after only a few hours of sleep (mid
    insomnia) and are then unable to sleep
    soundly again until near morning (terminal
    insomnia). They usually awaken in the morning
    feeling incredibly stiff (lasts 45 minutes
    to 4 hours), mentally listless, and unrefreshed
    by their sleep. Hence, it is surprising that they
    have difficulty napping during the day.
    Moldofsky (2002) observed that 60% of
    fibromyalgia patients exhibit an electroencephalogram
    (EEG) pattern of sleep architecture
    called alpha wave intrusions of deep, delta
    wave, non–rapid eye movement sleep, which
    relates to subjective fatigue and psychologic
    distress but is not specific for fibromyalgia. Its
    prevalence in the healthy general population
    or in those with insomnia or dysthymia is
    only 25%. [
143
Q
  1. Which one of the following statements is false
    regarding fatigue in fibromyalgia?
    (A) It should always be attributed to the
    fibromyalgia itself
    (B) It is rarely induced by medications
    (C) It manifests as a feeling of weakness as
    opposed to the feeling of tiredness felt
    in chronic fatigue syndrome
    (D) A and B
    (E) A, B, and C
A
  1. (E) Approximately 80% of patients with
    fibromyalgia have fatigue, while a small percentage
    of these actually meet the criteria for
    chronic fatigue syndrome (CFS). CFS is thought
    to affect approximately 4 per 1000 adults. For
    unknown reasons, CFS occurs more often in
    women and in adults in their 40s and 50s. The
    illness is estimated to be less prevalent in children
    and adolescents, but study results vary as
    to the degree. CFS often manifests with widespread
    myalgia and arthralgia, cognitive difficulties,
    chronic mental and physical exhaustion,
    often severe, and other characteristic symptoms
    in a previously healthy and active person. There
    remains no assay or pathologic finding which is
    widely accepted to be diagnostic of CFS. It
    remains a diagnosis of exclusion based largely
    on patient history and symptomatic criteria,
    although a number of tests can aid diagnosis.
    The fatigue of CFS is a feeling of weakness,
    while the fatigue of fibromyalgia is a feeling of
    tiredness. Fatigue may result from sedating
    medications (ie, TCAs being used for the treatment
    of insomnia in fibromyalgia). The rest of
    the differential diagnosis is quite extensive and
    must take into account:
    • Sleep disorders
    • Chronic infections
    • Autoimmune disorders
    • Psychiatric comorbidities
    • Neoplasia
144
Q
  1. Secondary fibromyalgia refers to
    (A) fibromyalgia that does not interfere with
    a patient’s functioning
    (B) fibromyalgia that occurs in the setting of
    another painful condition or inflammatory
    disorder
    (C) a multifocal pain syndrome that occurs
    only after a patient has been diagnosed
    with dysthymia
    (D) fibromyalgia that meets all the other
    characteristics of the disease but only
    produces between 8 and 10 tender
    points
    (E) none of the above
A
  1. (B) The rest of the answers are blatantly wrong.
    Secondary fibromyalgia may not be clinically distinguishable
    from that of primary fibromyalgia.
    Examples of secondary fibromyalgia:
    • Rheumatoid arthritis patients have fibromyalgia
    30% of the time
    • Systemic lupus erythematosus (SLE) 40%
    • Sjögren syndrome 50%
    • Lyme disease 20%; the symptoms of
    fibromyalgia may develop 1 to 4 months after
    infection, often in association with Lyme
    arthritis. The signs of Lyme disease will normally
    resolve with antibiotics, but the
    fibromyalgia symptoms can persist
    • Chronic hepatitis
    • Inflammatory bowel disease
    • Tuberculosis
    • Chronic syphilis
    • Bacterial endocarditis
    • AIDS
    • Hypothyroidism
    • Hypopituitarism
    • Hemochromatosis
    Patients with rheumatic disease and concomitant
    fibromyalgia experience joint pain
    out of proportion to their synovitis. The practitioner
    should treat each of the conditions
    separately, because increasing the dosage of
    antirheumatic medications in the absence of
    active inflammation might have minimal effect
    on the pain augmented by the fibromyalgia.
145
Q
  1. Avery inquisitive 40-year-old female, recently diagnosed with fibromyalgia, states that she has been reading about her condition on the Internet. She wants to know about substance P.
    You tell her that
    (A) substance P is a pronociceptive neurochemical
    mediator of pain because it
    carries or amplifies afferent signals
    (B) substance P levels in the patients with
    fibromyalgia have been found to be significantly
    higher in the CSF, serum, and
    urine
    (C) the elevation of substance P in the CSF
    is a result of lowered CSF substance P
    esterase
    (D) the elevation in CSF substance P is
    indicative of fibromyalgia
    (E) A and B
A
  1. (A)
    A. There are several neurochemical mediators
    of pain that appear to be factors in the
    pathogenesis of fibromyalgia:
    • Substance P
    • NGF (elevated in primary fibromyalgia, but
    not secondary)
    • Dynorphin A (normal or elevated in
    fibromyalgia)
    • Glutamate
    • Nitric oxide
    • Serotonin (decreased in fibromyalgia)
    • Noradrenaline (its inactive metabolite is
    significantly lowered in fibromyalgia)
    Substance P, NGF, dynorphin A, glutamate,
    and nitric oxide are considered pronociceptive
    because they transmit or intensify afferent
    signals, leading to the brain perceiving
    increased pain. On the other hand, serotonin,
    noradrenaline, the amino terminal peptide
    fragment of substance P, and endogenous opioids
    are considered to be antinociceptive
    because they hinder the transmission of nociceptive
    signals.
    B. All studies on substance P in fibromyalgia
    patients have found significantly higher
    average concentrations (two- to threefold)
    of substance P than in the CSF of healthy
    control subjects. However, the levels in
    other bodily fluids like saliva, serum, and
    urine, have been normal in fibromyalgia.
    C. The increased substance P is not because of
    decreased CSF substance P esterase activity,
    because the rate of cleavage of labeled substance
    P was found to be normal. In primary
    fibromyalgia, it is believed that NGF
    may be responsible for the elevated CSF
    substance P through its effects on central
    sensitization and neuroplasticity.
    D. Increased CSF substance P is not specific
    to fibromyalgia as it is also seen in painful
    rheumatic diseases irrespective of whether
    they have fibromyalgia. In patients that
    were status post–total hip replacement,
    elevated substance P prior to the procedure
    normalized after the surgery when
    the pain was gone. Certain chronic conditions
    such as low back pain and diabetic
    neuropathy present with lower than normal
    CSF substance P levels.
146
Q
  1. The management objectives for fibromyalgia
    are
    (A) not specific because there is still no cure
    (B) reestablish emotional balance
    (C) improve sleep
    (D) restore physical function
    (E) all of the above
A
481. (E) There is no current cure for fibromyalgia, so
its management is
• Nonspecific
• Multimodal
• Expectant
• Symptomatic
The goals are to
• Decrease pain
• Enhance sleep
• Reinstate physical function
• Maintain social interaction
• Restore emotional balance
• Decrease the excessive use of health care
resources
The best way to achieve these goals is through
a multidisciplinary approach of
• Education
• Exercise
• Physical therapy
• Medications
• Social support
147
Q
  1. The shared decision concept
    (A) is a method where half of the treatment
    decisions come from the physician and
    half come from the ancillary staff (physical
    therapists, massage therapists, etc)
    (B) improves both patient and physician
    satisfaction
    (C) must be used on selective patients
    because cultural background, beliefs, and
    religion can all inhibit its effectiveness
    (D) A and B
    (E) A, B, and C
A
  1. (B) The shared decision concept emphasizes
    the importance of simultaneous exchange of information until an agreement between the
    doctor and patient can be achieved concerning
    available diagnostic and treatment approaches.
    It improves both patient and physician satisfaction,
    is preferred by patients, and sets the
    stage for better outcomes.
    A. The shared decision concept involves the
    physician and the patient.
    C. The physician may outline the treatment
    options with associated risks and benefits,
    while the patient may disclose information
    about their culture, fears, expectations,
    beliefs, and attitudes.
148
Q
  1. A 60-year-old female recently diagnosed with
    fibromyalgia wants to discuss her treatment
    options. She is adamant about not taking medications.
    Which one of the following statements
    is false regarding her alternatives?
    (A) Relaxation techniques like progressive
    muscle relaxation, self-hypnosis, or
    biofeedback have been recommended
    (B) Cognitive behavioral therapies and
    support groups are efficacious is some
    patients
    (C) Aerobic exercise can yield positive
    outcomes
    (D) Heat and cold applications can provide
    relief
    (E) Deep massage does more harm than good
A
  1. (E)
    A. All these techniques have been recommended
    for some patients with fibromyalgia.
    Progressive muscle relaxation was
    developed by Jacobson, who argued that
    since muscular tension accompanies anxiety,
    one can reduce anxiety by learning how
    to relax the muscular tension. Jacobson
    trained his patients to voluntarily relax certain
    muscles in their body in order to reduce
    anxiety symptoms. He also found that the
    relaxation procedure is effective against
    ulcers, insomnia, and hypertension. Selfhypnosis
    is a naturally occurring state of
    mind which can be defined as a heightened
    state of focused concentration (trance), with
    the willingness to follow instructions (suggestibility).
    Biofeedback is a form of alternative
    medicine that involves measuring a
    subject’s quantifiable bodily functions such
    as blood pressure, heart rate, skin temperature,
    sweat gland activity, and muscle tension,
    conveying the information to the
    patient in real time. This raises the patient’s
    awareness and conscious control of their
    unconscious physiological activities. By providing
    the user access to physiologic information
    about which he or she is generally
    unaware, biofeedback allows users to gain
    control of physical processes previously
    considered an automatic response of the
    autonomous nervous system.
    B. Cognitive-behavioral therapies have
    improved pain scores, pain coping, pain
    behavior, depression, and physical functioning
    over several months in fibromyalgia
    patients. It is suspected that follow-ups with
    booster sessions may prolong the effects.
    While some think that support groups perpetuate
    griping, a resource-oriented selfsupport
    group can help a fibromyalgia
    patient come to terms with an illness and
    provide invaluable patient education.
    C. Aerobic exercise is one of the first nonpharmacologic
    strategies promoted for patients
    with fibromyalgia. Low-impact aerobics of
    sufficient intensity to produce cardiovascular
    stimulation can decrease pain, enhance
    sleep, improve mood, increase energy,
    advance cognition, and better a patient’s
    overall outlook. Fibromyalgia patients who
    exercise deal better with the disease.
    However, a fibromyalgia patient can also
    experience increased pain if the exercise regimen
    is too strenuous or carried out during
    an inopportune time in the treatment. These
    patients should begin with low-impact exercises
    (ie, aqua therapy). Continuing the
    patient on the exercise regimen becomes easier
    as the patient’s pain decreases.
    D. Heat helps fibromyalgia patients with tenderness,
    stiffness, and cephalgia. It can also
    calm muscles, ease exercising, and accentuate
    a sense of well-being. Cold application
    also works.
    E. Some patients do obtain relief by light
    massages that progress to more deep sedative
    ones.
149
Q
  1. A PhD student comes with questions. She
    wants to know how dopamine and serotonin
    play a role in fibromyalgia pathogenesis. You
    tell her that
    (A) dopamine levels directly correlate with
    pain levels
    (B) tryptophan, serotonin, 5-hydroxytryptophan,
    and 5-hydroxyindole acetic acid
    have been found to be decreased in
    fibromyalgia patients
    (C) the number of tender points in
    fibromyalgia patients have not been
    found to correlate with the concentration
    of serotonin in the serum
    (D) dopamine agonists have been found to
    decrease pain in fibromyalgia patients
    (E) B and D
A
  1. (E)
    A. and D. Dopamine is a neurotransmitter best
    known for its role in the pathology of schizophrenia,
    Parkinson disease, and addiction.
    There is also strong evidence for a role of
    dopamine in restless leg syndrome, which is
    a common comorbid condition in patients
    with fibromyalgia. In addition, dopamine
    plays a critical role in pain perception and
    natural analgesia. Accordingly, musculoskeletal
    pain complaints are common among
    patients with Parkinson disease, which is
    characterized by drastic reductions in
    dopamine owing to neurodegeneration of dopamine-producing neurons, while patients
    with schizophrenia, which is thought to
    arise, at least partly, from hyperactivity of
    dopamine-producing neurons, have been
    shown to be relatively insensitive to pain.
    Interestingly, patients with restless legs syndrome
    have also been demonstrated to have
    hyperalgesia to static mechanical stimulation.
    Fibromyalgia has been commonly
    referred to as a “stress-related disorder”
    owing to its frequent onset and worsening of
    symptoms in the context of stressful events.
    It was therefore proposed that fibromyalgia
    may represent a condition characterized by
    low levels of central dopamine that likely
    results from a combination of genetic factors
    and exposure to environmental stressors,
    including psychosocial distress, physical
    trauma, systemic viral infections, or inflammatory
    disorders (eg, rheumatoid arthritis,
    systemic lupus erythematosus). This conclusion
    was based on three key observations:
    (1) fibromyalgia is associated with stress;
    (2) chronic exposure to stress results in a disruption
    of dopamine-related neurotransmission;
    and (3) dopamine plays a critical role in
    modulating pain perception and central
    analgesia in such areas as the basal ganglia
    including the nucleus accumbens, insular
    cortex, anterior cingulate cortex, thalamus,
    periaqueductal gray, and spinal cord. In
    support of the “dopamine hypothesis of
    fibromyalgia,” a reduction in dopamine synthesis
    has been reported after using positron
    emission tomography (PET) and demonstrated
    a reduction in dopamine synthesis
    among fibromyalgia patients in several
    brain regions in which dopamine plays a
    role in inhibiting pain perception, including
    the mesencephalon, thalamus, insular cortex,
    and anterior cingulate cortex. A subsequent
    PET study demonstrated that, whereas
    healthy individuals release dopamine into
    the caudate nucleus and putamen during a
    tonic experimental pain stimulus (ie, hypertonic
    saline infusion into a muscle bed),
    fibromyalgia patients fail to release dopamine
    in response to pain and, in some cases, actually
    have a reduction in dopamine levels
    during painful stimulation. Moreover, a
    substantial subset of fibromyalgia patients
    respond well in controlled trials to pramipexole,
    a dopamine agonist that selectively
    stimulates dopamine D2/D3 receptors and
    is used to treat both Parkinson disease and
    restless legs syndrome.
    B. Tryptophan is decreased in the serum and
    CSF of fibromyalgia patients. Serotonin is low
    in fibromyalgia serum. 5-Hydroxytrytophan,
    the intermediary between tryptophan and
    serotonin, and 5-hydroxyindole acetic
    acid, the by-product of serotonin metabolism,
    are both low in the CSF of patients
    with fibromyalgia. The excretion in urine of
    5-hydroxyindole acetic acid was lower than
    normal in patients with fibromyalgia, lower
    in females versus males, and lower in females
    with fibromyalgia versus females who don’t
    have fibromyalgia.
    C. The numbers of active tender points in
    fibromyalgia patients directly correlated
    with the concentration of serotonin in
    fibromyalgia sera.
150
Q
  1. No treatments seem to be working for a 45-
    year-old female with a 5-year history of
    fibromyalgia. She has talked to a relative who
    told her that oxycodone/acetaminophen works
    for all pain. How do you respond?
    (A) Opioids work but only when a shortacting
    medication is combined with a
    long-acting medication
    (B) In combination with pregabalin and
    duloxetine, hydromorphone has displayed
    incredible synergy in extremely
    depressed fibromyalgia patients
    (C) Women with fibromyalgia have reduced
    μ-opioid receptor availability within
    regions of the brain that normally
    process and dampen pain signals
    (D) Fibromyalgia is so difficult to treat that
    you are willing to try anything that she
    thinks may help
    (E) You are not opposed to trying opioids,
    but the potential hyperalgesia has been
    found to be significantly worse in
    patients with fibromyalgia
A
  1. (C) Fibromyalgia, a common chronic pain condition
    characterized by widespread pain, is
    thought to originate largely from altered central
    neurotransmission. In this study, a sample of 17
    fibromyalgia patients and 17 age- and sexmatched
    healthy controls, were compared
    using μ-opioid receptor PET. PET scans measure
    blood flow in the brain. It was demonstrated
    that fibromyalgia patients display
    reduced μ-opioid receptor binding potential
    within several regions known to play a role in
    pain modulation, including the nucleus accumbens,
    the amygdala, and the dorsal cingulate.
    The reduced availability of the receptors
    could result from a reduced number of opioid
    receptors, enhanced release of opioids that are
    produced naturally by the body, or both.
    These findings indicate altered endogenous
    opioid analgesic activity in fibromyalgia and
    suggest a possible reason for why exogenous
    opiates appear to have reduced efficacy in this
    population. The reduced availability of the
    receptor was associated with greater pain among
    people with fibromyalgia.
    Answers (A), (B), (D), and (E) have no merit.
151
Q
  1. Numerous medications have been used to treat
    the insomnia associated with fibromyalgia.
    Which one of the following has not been used?
    (A) Amitriptyline
    (B) Cyclobenzaprine
    (C) Fluoxetine
    (D) Clonazepam
    (E) Pregabalin
A
  1. (C)
    A. and B. The sedating tricyclic biogenic amine
    reuptake drugs, such as amitriptyline and
    cyclobenzaprine, are the most commonly
    prescribed medications for fibromyalgia
    insomnia. These medications are mostly
    used in low doses to improve sleep and to
    enhance the effects of analgesics (amitriptyline
    10-25 mg at night and cyclobenzaprine
    5-10 mg at night). Patients can develop
    tachyphylaxis to them, but a 1-month
    holiday from the drugs may help restore
    effectiveness.
    C. SSRIs are so stimulating that they can
    interfere with sleep, and should never be
    taken at bedtime.
    D. Benzodiazepine decrease anxiety and
    allow less troubled sleep (alprazolam,
    clonazepam). Clonazepam in particular
    can help control nocturnal myoclonus
    when it is associated with fibromyalgia.
    E. Pregabalin is a sedative in addition to an
    antinociceptive medication.
152
Q
487. Pathophysiologic components of cancer pain
can be
(1) somatic (nociceptive) pain
(2) sympathetic pain
(3) neuropathic pain
(4) central pain
A
  1. (E)
153
Q
488. The skeletal sites most commonly involved in
osteolytic metastatic processes are
(1) ribs
(2) humerus
(3) femur
(4) tibia
A
  1. (B) In osteolytic bone metastases, the most
    commonly involved sites are vertebrae, pelvis,
    ribs, femur, and skull. Upper and lower extremity
    bones, except femur, are not commonly
    involved.
154
Q
  1. The primary compression of the spinal cord
    from metastatic deposits occurs in
    (1) the thoracic spine in 70% of patients
    (2) the lumbar spine in 20% of patients
    (3) the cervical spine in 10% of patients
    (4) multiple sites of the spine in 60% of
    patients
A
  1. (A) Multiple sites of metastatic epidural spinal
    cord compression occur in 17% to 30% of all
    patients. This is particularly common in prostatic
    and breast carcinoma and uncommon in
    lung cancer.
155
Q
  1. In a patient with skeletal metastases, bisphosphonates
    (1) inhibit recruitment and function of
    osteoclasts
    (2) stimulate osteoblasts
    (3) have greatest effect in breast cancer and
    multiple myeloma
    (4) have an acute pain-relieving effect
A
  1. (E) Bisphosphonates decrease resorption of
    bone directly, by inhibiting the recruitment and
    function of osteoclasts, and indirectly, by stimulating
    osteoblasts. In patients with bony
    metastases, they are the standard therapy for
    hypercalcemia after rehydration, and have the
    greatest effect in patients with breast cancer
    and multiple myeloma. Bisphosphonates also
    have an acute pain-relieving effect, which is
    thought to be derived from the reduction of
    various pain-producing substances.
156
Q
  1. The following substance(s) may be useful in
    treating a patient with a malignant disease:
    (1) Gabapentin
    (2) Amitriptyline
    (3) Samarium 153
    (4) Hydromorphone
A
  1. (E) Both gabapentin, an antiepileptic drug, and
    amitriptyline, TCA, are widely used in treating
    neuropathic pain, which is often a significant
    component of a cancer pain syndrome.
    Samarium 153 belongs to the group of
    bone-seeking radiopharmaceuticals emitting
    medium- to high-energy beta particle radiation.
    The most commonly used agent in this group is
    Strontium 89 with a documented pain-relieving
    effect in patients with bony metastases.
    Samarium 153, rhenium 186, and phosphorus
    32 are also available for clinical use.
157
Q
  1. The following is (are) the possible compilation(
    s) of a neurolytic celiac plexus block:
    (1) Persistent diarrhea
    (2) Aortic pseudoaneurysm
    (3) Intradiscal injection
    (4) Damage to the artery of Adamkiewicz
A
  1. E)
158
Q
  1. In a cancer pain patient, the following agent(s)
    can be used effectively via implantable intrathecal
    delivery system:
    (1) Opioids
    (2) α2-Adrenergic agonists
    (3) Local anesthetics
    (4) Ziconotide
A
  1. (E) Both gabapentin (an antiepileptic drug) and
    amitriptyline (a TCA) are widely used in treating
    neuropathic pain, which is often a significant
    component of a cancer pain syndrome.
    Samarium 153 belongs to the group of
    bone-seeking radiopharmaceuticals emitting
    medium- to high-energy beta particle radiation.
    The most commonly used agent in this group is
    Strontium 89 with a documented pain-relieving
    effect in patients with bony metastases.
    Samarium 153, rhenium 186, and phosphorus
    32 are also available for clinical use.
159
Q
  1. Which of the following conditions are the possible
    complications of chemotherapy in a cancer
    patient?
    (1) Toxic peripheral neuropathy
    (2) PHN
    (3) Avascular necrosis
    (4) Pseudorheumatism
A
  1. (E)
160
Q
  1. As compared to younger subjects which of the
    following is correct about older people with
    pain?
    (1) There may be difficulties in determining
    the etiology of pain in older people
    (2) Older people generally receive significantly
    lower amounts of opioid analgesia
    (3) There may be increased potency of
    opioids
    (4) The majority of older people choose
    quantity of life over quality of life
A
  1. (E)
161
Q
496. The goals of palliative care can be summarized
as follows:
(1) To help those who need not die to live,
and to live with the maximum of
freedom from constraints on their
quality of life arising from acute and
chronic conditions of the body
(2) To help those who can no longer live to
die on time—not too early and not late
(3) To help the dying, whether in hospital,
nursing home, hospice, or at home, to
die with dignity and in peace
(4) To administer euthanasia only to the
patients who truly understand the fact
that their condition is terminal and who
personally request it
A
  1. (A)
162
Q
497. Peripheral neuropathy(ies) is (are) characterized
by
(1) sensory loss
(2) fasciculations
(3) dysesthesias
(4) chronic pain
A
  1. (B) In clinical practice, most peripheral neuropathies
    do not produce chronic pain as
    impairment of nerve fibers carrying nociception
    should result in decrease pain perception.
    In most neuropathies, all components of the
    peripheral nervous system are affected, presenting
    with variable sensorimotor deficit and
    autonomic dysfunction.
163
Q
  1. Area(s) of acute pain processing in cortical and
    subcortical regions of the brain as determined
    by functional MRI include
    (1) anterior cingulate cortex
    (2) parietal cortex
    (3) prefrontal cortex
    (4) hypothalamus
A
  1. (B) The most commonly activated areas during
    acute processing of pain in humans are S-I, S-II,
    anterior cingulated cortex, insular cortex, prefrontal
    cortex, thalamus, and cerebellum.
164
Q
  1. Small-diameter peripheral neuropathies are
    commonly painful. Example(s) of these neuropathies
    include
    (1) Ross syndrome (segmental anhidrosis)
    (2) Fabry disease
    (3) Charcot-Marie-Tooth disease type 1
    (4) diabetic neuropathy
A
  1. (C) Many studies suggest that axonal injury
    along the nociceptive fiber in the peripheral nervous system is the main cause of neuropathic
    pain. Several conditions where the small
    fibers are spared support this concept. The
    Charcot-Marie-Tooth disease also known as
    hereditary motor and sensory neuropathy
    where the demyelination is limited to large
    myelinated fibers, do not manifest with pain.
    Segmental anhidrosis or Ross syndrome where
    only autonomic fibers are affected, is also not
    painful. On the other hand, conditions affecting
    the small nerve fibers, like diabetic neuropathy
    or Fabry disease, a rare lipid-storage disorder,
    commonly present with pain.
165
Q
500. Chronic renal failure neuropathy is commonly
manifested with
(1) restless leg syndrome
(2) painful neuropathy
(3) distal weakness
(4) selective loss of small nerve fibers
A
  1. (A) Chronic renal failure is associated with
    selective loss of large nerve fibers which is
    rarely painful. Common symptoms include
    restless leg syndrome, distal numbness, and
    paresthesias, with distal weakness usually in
    the lower extremities.
166
Q
501. Animal studies in neuropathic pain conditions
have shown
(1) intraplantar injections of interleukin
1(IL-1) reduces mechanical nociceptive
threshold
(2) IL-1 hyperalgesia is mediated by
bradykinin B-1 receptors
(3) effects of IL-1 on mechanical hyperalgesia
seems to be mediated by
prostaglandins
(4) IL-1 effects on nociceptions may be
mediated by vagal afferents
A
  1. (E) Cytokines, a heterogeneous group of peptides
    activate the immune system and mediate
    inflammation. They form a complex bidirectional
    system that communicates between the
    immune system and the CNS. IL-1 is the most
    extensively studied cytokine. Intraplantar as
    well as intraperitoneal injections of IL-1 reduce
    mechanical and probably thermal nociceptive
    threshold, which may be blocked by local
    cyclooxygenase inhibitors, supporting the role
    of prostaglandins in the process. The communicating
    pathway between the peripheral
    cytokines and the brain may involve vagal
    afferents terminating in the nucleus tractus solitarius
    and circumventricular sites that lack a
    blood-brain barrier.
167
Q
  1. Potential complication(s) of stellate ganglion
    block include
    (1) pneumothorax
    (2) lesion of the recurrent laryngeal nerve
    (3) neuritis
    (4) Horner syndrome
A
  1. (A) Blockade of the sympathetic innervation
    of the head can be documented by the presence
    of Horner syndrome, which is characterized
    by myosis, ptosis, and enophthalmus.
    Associate findings include conjunctival injection,
    nasal congestion, and facial anhidrosis.
    Horner syndrome is an expected finding after
    blockade of the sympathetic afferents to the
    face and can not be considered a complication.
168
Q
  1. Important factor(s) involved in the development
    of neuropathic pain include
    (1) behavioral studies have shown that
    NMDA is involved in the induction and
    maintenance of pain-related behaviors
    (2) the spinal N-type voltage-dependent
    calcium channels are the predominant
    isoform involved in the pre- and postsynaptic
    processing of sensory nociceptive
    information
    (3) tactile allodynia in the spinal nerve ligation
    model may be blocked by intrathecal
    N-type Ca2+ blockers like ziconotide
    (4) after nerve injury there is upregulation
    of the NMDA receptors
A
  1. (E) Behavioral studies have shown that activation
    of NMDA receptors are required for the
    development and maintenance of pain-related
    behaviors. Calcium channels are the key ion
    involved in the release of transmitters. Different
    subtypes of calcium channels (L-, N-, and P/Qtypes)
    may have a differential role depending
    on the nature of the pain state. The N-type voltagedependent
    calcium channels appear to be the
    predominant isoform involved in the pre- and
    postsynaptic processing of sensory nociceptive
    inputs. Animal and clinical studies have shown
    partial pain relief with the use of a specific
    N-type calcium channel blocker synthetically
    derived from a conotoxin, SNX-111. The generic
    name of this substance derived from the snail’s
    natural conotoxin is Ziconotide.
169
Q
  1. Which of the following is (are) effect(s) of
    μ-opioid agonists in neuropathic pain conditions?
    (1) Decrease dynamic allodynia
    (2) Decrease temperature threshold for cold
    pain
    (3) Decrease static allodynia
    (4) μ-Opioid agonists do not have any
    beneficial effects in patients with
    neuropathic pain conditions
A
  1. (A) The effectiveness of opioid agonists in the
    management of neuropathic pain has created
    significant controversy over the last two
    decades. Recent studies have increased our
    understanding about this topic. In patients
    with SCI and stroke, IV morphine showed poor
    effects in reducing spontaneous pain, but significantly
    reduced stroking allodynia. Other
    studies used alfentanil in the treatment of neuropathic
    pain independently of the etiology
    and observed decrease in dynamic, stroking
    allodynia, and spontaneous pain, while increase
    the temperature at which heat pain was detected
    and decrease the temperature at which cold
    pain was detected.
170
Q
  1. Effect(s) of GABA in the modulation of afferent
    nociceptive input include
    (1) GABAA produces postsynaptic inhibition
    via metabotropic receptors, which
    are ligand-gated Cl− channels
    (2) the dominant type of inhibition of glutaminergic
    excitatory postsynaptic action
    potential is produce by GABA and/or
    glycine
    (3) GABAB and adenosine produce postsynaptic
    hyperpolarization by activation
    of K+ channels
    (4) GABA and glycine produce slow
    activation of postsynaptic potentials
A
  1. (A) Action potentials in the dorsal horn neurons
    are mediated by glutaminergic excitatory
    postsynaptic potentials, this activity may be
    inhibited predominantly by the inhibition produced
    by GABA and/or glycine which causes
    fast inhibition of postsynaptic potentials.
    GABAAand glycine receptors are ligand-gated
    Cl− channels, while GABAB, adenosine, and
    opioids exert their typically produced postsynaptic
    hyperpolarization by activation of K+
    channels.
171
Q
  1. During the windup process
    (1) sustained depolarization may recruit K+
    channels, leading to decrease in the
    intracellular Ca2+ levels
    (2) cumulative recruitment of NMDAreceptor
    current leads to progressive
    relief of the Mg2+ blockade of the
    NMDA-receptor pore
    (3) more intense or sustained noxious
    peripheral stimulation induces a
    decrease in the release of neuromodulator
    peptides, leading to an excitatory
    state
    (4) intracellular calcium levels play a major
    role in the development of windup
A
  1. (C) More intense or sustained noxious peripheral
    stimulation induce primary afferent nociceptors
    to discharge at higher frequencies and
    to release from central nociceptor terminals
    neuromodulator peptides like CGRP, substance
    P, and glutamate. As more and more dorsal horn neurons get depolarized, NMDA receptors
    open by removing the Mg2+ blockade,
    allowing for intracellular calcium levels to
    increase. The end result of these intracellular
    signaling cascades is windup.
172
Q
  1. In patients with PHN
    (1) histopathologic studies in patients with
    PHN commonly show ganglion cell loss
    and fibrosis
    (2) sensory loss function in the affected
    dermatome with increase heat pain
    perception is an almost universal
    finding
    (3) antiviral drugs used in chronic PHN
    usually are ineffective in alleviating
    pain
    (4) cold stimuli–evoked pain is more common
    than heat-evoked pain
A
  1. (B) Histopathological studies in patients with
    PHN have found ganglion cell loss and fibrosis
    and atrophy of the dorsal horn, DRG, dorsal
    root, and peripheral nerves. Up to 30% of
    patients with PHN have no loss of sensation in
    the affected dermatome demonstrating minimal
    or no loss of neuronal function and interestingly
    thermal sensory thresholds are not
    affected or even decreased. Antiviral drugs
    have shown disappointing results in patients
    with chronic PHN. Heat hyperalgesia is more
    common than cold hyperalgesia, which only
    occurs in less than 10% of the patients.
173
Q
  1. The Special Olympics has brought together
    thousands of people with disabilities. Often
    enough, the race times in these events are significantly
    better than those of participants in
    the traditional games. In addition to undergoing
    the same type of grueling training regimens,
    however, participants in the Special
    Olympics often have to deal with difficulties
    performing activities of daily living as well as
    comorbidities associated with their primary
    disease. The likelihood that an amputee in a
    wheelchair race has phantom limb pain would
    be decreased if the
    (1) participant is a young child
    (2) participant is a male
    (3) participant is a congenital amputee
    (4) amputation is a below the knee amputation
    versus an above the knee amputation
A
  1. (B) Phantom pain is equally frequent in men
    and women and is not influenced by age in
    adults, side or level of amputation, and cause
    (civilian versus traumatic) of amputation.
    Phantom pain is less frequent in young children
    and congenital amputees.
174
Q
  1. A triple amputee (bilateral lower extremities,
    left upper extremity) presents to the pain clinic
    for work-up and treatment of phantom limb
    pain. This patient’s pain most likely
    (1) occurs intermittently
    (2) is primarily localized to the fingers or
    palm of the hand in the upper extremity
    or toes, feet, or ankles in the bilateral
    lower extremities
    (3) is of stabbing, shooting, or pins and
    needles character
    (4) presents with attacks that last several
    minutes to an hour
A
  1. (A) Phantom pain is usually intermittent. Only
    a few patients are in constant pain. Episodes of
    pain are most often reported to occur daily, or
    at daily or weekly intervals.
    Phantom pain is usually localized to the
    distal parts of the missing limb. Pain is normally
    felt in the fingers and palm of the hand
    in upper limb amputees and toes, foot, or
    ankle in lower limb amputees. This may be
    because of the larger cortical representation of
    the hand and foot as opposed to the lesser representation
    of the more proximal parts of the
    limb.
    The character of phantom pain is usually
    described as shooting, pricking, burning, stabbing,
    pins and needles, tingling, throbbing,
    cramping, and crushing
175
Q
  1. True statement(s) about phantom sensations is
    (are):
    (1) They are less frequent than phantom
    pains
    (2) They usually appear 1 month after the
    amputation
    (3) The phantom sensation usually manifests
    as enlargement of the missing limb
    (4) A common position of the phantom for
    upper limb amputees is the fingers
    clenched in a fist
A
  1. (D) Phantom sensations are more common than
    phantom pain, and are experienced by nearly all
    amputees. The incidence of phantom sensations
    ranges from 71% to 90%, 8 days to 2 years after
    amputation. Duration and frequency, but not
    incidence, decrease as time passes. Phantom sensations
    are less common in congenital amputees
    and in patients who underwent amputation
    before the age of 6 years
    Nonpainful sensations normally appear
    within the first days after amputation. The
    amputee often wakes up from anesthesia with
    a feeling that the amputated limb is still there.
    Just after the amputation, the phantom limb
    often resembles the preamputation limb in
    shape, length, and volume. As time passes, the
    phantom fades, leaving back the distal parts of
    the limb. For example, upper limb amputees
    may feel hand and fingers, and lower limb
    amputees may feel foot and toes.
    Commonly, upper limb amputees feel the
    fingers clenched in a fist, while the phantom
    limb of lower limb amputees is often described
    as toes flexed In some cases, phantom sensations
    include feeling of movement and posture;
    however, in others only suggestions of the
    phantom are felt.
    Telescoping (shrinkage of the phantom) is
    reported to occur in about one-third of patients.
    The phantom progressively approaches the
    stump and eventually becomes attached to it. It
    has even been experienced within the residual
    limb. Phantom pain does not retard shrinkage
    of the phantom.
176
Q
  1. Which of the following is (are) example (s) of
    how the peripheral nervous system may play a
    role in phantom pain modulation?
    (1) Dorsal root ganglion (DRG) cells display
    an altered expression pattern of
    different sodium channels
    (2) Generation, but not maintenance of
    phantom pain by the sympathetic
    nervous system
    (3) Long after limb amputation, injection of
    noradrenaline around a stump neuroma
    is reported to be intensely painful
    (4) Phantom pain is directly related to the
    skin temperature of the stump
A
  1. (B) Changes occur in the DRG cells, after a complete
    nerve cut. Cells in the DRG show similar
    abnormal spontaneous activity and increased
    sensitivity to mechanical and neurochemical
    stimulation. Local anesthesia of neuromas abolished
    tap-induced afferent discharges and tapinduced
    accentuation of phantom pain, but
    spontaneous pain and recorded spontaneous
    activity were unchanged which is consistent
    with the activity in DRG cells. DRG cells exhibit
    major changes in the expression of sodium channels,
    with an altered expression pattern of different
    channels.
    The sympathetic nervous system may
    play a role in generating and especially in
    maintaining phantom pain. It was noted that
    application of noradrenaline or activation of
    the postganglionic sympathetic fibers excites
    and sensitizes damaged, but not normal nerve
    fibers. Injecting noradrenaline into the skin can reestablish neuropathic pain that has just
    been relieved with a sympathetic block and
    injecting into a neuroma is reported to be
    intensely painful.
    The catecholamine sensitivity may also
    manifest itself in the skin with a colder limb on
    the amputated side, and it has been implied
    that phantom pain intensity is inversely related
    to the skin temperature of the stump.
177
Q
  1. Phantom pains are often a replica of preamputation
    pain. It has also been noted that
    amputees with phantom pain have more often
    suffered from intense and long-lasting preamputation
    pain than have patients without phantom
    pain. These observations led to the premise
    that preemptive analgesia may help decrease
    postamputation pain. Of the studies done on
    this subject matter
    (1) most have been of very poor methodological
    quality
    (2) the two which included blinding and
    randomization showed no significant
    differences versus controls
    (3) the aim has been to thwart spinal sensitization
    by blocking the cascade of intraneuronal
    responses that take place after
    peripheral nerve injury
    (4) the sample size was always greater
    than 100
A
  1. (A)
  2. Numerous studies on preemptive analgesia
    using epidural, epidural/perineural,
    and just perineural administration have
    been conducted. Only two were noted to
    utilize proper patient randomization and
    blinding.
  3. Persistent pain has been reported in up to
    80% of patients after limb amputation. The
    mechanisms are not fully understood, but
    nerve injury during amputation is important,
    with evidence for the crucial involvement
    of the spinal NMDA receptor in central
    changes. The study objective was to assess
    the effect of preemptively modulating sensory
    input with epidural ketamine (an
    NMDA antagonist) on postamputation
    pain and sensory processing.
  4. The aim of preemptive analgesia is to avoid
    spinal sensitization by blocking, in advance,
    the cascade of intra-neuronal responses that
    take place after peripheral nerve injury.
    True preemptive treatment is not likely possible
    in patients scheduled for amputation
    as most have been suffering from ischemic
    pain and are almost certainly presenting
    with preexisting neuronal hyperexcitability.
    To conclude: epidural blockade has been
    shown to be effective in the treatment of
    preoperative ischemic pain and postoperative
    stump pain.
178
Q
  1. Which of the following dietary modification(s)
    should be made to alleviate symptoms of interstitial
    cystitis?
    (1) Restrict spicy foods
    (2) Eliminate alcohol intake
    (3) Cease smoking
    (4) Increase orange juice intake
A
  1. (A) Acidic foods, such as orange juice, carbonated
    drinks, tomatoes, and vinegar may aggravate
    the symptoms of interstitial cystitis. Spicy
    food, alcohol, coffee, chocolate, tea, cola, and
    smoking should also be either restricted or
    completely eliminated.
179
Q
  1. Sodium pentosan polysulfate (Elmiron)
    (1) is an antispasmodic medication
    (2) is an oral analogue of heparin
    (3) alleviates symptoms of interstitial
    cystitis by relaxing smooth musculature
    (4) increases antiadherent surface of the
    bladder lining
A
  1. (C) Sodium pentosan polysulfate (Elmiron) is
    an oral analogue of heparin. Inside the bladder,
    it acts as a synthetic glycosaminoglycan layer
    and fortifies bladder wall defenses from bacteria
    by increasing the antiadherent surface of
    the bladder mucosa.
180
Q
  1. Which of the following substance(s) is (are)
    thought to be involved in descending inhibition?
    (1) GABA
    (2) Serotonin
    (3) Endogenous opioid peptides
    (4) Norepinephrine
A
  1. (E) Endogenous opioid peptides as other neurotransmitters,
    such as serotonin, norepinephrine,
    and GABA, are thought to be involved in
    descending inhibition.
181
Q
516. Which of the following is (are) the psychological
factor(s) affecting pain response?
(1) Fear and helplessness
(2) Sleep deprivation
(3) Anxiety
(4) Cultural differences
A
  1. (E) Anxiety, fear, helplessness, and sleep deprivation
    are part of the vicious cycle of pain.
    Cultural background has been shown to play a
    significant role in the individual’s pain response.
182
Q
  1. Which of the following is (are) psychological
    method(s) for reducing pain?
    (1) Placebo and expectation
    (2) Psychological support
    (3) Procedural and instructional information
    (4) Cognitive coping strategies
A
  1. (E)
183
Q
518. Gastrointestinal (GI) impairment in a postsurgical
patient can be
(1) worsened by increased sympathetic
activity because of severe pain
(2) contributed to by administration of
opioids
(3) mitigated by early mobilization of the
patient
(4) worsened by epidural blockade with
local anesthetic
A
  1. (A) There is evidence that pain-related impairment
    of intestinal motility may be relieved by
    epidural local anesthetics.
184
Q
  1. Which of the following feature(s) suggest neuropathic
    pain?
    (1) Pain in the area of sensory loss
    (2) Good response to opioids
    (3) Pain in response to nonpainful stimuli
    (4) Absence of Tinel sign
A
  1. (B) Pain in the area of sensory loss, also called
    deafferentation pain, or anesthesia dolorosa, is
    a prominent sign of a neuropathic pain.
    Neuropathic pain generally responds less well
    to opioid treatment than somatic pain. One of
    the significant signs of neuropathic pain is also
    allodynia-a painful response to nonpainful
    stimuli. Tapping of neuromas produces radiating
    electric shock sensation in the distribution
    of the damaged nerve is called Tinel sign—
    another feature of a neuropathic pain.
185
Q
  1. The correct corresponding vertebral levels for
    optimal epidural catheter placement for various
    surgical procedures are
    (1) T10-12 for lower abdominal surgery
    (2) T8-10 for upper abdominal surgery
    (3) L2-4 for lower extremity surgery
    (4) C7-T2 for upper extremity surgery
A
  1. (E)
186
Q
  1. Which of the following medications are useful
    in an inpatient management of a post–burn
    injury pain?
    (1) Opioids
    (2) Ketamine
    (3) Benzodiazepines
    (4) Nitrous oxide
A
  1. (E) Post–burn injury pain has two components:
    a constant background pain and an intermittent
    procedure-related pain. Continuous infusion of
    opioids is useful for control of the background
    component of pain. Pain related to wound care,
    dressing changes, and others (procedure-related
    pain) may require brief but profound analgesia.
    This may be achieved by administration of
    supplemental IV opioids, or addition of the
    adjuvant drugs, such as IV ketamine, IV benzodiazepines,
    inhaled nitrous oxide-oxygen mixture,
    or even general anesthesia.
187
Q
  1. In patients with a traumatic chest injuries thoracic
    epidural analgesia has been shown to significantly
    improve inspiratory effort, negative
    inspiratory force, gas exchange, ability to cough,
    and ability to clear bronchial secretions. The
    following finding may be considered relative
    contraindications for epidural analgesia in a
    patient with posttraumatic chest injury:
    (1) Inadequate coagulation function
    (2) Spine fractures
    (3) Inadequate intravascular volume
    resuscitation
    (4) Concomitant head injury
A
  1. (E) All answers are correct. Aconcomitant head
    injury may be associated with increased intracranial
    pressure, which could be considered a contraindication
    for an epidural catheter placement.6yt
188
Q
  1. The most common characteristics of pain in
    PHN include
    (1) steady burning or aching
    (2) dull and poorly localized
    (3) paroxysmal and lancinating
    (4) usually not aggravated by contact with
    the affected skin
A
  1. (B) Two types of pain may be found in PHN:
    a steady burning or aching, the other, a paroxysmal,
    lancinating pain. Both may occur spontaneously
    and are usually aggravated by any
    contact with the involved skin.
189
Q
  1. Which of the following group(s) of medications
    was found to be useful in treatment of PHN?
    (1) Opioids
    (2) Antiepileptic drugs
    (3) Topical agents
    (4) Antidepressants
A
  1. (E) All of the listed groups of medication were
    found to be effective to some extent in the treatment
    of PHN. Amultimodal approach seems to
    be more effective because of the synergistic
    effect different modes of action
190
Q
  1. Which of the following is (are) true about interventional
    therapy for PHN?
    (1) No proven surgical cure for PHN has
    been found
    (2) Cryotherapy is likely to bring only
    short-term relief
    (3) Topical lidocaine may provide effective
    analgesia for PHN
    (4) Transcutaneous nerve stimulation
    (TENS) has been shown to give an effective
    symptomatic relief in some patients
A
  1. (E)
191
Q
526. Antiviral agents in the acute phase of herpes
zoster
(1) competitively inhibit DNA polymerase,
terminating DNA synthesis and viral
replication
(2) are generally well tolerated
(3) hasten healing of the rash
(4) may reduce the duration of PHN
A
  1. (E)
192
Q
  1. Oral steroids for acute herpes zoster
    (1) are not currently recommended
    (2) may provide pain relief in the acute
    phase
    (3) have no benefit in prevention of PHN
    (4) have almost no side effects in patients
    with herpes zoster
A
  1. (A) Oral steroids may provide pain relief in
    acute phase of herpes zoster, as well as shorten
    the time to full crusting of lesions. However,
    controlled trials showed no benefit in the prevention
    of PHN. With the development of
    antiviral agents, the use of oral steroids is currently
    not recommended because of more frequent
    side effects
193
Q
  1. Which of the following statement(s) is (are)
    true about diabetic amyotrophy?
    (1) It is commonly associated with pain
    (2) It responds well to a complicated multimodal
    treatment
    (3) Involves weakness and atrophy of the
    involved muscles
    (4) Sciatic nerve and its supplied muscles
    are most commonly affected
A
  1. (B) Diabetic amyotrophy is a condition occurring
    in diabetic patients, more commonly with
    type-2 diabetes, which begins with pain in the
    thighs, hips, and buttocks. Weakness and atrophy
    of the proximal pelvic muscles groups,
    iliopsoas, obturator, and adductor muscles follows
    the painful manifestations. It usually does
    not involve sciatic nerve, or distal muscles of
    the lower extremity. The therapy for diabetic
    amyotrophy is primarily supportive.
194
Q
  1. Charcot joint
    (1) affects primarily weight-bearing joints
    (2) can be caused by multiple causes other
    than DM
    (3) is related to the destruction of afferent
    proprioceptive fibers
    (4) is related to the destruction of efferent
    neural fibers
A
  1. (A) Neuropathic arthropathy (Charcot joint) develops
    most often in weight-bearing joints. The predominant
    cause is DM, but also associated with
    neuropathic arthropathy are leprosy, yaws, congenital
    insensitivity to pain, spina bifida,
    myelomeningocele, syringomyelia, acrodystrophic
    neuropathy, amyloid neuropathy, peripheral
    neuropathy secondary to alcoholism and
    avitaminosis, SCI, peripheral nerve injury, postrenal transplant arthropathy, intraarticular
    steroid injections, and syphilis. The etiology of
    Charcot joint is believed to be related to the
    destruction of afferent proprioceptive fibers and
    subsequent unrecognized trauma to the joint.
195
Q
  1. Which of the following statement(s) about
    treatment of diabetic peripheral neuropathic
    pain (DPNP) is (are) true?
    (1) Most of the antidepressants are Food
    and Drug Administration (FDA)
    approved for the treatment of DPNP
    (2) Most therapies for DPNP result in more
    than 90% reduction in pain
    (3) Most of the anticonvulsant drugs are
    FDA approved for the treatment of
    DPNP
    (4) NSAIDs are the most commonly utilized
    medications
A
  1. (D) Across-sectional, community-based survey
    of 255 patients with DPNP recruited through
    the offices of endocrinologists, neurologists,
    anesthesiologists, and primary care physicians
    found that NSAIDs were the most commonly
    used medications, with 46.7% reporting their
    use. This is despite the fact, that there is little
    evidence to support the efficacy of NSAIDs in
    DPNP, and that NSAIDs have a high potential
    for renal impairment in patients with diabetic
    neuropathy.
196
Q
  1. Treatment of painful diabetic neuropathy
    (PDN) rests on modification of the underlying
    disease and control of pain symptoms. In turn,
    the modification of the underlying disease
    includes strict glycemic control. Which of the
    following is (are) true?
    (1) Tight glycemic control can halt or slow
    the progression of distal sensorimotor
    neuropathy
    (2) Hemoglobin A1c target should be
A
  1. (E)
197
Q
  1. The current treatments of the PDN include
    (1) antiepileptic drugs
    (2) antidepressants
    (3) opioids
    (4) aldose reductase inhibitors
A
  1. (A) Given the frequency of imperfect glycemic
    control, attempts have been made to identify
    oral medications which can target downstream
    metabolic consequences of hyperglycemia,
    thereby preventing production of reactive
    oxygen species, which are felt to contribute to
    diabetic neuropathy. Unfortunately, trials of
    aldose reductase inhibitors, which decrease
    aberrant metabolic flux, have been disappointing
    (eg, sorbinil, zopolrestat).
198
Q
  1. The convulsive tic
    (1) is more severe in males
    (2) may indicate the presence of a tumor,
    vascular malformation, or ecstatic
    dilation of the basilar artery
    (3) is because of presence of bilateral facial
    spasms
    (4) is a result of painful periodic unilateral
    facial contractions
A
  1. (B) The combination of trigeminal neuralgia
    and hemifacial spasm is known as convulsive
    tic. It is reported to be more severe in women
    than in men. Occasionally, strong spasms
    involve all of the facial muscles unilaterally
    almost continuously. Seldom, facial weakness
    may be present. Convulsive tic may indicate
    the presence of a tumor, vascular malformation,
    or ecstatic dilation of the basilar artery,
    compressing the trigeminal or facial nerves.
199
Q
  1. Which of the following support(s) the diagnosis
    of idiopathic trigeminal neuralgia?
    (1) Periods of weeks or months without
    pain
    (2) Increase pain by commonly benign
    stimuli, like talking, eating, or washing
    (3) Pain often alleviated by sleep
    (4) Bilateral pain in the distribution of the
    trigeminal nerve, described as shooting
    or lancinating
A
  1. (A) The diagnostic criteria for trigeminal neuralgia
    are: shooting, electric-like, sharp, severe
    pain which last for seconds but sometimes
    experienced together with pain-free intervals.
    The pain is periodic with weeks or months
    without pain. The pain is typically unilateral
    and triggered by light touch, eating, talking,
    or washing.
200
Q
  1. Which of the following is (are) true regarding
    trigeminal neuralgia?
    (1) Very often, trigeminal neuralgia is the
    presenting symptom in patients affected
    with multiple sclerosis
    (2) Trigeminal neuralgia is 20 times more
    common in patients with multiple
    sclerosis
    (3) Trigeminal neuralgia tends to occur in
    the early stages of multiple sclerosis
    (4) Bilateral trigeminal neuralgia is seen
    more often than expected in patients
    with multiple sclerosis
A
  1. (D) Rarely, trigeminal neuralgia is the presenting
    symptom of multiple sclerosis. More often,
    trigeminal neuralgia presents in patients with
    advanced stages of multiple sclerosis.
201
Q
  1. The retrogasserian glycerol injection
    (1) is a selective neurolytic agent with preference
    for sensory fibers, leaving intact
    motor neurons
    (2) recurrence rates are the highest of all
    ablative techniques
    (3) sensory loss is almost unseen in patients
    after this procedure
    (4) sensory loss is less common than with
    radiofrequency thermocoagulation
A
  1. (C) Glycerol is a nonselective neurolytic agent.
    Although less common than with radiofrequency
    thermocoagulation, the frequency of
    patients affected by sensory loss is high.
    Recurrence rate is the highest among all ablative
    techniques.
202
Q
  1. Which of the following is (are) true for trigeminal
    neuralgia?
    (1) Trigeminal neuralgia is the most
    common cranial neuralgia
    (2) It is more common in females
    (3) The highest incidence is in elderly
    patients
    (4) The disease most frequently linked with
    trigeminal neuralgia is multiple sclerosis
A
  1. (E) The trigeminal neuralgia is the most
    common cranial neuralgia and its most frequent
    form is idiopathic. The incidence of
    trigeminal neuralgia is 5.7 per 1000 females
    and 2.5 per 1000 in males. Patients with multiple
    sclerosis have a higher risk for trigeminal
    neuralgia. Other potential relation was found
    in patients with family history.
203
Q
  1. Potential factors involved in the development
    of trigeminal neuralgia include
    (1) ion channel upregulation in the area of
    the trigeminal injury
    (2) focal demyelination
    (3) up to 30% of patients with trigeminal
    neuralgia have arterial cross compression
    at the level root entry zone
    (4) cell body degeneration in the trigeminal
    complex of the mesencephalon
A
  1. (A) The trigeminal neuralgia is a primary axonal
    degenerative disease. The ignition hypothesis
    combines the current knowledge of the role of
    ion channels in the development of neuropathic
    pain. Focal demyelination adjacent to the area of
    arterial compression has been shown by electron
    microscopy in patients undergoing posterior
    fossa surgery. In up to 30% of patients with arterial
    cross compression, there is a groove or an
    area of discoloration lateral where the root entry
    zone would be expected.
204
Q
  1. Spontaneous intracranial hypotension (SIH)
    (1) the most common site of idiopathic
    dural tears is the lower lumbar region
    (2) congenital subarachnoid or Tarlov cysts
    are a potential site for dural weakness
    and rupture
    (3) the most obvious difference between
    PDPH and SIH, is the lack of postural
    symptoms in the second
    (4) are no characteristic findings on MRI
A
  1. (C) SIH and PDPH have similar presentation,
    pathophysiology, and treatment. The most
    important distinction is the initiating event,
    which is obvious in PDPH. MRI of the brain
    with gadolinium enhancement in patients with
    SIH shows meningeal enhancement, and thickening,
    and a possible caudad shift of the brain
    toward the foramen magnum. The most common
    location of spontaneous dural tear is the thoracic
    region followed by the cervicothoracic
    and thoracolumbar junction regions.
205
Q
  1. A common treatment for patients with PDPH is
    epidural blood patch (EBP). Which of the following
    is (are) true regarding this therapy?
    (1) Maintenance of supine position for
    2 hours after the patch provides higher
    chances for success
    (2) As a result of the predominant caudad
    spread of the blood after EBP, a level of
    placement above the suspected dural
    tear is recommended
    (3) The effectiveness of EBP is reduced
    when the dural tear was caused by a
    large-size needle
    (4) The long-term relief of an initial EBP is
    close to 98%
A
  1. (B) It has been shown that keeping a supine
    position for 2 hours after the EBP provides
    higher chances of success when compared with
    30 minutes. Although initial relief is very high
    (close to 100%), the overall long-term relief of PDPH after EBP is between 61% and 75%. The
    effectiveness of EBP is reduced when the dural
    tear was caused by a large-size needle.
206
Q
  1. The incidence of PDPH is between 1% and
    75%. Factor(s) that prevent its development at
    the time of dural puncture include
    (1) use of an interlaminar approach
    (2) use of intrathecal catheter
    (3) bed rest after the puncture
    (4) use of small-gauge spinal needle
A
  1. (C) Two important factors in the prevention of
    PDPH are the use of small and blunt bevel
    spinal needles. Other factors that may prevent
    the development of PDPH include the use of
    paramedian approach (with angles of 35° or
    greater) and the use of intrathecal catheters. Bed
    rest as a preventive measure is not effective.
207
Q
  1. Diagnostic criteria for cervicogenic headache
    by the International Headache Society and the
    International Association for the Study of Pain
    (IASP) include
    (1) unilateral headache
    (2) relief of acute attacks by blocking the
    greater occipital nerve with local
    anesthetic
    (3) aggravation of the headache with neck
    movements
    (4) decrease range of neck motion
A
  1. (E) Cervicogenic headache is defined as
    headache that arises from painful disorders of
    structures in the upper neck, which generates
    irritation of the upper cervical roots or their
    nerve branches. The current classification by
    the IHS and the IASP accepts these headaches
    to be unilateral or bilateral. All the other
    options in the questions are true.
208
Q
  1. The cervicogenic headache
    (1) has a prevalence of 0.4% to 2.5% in the
    general population and may account for
    up to 15% to 20% of patients with
    chronic headache
    (2) is more common in females; a female to
    male ratio of 4 to 1
    (3) mean age is the beginning of the fourth
    decade
    (4) is aggravated by neck movement, and
    alleviated by occipital nerve block
A
  1. (E) According to the International Headache
    Society and the IASP, cervicogenic headache
    (CGH) is a pain originated in the neck, mostly
    unilateral although it may be bilateral, exacerbated
    by neck movement, and alleviated by
    local anesthetic block of the occipital nerve.
    The prevalence of CGH is 0.4% to 2.5% in the
    general population and may account for up to
    15% to 20% of patients with chronic headache,
    more common in females with a 4:1 ratio and
    mean age of patients with 42.9 years.
209
Q
  1. The cortical spreading depression
    (1) may produce the aura symptoms
    (2) produces activation of the trigeminal
    nerve endings
    (3) consist of decreased cerebral blood flow
    spreading forward from the occipital
    cortex
    (4) is followed by generalized cerebral vascular
    dilation that explains the headache
A
  1. (A) During the aura in classic migraine a
    decrease in cerebral blood flow decreases
    spreads from the occipital cortex. The variation
    in the cerebral blood flow causes the aura
    and activates trigeminal nerve endings. It is
    possible that cortical spreading depression may
    stimulate peripheral nerve terminals of the
    nucleus caudalis trigeminalis.
210
Q
  1. In terms of migraine which of the following is
    (are) true?
    (1) Migraine with aura is associated with an
    increase of cerebral blood flow that
    happens after the headache begins
    (2) In migraine with aura there is a
    decrease of cerebral blood flow that
    starts after the headache begins
    (3) In migraine without aura there is no
    change in cerebral blood flow
    (4) In migraine without aura there is
    increase of cerebral blood flow before
    the headache begins
A
  1. (B) During the aura in classic migraine there is
    a decrease in cerebral blood flow.
    In patients with classic migraine (migraine
    with aura), there is increase of cerebral blood
    flow that happens after the headache begins
    and this change persist until the headache
    resolves. In migraine without aura, there is no
    change in cerebral blood flow.
211
Q
  1. Migraine is a risk factor for
    (1) major depression
    (2) manic episodes
    (3) anxiety disorders
    (4) panic disorders
A
  1. (E) Migraine is a risk factor for affective disorders.
    When comparing with nonmigraineurs,
    patients with migraine have a 4.5-fold increased
    risk of major depression, a sixfold risk of manic
    episodes, a threefold increase in anxiety disorder,
    and a sixfold prevalence in panic disorder.
212
Q
  1. Migraine happens in 18% of women, 6% of
    men, and 6% of children. Migraine usually
    (1) begins in the first three decades of life
    (2) is of higher prevalence in the fifth
    decade
    (3) decrease symptoms in the last trimester
    of their pregnancy in most females
    (4) is improved, common after surgical
    menopause
A
  1. (A) Although migraine begins in the first three
    decades of life, the higher prevalence is in the
    fifth one. Family history is a common finding,
    and pregnant females often experience worsening
    symptoms in the first trimester and
    improvement during the third. Many women
    experience improvement of their symptoms
    after natural, but not surgical menopause.
213
Q
  1. Tension-type headache
    (1) is the result of sustained contraction of
    the pericranial muscles with subsequent
    ischemic pain
    (2) has more common onset during adolescence
    and young adulthood
    (3) has increased EMG activity in muscles
    with tenderness
    (4) reduces CNS levels of serotonin that may
    be responsible for abnormal pain
    modulation
A
  1. (C) Muscle tenderness is common in patients
    suffering from TTH, but is not secondary to
    pericranial muscle contraction or ischemic pain
    in response to emotion or stress. Increased
    EMG activity is independent of tenderness and
    pain. Reduced pain threshold observed in
    chronic TTH may be the result of low CNS
    levels of serotonin. Although TTH can begin at
    any age, the most common onset is during adolescence
    and young adulthood. The prevalence
    of TTH decreases with increasing age.
214
Q
  1. For any structure to be deemed a cause of low
    back pain, it must have the following characteristic(s):
    (1) A nerve supply
    (2) Be capable of causing low back pain in
    healthy volunteers
    (3) Be susceptible to disease or injuries
    known to be painful
    (4) Be shown to be a source of pain in a
    patient using diagnostic techniques of
    known reliability and validity
A
  1. (E) For any structure to be considered as a
    source for low back pain it must have the following
    characteristics: a nerve supply, the capability
    of causing low back pain similar to what
    is seen clinically (ideally in healthy volunteers),
    a susceptibility to disease or injuries known to
    be painful, and should be able to be shown as
    a source of pain using diagnostic techniques
    of known reliability and validity
215
Q
  1. Randomized controlled trials (RCTs) have generated
    evidence-based conclusions for preventive
    interventions back and neck pain. Which
    of the following statement(s) is (are) true based
    on the evidence of RCTs?
    (1) Lumbar supports are not effective in
    preventing neck and back pain
    (2) Exercise may be effective in preventing
    neck and back pain
    (3) Back schools are not effective in
    preventing back and neck pain
    (4) Ergonomic interventions are effective in
    preventing back and neck pain
A
  1. (A) There have been RCTs showing that lumbar
    supports and back schools are not effective in
    preventing back pain. Exercise has been proven
    by RCTs to prevent back pain. To date, there are
    no RCTs on the effectiveness of ergonomics in
    preventing back pain.
216
Q
  1. Anomalies of lumbar nerve roots include
    which of the following?
    (1) Two pairs of nerve roots arise from a
    single dural sleeve
    (2) A dural sleeve arises from a lower position
    in the dural sac
    (3) The vertebral foramen is unoccupied by
    a nerve or contains a supernumerary set
    of roots
    (4) Extradural anastomoses between roots
    in which a bundle of nerve fibers leaves
    one dural sleeve to enter an adjacent one
A
  1. (E) The most significant anomalies of the
    lumbar nerve roots are aberrant courses and
    anastomoses between nerve roots. Type 1
    anomalies are aberrant courses of which there
    are two kinds. Type 1A describes two pairs of nerve roots arising from a single dural sleeve,
    whereas type 1B defines a dural sleeve arising
    from a low position on the dural sac. Type 2
    anomalies include those in which the number
    of roots in the intervertebral foramen varies.
    An empty foramen is classified as type 2A, and
    a foramen with extra nerve roots is known as a
    type 2B. Type 3 anomalies are those involving
    extradural anastomoses between roots in which
    a bundle of nerves leaves on dural sleeve to
    enter one nearby. Type 3 anomalies may coexist
    with type 2 anomalies.
217
Q
  1. Low back pain is defined as pain perceived
    within a region bounded
    (1) superiorly by an imaginary line through
    the T12 spinous process
    (2) inferiorly by a transverse line through
    the posterior sacrococcygeal joints
    (3) laterally by the lateral borders of the
    erector spinae
    (4) within the region overlying the sacrum
A
  1. (E) The IASP published standardized terms to
    define low back pain as pain perceived to arise
    for lumbar spinal pain and/or sacral spinal
    pain. Lumbar spinal pain is defined as pain
    perceived to arise from the region bordered
    superiorly by an imaginary line through the
    T12 spinous process, inferiorly by a line
    through the S1 spinous process, and laterally
    by the lateral borders of the erector spinae.
    Sacral spinal pain is that defined as pain perceived
    to arise from the region bordered laterally
    by imaginary vertical lines through the
    posterior superior and posterior inferior iliac
    spines, superiorly by a transverse line through
    the S1 spinous process, and inferiorly by a
    transverse line through the posterior sacrococcygeal
    joints.
218
Q
553. Complications of cervical transforaminal injections
include which of the following?
(1) Cerebellar infarction
(2) Cerebral infarction
(3) Spinal cord infarction
(4) Anterior spinal artery syndrome
A
  1. (E) Transforaminal injections have been the cause
    of some of the most worrisome recent complications.
    These included cerebellar and cerebral
    infarct, SCI, and infarction, massive cerebral
    edema, paraplegia, visual defects with occlusion
    following particulate depo-corticosteroids, anterior
    spinal artery syndrome, persistent neurologic
    deficits, transient quadriplegia, cauda
    equina syndrome, subdural hematoma, and
    paraplegia following intracordal injection during
    attempted epidural anesthesia under general
    anesthesia.
219
Q
  1. Discographic stimulation (formally known as
    discography) is considered positive if
    (1) adjacent disc stimulation causes pain
    (2) thermal stimulation with a wire electrode
    causes pain
    (3) pain is reproduced at pressures greater
    than 80 psi
    (4) pain is reproduced at pressures less than
    50 psi and preferably less than 15 psi
A
  1. (C) Despite its controversial history, disc stimulation
    (formerly known as discography)
    remains the only means by which to determine
    whether or not a disc is painful. The test is positive
    if upon stimulating a disc the patient’s
    pain is reproduced provided that stimulation of adjacent discs does not reproduce their pain.
    Discs are also considered to be symptomatic
    only if pain is reproduced at injection pressures
    less than 50 psi and preferably less than 15 psi.
    At injection pressures greater than 80 psi, some
    discs are painful in normal individuals. The
    stimulation of discs has been complemented
    by another approach, heating a wire electrode
    that has been inserted into a disc annulus.
    Heating a disc evokes pain that is perceived in
    the back. This pain may also radiate to the
    lower extremities and be responsible for
    referred pain in the thigh and leg.
220
Q
  1. The use of chemonucleolysis for lumbar disc herniations
    is indicated for which of the following?
    (1) Contained disc protrusions
    (2) Extruded disc herniations
    (3) Herniations unresponsive to nonsurgical
    management
    (4) Sequestered disc herniations
A
  1. (B) Chemonucleolysis is indicated for contained
    disc protrusions causing sciatic pain that
    have been unresponsive to conservative management.
    The injection is contraindicated for
    extruded and sequestered disc herniations, and
    in patients with cauda equine syndrome.
    Relative contraindications include previous
    chymopapain injections, previous surgery for
    lumbar disc herniation, spinal stenosis, severe
    degenerative disc or facet osteoarthritis, and
    spondylolisthesis.
221
Q
  1. Causes of FBSS include which of the following?
    (1) Inappropriate selection of patients
    (2) Irreversible neural injury
    (3) Inadequate surgery
    (4) New injury to nerves and spine
A
  1. (E) Inappropriate or premature selection of
    patients for surgery is the most common cause
    of FBSS. The second most common cause is
    persistence of pain secondary to irreversible
    neural injury. A less common cause is inadequate
    surgery. Lastly, a variant of FBSS results
    from new pathologic processes initiated by the
    initial surgery.
222
Q
557. Selection criteria for elective lumbosacral spine
surgery include
(1) radicular pain with corresponding
dermatomal segmental sensory loss
(2) abnormal imaging study showing nerve
root compression
(3) signs of segmental instability consistent
with symptoms
(4) success of conservative therapy
A
  1. (A) The American Association of Neurological
    Surgeons and the American Academy of
    Orthopedic Surgeons have published criteria
    for patient selection for elective lumbosacral
    spine surgery. They are applicable to new
    patients, as well as FBSS patients. They include
    the following:
  2. Failure of conservative therapy.
  3. An abnormal diagnostic imaging study showing
    nerve root or cauda equina compression
    and/or signs of segmental instability consistent
    with the patient’s signs/symptoms.
    Radicular pain with one or more of the
    following: (a) corresponding dermatomal segmental sensory loss, (b) corresponding
    dermatomal motor loss, (c) abnormal deep
    tendon reflexes consistent with appropriate
    dermatomes.
223
Q
558. The main types of cervical involvement in
rheumatoid arthritis include
(1) atlantoaxial subluxation
(2) cranial settling
(3) subaxial subluxation
(4) occipital condyle fractures
A
  1. (A) There are three main types of cervical
    spine involvement in rheumatoid arthritis:
    atlantoaxial subluxation, cranial settling, and
    subaxial subluxation. The inflammatory
    changes affecting synovial joints and bursae
    target structures lined with a synovial membrane
    in the cervical spine. Patients with cervical
    spine involvement are thought to have a
    more severe form of rheumatoid arthritis, and
    their prognosis is usually worse. Occipital
    condylar fractures result from a full-energy
    blunt trauma complemented with axial compression,
    lateral bending, or rotational injury
    to the alar ligament.
224
Q
  1. Whiplash and whiplash-associated disorders
    (WAD) comprise a range of injuries to the neck
    caused by or related to a sudden distortion of
    the neck. Characteristics include
    (1) spinal cord injury (SCI)
    (2) referred shoulder pain
    (3) sensory deficits
    (4) headaches
A
  1. (E) The Québec Task Force (QTF) was a task
    force sponsored by the Société de l’assurance
    automobile du Québec, the public auto
    insurer in Quebec, Canada. In 1995, the QTF
    submitted a report on WADs which made
    specific recommendations on prevention, diagnosis,
    and treatment of WAD. These recommendations
    have become the base for Guideline on
    the Management of Claims Involving Whiplash-
    Associated, a guide to classifying WAD and
    guidelines on managing the disorder. The full
    report titled Redefining “Whiplash” was published
    in the April 15, 1995 issue of Spine. An
    update was published in January 2001.
    Four grades of WAD were defined by the
    QTF :
    Grade 1: Complaints of neck pain, stiffness or
    tenderness only but no physical signs are
    noted by the examining physician.
    Grade 2: Neck complaints and the examining
    physician finds decreased range of motion
    and point tenderness in the neck.
    Grade 3: Decreased range of motion plus neurologic
    signs, such as decreased deep tendon
    reflexes, weakness, insomnia, and sensory
    deficits.
    Grade 4: Neck complaints and fracture or dislocation,
    or injury to the spinal cord.
225
Q
  1. Distraction testing allows an examiner to identify
    neurologic and mechanical abnormalities in
    the cervical spine. It is characterized by
    (1) relief of neck pain
    (2) lifting head from the chin and occiput
    (3) relief of pressure on zygapophyseal joints
    (4) examiner standing in front of a standing
    patient
A
  1. (A) The distraction test is performed with an
    examiner standing behind a seated patient, lifting
    their head from the chin and occiput, and
    removing the weight of the head from the neck.
    If relief of neck pain occurs, the test might point
    to foraminal intrusion on a nerve root as the
    source of pain.
226
Q
  1. A 45-year-old male with complaints of cervical
    neck pain radiating down his left arm is examined
    by a physician. With one particular maneuver,
    his pain is exactly reproduced. The test(s)
    that can reproduce his symptoms include
    (1) distraction testing
    (2) Valsalva test
    (3) spurling maneuver
    (4) Adson test
A
  1. (A) Spurling maneuver is used to identify
    nerve root compression or irritation. The head
    is tilted toward the affected side and manual
    pressure is applied to the top of the head.
    Radicular pain should be reproduced with
    this maneuver. Valsalva test allows a patient to
    experience painful or sensory changes when
    bearing down. The test increases intrathecal
    pressure and exacerbates compression within
    the cervical canal caused by tumors, infections,
    disc herniations, or osteophyte changes.
    The distraction test is performed with an
    examiner standing behind a seated patient,
    lifting their head from the chin and occiput,
    and removing the weight of the head from the
    neck. If relief of neck pain occurs, the test
    might point to foraminal intrusion on a nerve
    root as the source of pain. Adson test is used
    to assess vascular compromise because of subclavian
    artery impingement from thoracic
    outlet syndrome.
227
Q
  1. The following are true about HIV infection–
    related neuropathies, EXCEPT
    (1) inflammatory demyelinating polyneuropathies
    occur early in the course of
    HIV infection
    (2) vasculitis-related neuropathies occur
    midcourse in HIV infection
    (3) distal sensory neuropathies occur late in
    HIV infection
    (4) HIV-related neuropathies tend to be
    nonspecific to the stage of HIV infection
A
  1. (D) Peripheral neuropathic pain syndromes in
    patients with HIV infection tend to be specific
    to the stage of HIV infection as outlined in
    answers (1) through (3).
228
Q
  1. Which of the following is (are) true about the
    predominantly sensory neuropathy of AIDS?
    (1) The predominant symptom is pain in
    the soles of the feet
    (2) Ankle jerks are often absent or reduced
    (3) As symptoms of the neuropathy progress,
    they usually remain confined to the feet
    (4) EMG demonstrate sensory, but not
    motor involvement
A
  1. (A) In patients with predominantly sensory
    neuropathy of AIDS, the complaints are
    mostly sensory. However, the NCV and EMG
    studies demonstrate both sensory and motor
    involvement.
229
Q
  1. Which of the following group(s) of medication(
    s) is (are) useful in treatment of pain in
    HIV and AIDS patients?
    (1) Opioids
    (2) Anticonvulsants
    (3) Psychostimulants
    (4) Antidepressants
A
  1. (E) Psychostimulants, such as dextroamphetamine,
    methylphenidate, may be useful agents
    in patients with HIV infection or AIDS who
    are cognitively impaired. Psychostimulants
    enhance the analgesic effects of the opioid
    drugs. They are also useful in diminishing
    sedation secondary to opioids. In addition, psychostimulants
    improve appetite, promote sense of well-being, and improve feelings of weakness
    and fatigue in patients with malignancies.
230
Q
  1. Pathophysiologic tissue injury in SCD generates
    multiple pain mediators. The facilitators of
    the pain transmission include
    (1) bradykinin
    (2) serotonin
    (3) substance P
    (4) dynorphin
A
  1. (B) Tissue injury generates several major pain
    mediators, including, but not limited to IL-1,
    bradykinin, K+, H+, histamine, substance P, and
    CGRP. The pathway for painful stimuli is subject
    not only to activators, sensitizers, and facilitators
    but also to inhibitors. Serotonin,
    enkephalin, β-endorphin, and dynorphin are
    endogenous central pain inhibitors.
231
Q
566. Which of the following established four components
of SCD is responsible/associated with
the patient’s pain?
(1) Anemia and its sequelae
(2) Organ failure
(3) Comorbid conditions
(4) Pain syndromes
A
  1. (E) SCD is a quadrumvirate of: (1) pain syndromes,
    (2) anemia and its sequelae, (3) organ
    failure, including infection, and (4) comorbid
    conditions. Pain, however, is the insignia of
    SCD and dominates its clinical picture throughout
    the life of the patients. Pain may precipitate
    or be itself precipitated by the other three components
    of the quadrumvirate.
232
Q
  1. Which of the following statement(s) is (are)
    true about avascular necrosis (AVN) and SCD?
    (1) Core decompression is an effective treatment
    of late stages of the AVN
    (2) Treatment of AVN is mostly
    symptomatic
    (3) AVN affects mostly femoral head
    (4) AVN is the most common complication
    of SCD in adults
A
  1. (C) Avascular necrosis is the most commonly
    observed complication of SCD in adults.
    Although it tends to be most severe and disabling
    in the hip area, it is a generalized bone
    disorder in that the femoral and humeral heads
    and the vertebral bodies may be equally
    affected. Treatment of avascular necrosis is
    symptomatic and includes providing nonopioid
    or opioid analgesics in the early stages of
    the illness; advanced forms of the disease
    require total joint replacement. Core decompression
    appears to be effective in the management
    of avascular necrosis if performed
    during its early stages
233
Q
568. Which of the following statement(s) is (are)
true about leg ulceration and SCD?
(1) Leg ulcers occur in 5% to 10% of the
adult SCD patients
(2) Skin grafting is a very effective treatment
for chronic leg ulcers in SCD
patients
(3) Many leg ulcers heal within a few
months with good localized treatment
(4) Regranex, used to treat leg ulcers, contains
an autologous platelet-derived
growth factor
A
  1. (B) Leg ulceration is a painful and sometimes
    disabling complication of sickle cell anemia
    that occurs in 5% to 10% of adult patients with
    SCD. With good localized treatment, many
    ulcers heal within a few months. Leg ulcers
    that persist beyond 6 months may require skin
    grafting, although results of this treatment have
    been disappointing. Recent advances in management
    include the use of platelet-derived
    growth factor, prepared either autologously
    (Procuren) or by recombinant technology
    (Regranex).
234
Q
  1. Management of painful vasoocclusive crises in
    SCD patients frequently employs supplemental
    oxygen. Which of the following is (are) true
    about the supplemental oxygen administration
    in SCD patients?
    (1) Supplemental low-flow oxygen is often
    given to patients with SCD painful crisis
    in efforts to diminish the number of
    reversibly sickled cells
    (2) There is little supportive data for the
    use of supplemental oxygen in SCD
    patients
    (3) Routine oxygen administration in the
    absence of hypoxemia may impair reticulocytosis
    in SCD patients
    (4) Routine oxygen administration in the
    absence of hypoxemia has no proven
    benefit in SCD patients
A
  1. (E)
235
Q
  1. Which of the following statement(s) is (are)
    true about epidemiology of SCD?
    (1) It is the most common hemoglobinopathy
    in the United States
    (2) The prevalence is significantly higher in
    the African American population than
    in the general population
    (3) It occurs in 0.3% to 1.3% of the African
    American population
    (4) The prevalence of SCD does not
    depend on the ethnic background of
    the population
A

.570. (A) Sickle cell anemia affects millions throughout
the world. It is particularly common among
people whose ancestors come from sub-
Saharan Africa; Spanish-speaking regions
(South America, Cuba, Central America); Saudi
Arabia; India; and Mediterranean countries
such as Turkey, Greece, and Italy

236
Q
571. Which of the following is (are) the measure(s)
used to treat vasoocclusive crises of SCD
during pregnancy?
(1) Aggressive hydration
(2) Supplemental oxygen in patients with
hypoxemia
(3) Partial exchange transfusions
(4) Prophylactic transfusions
A

.571. (E) General management of vasoocclusive crisis
during pregnancy begins with aggressive
hydration to increase intravascular volume and
decrease blood viscosity. Supplemental oxygen
is essential in those patients with hypoxemia.
Partial exchange transfusions are used to reduce
polymerized hemoglobin S. Prophylactic transfusions
may reduce the incidence of severe sickling
complications during pregnancy.

237
Q
  1. When an opioid-tolerant patient in a sickle cell
    vasoocclusive crisis is admitted to a hospital,
    which of the following step(s) should be taken?
    (1) A baseline opioid infusion should be
    started immediately at an equianalgesic
    dose to patient’s home opioid requirement
    (2) A baseline infusion should be supplemented
    with a patient-controlled
    analgesia (PCA) on demand for breakthrough
    pain
    (3) As patient’s new opioid requirement
    becomes known from the PCA history,
    conversion to a combination of longand
    immediate-release opioid can be
    undertaken
    (4) Fast opioid dose increases may lead to
    hypoxemia and/or hypercarbia, which
    may exacerbate sickling of erythrocytes
A
  1. (E) Some patients with SCD are opioid-tolerant
    secondary to their home opioid management.
    Therefore, the home opioid requirement should
    be taken into account for faster and more efficient
    pain control of sickle crisis pain. Opioid
    titration, however, may require some additional
    care because hypoxemia and hypercarbia
    further exacerbate sickling of erythrocytes.
238
Q
  1. Autonomic dysreflexia
    (1) is usually triggered by a spontaneous
    sympathetic discharge above the SCI
    level
    (2) is rarely associated with headache
    (3) is never life-threatening
    (4) manifests itself with increased blood
    pressure
A
  1. (D) Autonomic dysreflexia is a potential lifethreatening
    condition, which is triggered by
    sensory input below the lesion and manifests
    itself with increased blood pressure, headache,
    and a risk of cerebral hemorrhage and seizure.
239
Q
  1. Which of the following is (are) true about the
    anterior cord syndrome?
    (1) It is characterized by complete sensory
    loss
    (2) Prognosis for motor function recovery is
    very poor
    (3) It is a complete SCI syndrome
    (4) It is characterized by complete motor
    function loss
A
  1. (C) Anterior cord syndrome is a common
    incomplete cord syndrome. A patient with
    anterior cord syndrome may exhibit complete
    motor and incomplete sensory loss, with the
    exception of retained trunk and lower extremity
    deep pressure sensation and proprioception.
    This syndrome carries the worst prognosis
    for return of function, and only a 10% chance of
    functional motor recovery has been reported.
240
Q
  1. Which of the following is (are) true about the
    posterior cord syndrome?
    (1) It is characterized by preservation of
    temperature sensation
    (2) It is characterized by preservation of
    normal gait
    (3) It is uncommon
    (4) It is characterized by preservation of
    proprioception
A
  1. (B) Posterior cord syndrome is a rare incomplete
    cord syndrome consisting of loss of the
    sensations of deep pressure and deep pain and
    proprioception, with otherwise normal cord
    function. The patient ambulates with a footslapping
    gait similar to that of someone
    afflicted with tabes dorsalis.
241
Q
576. Which of the following is (are) true feature(s) of
Brown-Séquard SCI syndrome?
(1) Ipsilateral motor deficit
(2) Contralateral pain sensation deficit
(3) Contralateral temperature sensation
deficit
(4) Uncommonness
A
  1. (E) Brown-Séquard syndrome is an uncommon
    incomplete spinal cord syndrome. It is anatomically
    a unilateral cord injury, such as a missile
    injury. It is clinically characterized by a motor
    deficit ipsilateral to the SCI in combination
    with contralateral pain and temperature hypesthesia.
    Almost all these patients show partial
    recovery, and most regain bowel and bladder
    function and the ability to ambulate.
242
Q
  1. Anticonvulsants are commonly used for the
    treatment of neuropathic pain in the SCI patients.
    Which of the following correctly describe(s) their
    pharmacologic actions?
    (1) Modulation of calcium channels
    (2) Modulation of sodium channels
    (3) Increase of GABA inhibition
    (4) Blockade of reuptake of norepinephrine
A
  1. (A) Anticonvulsants have several pharmacologic
    actions, such as modulation of sodium
    and calcium channels, increasing GABA inhibition,
    and suppressing abnormal neuronal
    hyperexcitability, which suggest an effect in
    neuropathic pain
243
Q
  1. Which of the following is (are) the usual symptom(
    s) of the autonomic dysreflexia?
    (1) Dramatic rise in blood pressure
    (2) Flushing and sweating in areas above
    the SCI
    (3) Marked reduction in peripheral blood
    flow
    (4) Decline in heart rate
A
  1. (E) Patient with autonomic dysreflexia usually
    exhibit decline in heart rate, dramatic changes
    in blood pressure, flushing and sweating above
    the level of the injury, and a marked reduction
    on peripheral blood flow through the reflex
    pathways in the preserved vagus nerves.
244
Q
  1. Heterotopic ossification (HO) is commonly seen
    in patients with traumatic brain injury (TBI),
    cerebral vascular accident, burns, trauma, total
    joint arthroplasty, and SCI. Which of the following
    is (are) true about HO in SCI patients?
    (1) It is always painful
    (2) Hip is the most commonly affected
    (3) It is defined as ossification inside the
    joint capsule
    (4) Osteoclast inhibitors are use for both
    treatment and prophylaxis of HO
A
  1. (C) HO is the formation of mature, lamellar
    bone in nonskeletal tissue, usually occurring
    in soft tissue surrounding joints. The bone formation
    in HO differs from other disorders of
    calcium deposition in that HO results in encapsulated
    bone between muscle planes, not intraarticular
    or connected to periosteum.
    In neurogenic HO secondary to TBI or
    SCI, the hip is the most common joint affected.
    Even though the most common symptom
    of HO is pain, it may be painless in patients
    with complete SCI.
    Etidronate disodium, a bisphosphonate, is
    an osteoclast inhibitor. It is structurally similar
    to inorganic pyrophosphate and is shown
    to delay the aggregation of apatite crystals
    into large, calcified clusters in patients with
    TBI and SCI. The recommended prophylactic
    treatment for HO in SCI is 20 mg/kg/d for
    2 weeks, then 10 mg/kg/d for 10 weeks. The
    current treatment recommendation for established
    HO is 300 mg IV daily for 3 days followed
    by 20 mg/kg/d for 6 months in spinal
    cord patients.
245
Q
580. CRPS in the initial stages may be associated
with
(1) neurogenic inflammation
(2) higher local levels of tumor necrosis factor
alpha
(3) high systemic CGRP levels
(4) Increase in protein concentration in
fluid of affected joints
A
  1. (E) Localized neurogenic inflammation may
    explain the acute edema, vasodilation, and
    sweating observed in early stages of CRPS.
    Increased protein concentration and synovial
    hypervascularization is observed in the intraarticular
    fluid of affected joints. Findings that
    support the role of neurogenic inflammation
    in the generation of CRPS include elevated systemic
    levels of CGRP and local increase of
    IL-6 and tumor necrosis factor alpha in artificially
    produced blisters.
246
Q
  1. Which of the following is true about motor
    abnormalities in CRPS?
    (1) Dystonia of the hand or affected foot
    occurs in about 30% of the patients in
    the acute stages
    (2) Decrease active range of motion and
    increase amplitude of physiological
    tremor is seen in about 50% of the
    patients
    (3) They are likely related to an abnormal
    peripheral process
    (4) They may be explained by abnormalities
    in the cerebral motor processing
A
  1. (C) Up to 50% of CRPS patients show decrease
    range of motion, increase amplitude of physiologic
    tremor, and reduce active motor force,
    with dystonia of the affected limb observed in
    only 10% of the chronic cases. Those motor
    changes are unlikely associated with a peripheral
    process and more likely the result of
    changes of activity in the motor neurons which
    point to abnormalities of cerebral motor processing.
247
Q
582. In terms of CRPS which of the following is (are)
true?
(1) Incidence of CRPS is 20% after brain
lesion
(2) Affected extremities after brain injury
are at higher risk of developing CRPS
than unaffected
(3) CRPS following SCI is frequent
(4) Upper extremities are more commonly
affected than lower extremities
A
  1. (C) It is estimated that the risk of CRPS after
    fractures is 1% to 2% and 12% after brain
    lesions. Retrospective studies in large cohorts
    shows a distribution in the upper and lower
    extremity from 1:1 to 2:1. CRPS following SCI
    are rare. Affected extremities after brain injury
    are more likely affected than unaffected ones.
248
Q
  1. Which of the following is true regarding bone
    scintigraphy?
    (1) The three stages of the three-phase bone
    scan include the perfusion, blood-pool,
    and mineralization phases
    (2) Homogeneous unilateral hyperperfusion
    in the perfusion phase is consistent
    with CRPS
    (3) Homogeneous unilateral hyperperfusion
    in the blood-pool phase is consistent
    with CRPS
    (4) Patients with CRPS show increase unilateral
    periarticular trace uptake in the
    mineralization phase
A
  1. (E) The three stages of the three-phase bone scan
    include the perfusion phase 30 seconds postinjection,
    the blood-pool is 2 minutes postinjection,
    and mineralization phases is evaluated
    3 hours postinjection. Homogeneous unilateral
    hyperperfusion in the perfusion and blood-pool
    phase is consistent with CRPS and excludes the
    differential diagnosis of osteoporosis because of
    inactivity. The mineralization phase in patients
    with CRPS shows elevated unilateral periarticular
    uptake.
249
Q
584. Which of the following is true regarding C fiber
impulses?
(1) After sensitization, antidromic impulses
to peripheral C fiber terminals release
vasoactive substance
(2) Neurally released substances trigger
neurogenic inflammation
(3) Neurogenic inflammation includes
axonal reflex, vasodilation, and plasma
extravasation
(4) C fiber activation peripherally releases
CGRP and substance P
A
  1. (E)
250
Q
585. The major peripheral pathologic finding(s) in
patients with CRPS is (are)
(1) patch atrophy of some muscle cells
(2) capillary microangiopathy
(3) Wallerian degeneration
(4) generalized osteopenia
A
  1. (A) The major peripheral pathologic findings in
    CRPS patients include (a) patchy atrophy of
    some muscle cells, secondary to disuse and
    nerve damage; (b) capillary microangiopathy,
    with accelerated turnover of endothelial cells and pericytes; (c) Wallerian degeneration of
    several types of axons; and (d) focal osteopenia
    in the territory innervated by a damaged nerve,
    and synovial cell disorganization and edema.
251
Q
586. Which of the following is (are) true regarding
CRPS?
(1) Medical procedures are the second most
common cause of CRPS
(2) Decrease deep tendon reflexes are a
result of muscle atrophy
(3) Cutaneous dynamic mechanical
allodynia is a hallmark of central
sensitization
(4) Hypoesthesia may be rarely seen in
patients with CRPS
A
  1. (B) Medical procedures are the second most
    common cause of CRPS. The finding of exaggerated
    deep tendon reflexes in CRPS patients
    has been attributed to cortical disinhibition.
    Focal deficit of touch (hypoesthesia) were present
    in 50% of patients. Brushing skin activates
    low threshold mechanoreceptors which under
    normal circumstances has no connections with
    central pain neurons. Brush evoked pain (cutaneous
    dynamic mechanical allodynia)is a hallmark
    of central sensitization.
252
Q
  1. Characteristics of CRPS I in pediatrics include
    (1) CRPS I is more common in girls
    (2) the lower extremity is more often
    affected
    (3) CRPS may have genetic predisposition
    (4) CRPS is more common in Hispanics
A
  1. (A) In contrast to CRPS II which has similar
    frequencies in boys and girls, CRPS I is more
    common in girls with a ratio of 4:1. The lower
    extremity is more affected (5:1 ratio). CRPS I is
    more common in Caucasians. There is also evidence
    that CRPS may have a genetic predisposition,
    with increase incidence in patients with
    HLAA3, B7, and DR2.
253
Q
  1. Characteristics of CRPS II in pediatrics include
    (1) the incidence is similar in boys and girls
    (2) brachial plexus injury during delivery
    commonly leads to chronic pain
    (3) Erb palsy do not generally develop
    CRPS
    (4) patients with Erb palsy need a comprehensive
    treatment to avoid the development
    of CRPS II
A
  1. (B) The gender distribution of CRPS II in the
    pediatric population is roughly similar in
    boys and girls. Even though patients with
    brachial plexus injury during delivery (Erb
    palsy) is common and can lead to prolonged
    motor weakness, they rarely develop pain.
    Interestingly most of these patients do better
    without treatment.
254
Q
  1. To confirm the diagnosis of CRPS:
    (1) There are no laboratory tests to confirm
    the diagnosis
    (2) Disturbed vascular scintigraphy is necessary
    to make the diagnosis of CRPS
    (3) Bone scan is nonspecific for the diagnosis
    of CRPS
    (4) There is no utility in ordering bone scan
    in patients with CRPS
A
  1. (B) The diagnosis of CRPS remains a clinical
    decision based on findings in the history and
    physical examination. There are no laboratory
    tests that can absolutely confirm or exclude
    the diagnosis. Although controversial, most
    of the authors find that bone scans are quite
    nonspecific for the diagnosis of CRPS. Patients
    with a clinical diagnosis of CRPS may have
    bone scans showing hypofixation or hyperfixation
    or may be normal. The primary utility
    of the bone scan could be in ruling out
    some underlying orthopedic abnormality that
    might be triggering neurovascular changes
    that may confused the findings with those of
    CRPS.
255
Q
  1. Which of the following is true regarding movement
    disorders in CRPS patients?
    (1) Motor dysfunction is the result of
    voluntary defensive response to protect
    the limb from painful stimuli
    (2) Deep tendon reflexes in these patients
    are normal to brisk
    (3) Movement disorders often happen in
    early stages of the disease
    (4) Akinesia is a prominent finding in CRPS
    patients
A
  1. (C) Movement disorders are an essential feature
    of patients with CRPS. Motor dysfunction
    is not simply a voluntary defensive response
    to protect the limb from painful stimuli, but
    may represent the interaction of peripheral and
    central mechanisms. Deep tendon reflexes are
    often brisk. The prevalence of movement
    disorders increases with the duration of the disease.
    One characteristic form of these movement
    disorders is the presence of inability to
    start a movement (akinesia).
256
Q
  1. In terms of CRPS and dystonia, which is characterized
    by involuntary contractions of one
    or more muscles, it can be said that
    (1) dystonia is a prominent feature of CRPS
    (2) dystonia in patients with CRPS typically
    presents with flexure postures
    (3) tonic dystonia often spares the first two
    digits
    (4) extensor postures occur early in the
    development of dystonia
A
  1. (A) Dystonia in CRPS patients causes twisting
    movements or abnormal postures of the
    affected body parts. The most prominent motor
    feature is flexor postures (tonic dystonia) of the
    fingers, feet, and wrist. Extensor postures occur
    but are rare.
257
Q
  1. Supraspinal regulatory mechanisms that may
    explain some of the features of CRPS include
    (1) spread of cortical representation of the
    affected limb
    (2) patients with generalized dystonia have
    increased intracortical excitability to
    sensory stimuli
    (3) motor cortical disinhibition
    (4) early increase activity of the thalamus
    contralateral to the affected limb
A
  1. (E) Supraspinal mechanisms play a major role
    in the abnormal sensory perception of patients
    with CRPS. Segmental dystonia is characterized
    by spread of the cortical representation of
    the affected extremity and its corresponding
    synaptic connections to adjacent cortical areas.
    On the other hand, generalized dystonia have
    increased intracortical excitability to sensory
    stimuli, and motor cortex disinhibition has
    been confirmed in CRPS I. PET scan and SPET
    have shown increased activity of the contralateral
    thalamus in patients in early stages of
    CRPS and hypoperfusion in advanced stages.
258
Q
593. Which of the following is (are) myofascial
trigger point characteristic(s)?
(1) Weakness with muscle atrophy
(2) Referral of pain to a distant site upon
activation of the trigger point
(3) Range of motion not restricted
(4) Autonomic phenomenon, such as
piloerection or changes in local
circulation (regional blood flow and
limb temperature) in response to trigger
point activation
A
  1. (C) Myofascial trigger point characteristics:
    • Focal severe tenderness in a taut band of
    muscle
    • Referral of pain to a distant site upon activation
    of the trigger point
    • Contraction of the taut band (local twitch
    response) upon mechanical activation of
    the trigger point
    • Reproduction of the pain by mechanical
    activation of trigger point
    • Restriction of range of motion
    • Weakness without muscle atrophy
    • Autonomic phenomenon such as piloerection
    or changes in local circulation (regional
    blood flow and limb temperature) in
    response to trigger point activation
    Individual features of the trigger point are
    differentially represented in different muscles.
    An examiner should not expect to find each
    feature of the trigger point in every muscle by
    physical examination.
259
Q
  1. The definitive goal of treatment of persons with
    myofascial pain syndrome is (are)
    (1) restoration of function through inactivation
    of the trigger point
    (2) restoration of normal tissue mobility
    (3) relief of pain
    (4) increased range of motion
A
  1. (A) Inactivation of the trigger point is a means
    to achieve pain relief, to improve biomechanical
    function, and then to improve the ability of
    the patient to better perform whatever tasks
    have been selected as goals. Relief (not elimination)
    of pain or increased range of motion,
    both of which can be the result of trigger point
    inactivation, are not in themselves the goals of
    treatment.
260
Q
  1. Inactivation of the myofascial trigger point can
    be accomplished
    (1) manually
    (2) by direct injection of a local anesthetic
    into the muscle
    (3) by dry needle intramuscular stimulation
    of the myofascial trigger point
    (4) by correcting structural mechanical
    stressors
A
  1. (E) While chronic myofascial pain syndrome
    is best treated with a multidisciplinary team
    approach including the patient, physicians,
    psychologists, clinical social workers, occupational
    therapists, physical therapists, ergonomists,
    massage therapists, and others actively
    involved in patient care; patients with acute
    myofascial pain syndrome may only require
    treatment by physicians and physical therapists.
    Too frequently, patients with chronic
    myofascial pain are started too soon on isotonic
    training and conditioning, causing further
    aggravation of active trigger points and an
    increase in pain and dysfunction.
    The acute treatment plan may be divided
    into a pain-control phase and a training or conditioning
    phase. During the pain-control phase,
    the most essential component is inactivation of
    the trigger point. Patients must change their
    behaviors and avoid overstressing their muscles
    without becoming excessively inactive. The
    pain-control phase must have a definitive endpoint.
    If patients do not move beyond the paincontrol
    phase to the conditioning phase, patients
    can be restricted in their functional abilities and
    be at greater risk of reinjury. The training or
    conditioning phase follows and it involves therapeutic
    exercises, movement reeducation, and
    overall conditioning
261
Q
596. With regard to trigger point injections, botulinum
toxin which of the following is true?
(1) Has been tried unsuccessfully in
myofascial trigger point inactivation
(2) Can cause a flulike myalgia
(3) Occasionally causes weakness that is
confined to the area of injection
(4) Is a long-lasting trigger point injection
capable of about a 3 month inactivation
of the trigger point
A
  1. (C) Botulinum toxin has been tried successfully
    in myofascial trigger point inactivation; however,
    it can cause a flu-like myalgia that lasts
    days to a week and sporadically causes weakness beyond the area of injection. It functions as a
    long-lasting trigger point injection that can provide
    up to 3 months of relief in contrast to the
    days to 1-week effect of traditional trigger point
    injection with local or no anesthetic.
262
Q
  1. A 40-year-old female with chronic myofascial
    neck pain wants to go to an acupuncturist. She
    should know that
    (1) in one study it was found that shallow
    needling reduced the pain of chronic
    myofascial neck pain
    (2) in a randomized, double-blind, shamcontrolled
    study, acupuncture was
    found to be superior to dry needling in
    improving range of motion
    (3) in a randomized, double-blind, shamcontrolled
    study, acupuncture was found
    to be better than placebo when treating
    trigger points in chronic neck pain
    (4) in a randomized, double-blind, controlled
    study, acupuncture was found to
    be better than control only when it was
    followed by transcutaneous electrical
    stimulation
A
  1. (A)
  2. Japanese acupuncture or shallow needling
    reduced the pain of chronic myofascial
    neck pain in one study.
  3. and 3. These have been found to be true.
  4. While this study does not exist, a technique
    of dry needling called intramuscular stimulation
    does exist. It involves the insertion of
    the needle into the taut band without necessarily
    considering the actual trigger
    point. It may be combined with electrical
    stimulation delivered through the needle
    (percutaneous electroneutral stimulation).
263
Q
  1. A 40-year-old woman comes to the pain clinic
    for initial evaluation. After a thorough history
    and physical examination, the patient is diagnosed
    with fibromyalgia. Which symptom(s)
    would support your diagnosis of fibromyalgia
    as opposed to myofascial pain syndrome?
    (1) Widespread pain
    (2) Irritable bowel syndrome
    (3) Distal paresthesias
    (4) Occipital headaches
A
  1. (E)
264
Q
  1. Which of the following is (are) true of
    fibromyalgia?
    (1) Adult women are twice as likely to be
    affected as adult men
    (2) Prevalence peaks in the fourth decade of
    life
    (3) Many children diagnosed with
    fibromyalgia will have worsening of
    their symptoms as they reach adulthood
    (4) Affects all ethnic groups
A
  1. (D) Fibromyalgia has been found among all
    cultures throughout the world with an incidence
    of 2% to 12% of the population. In adulthood,
    women are affected four to seven times
    as often as men. The frequency of fibromyalgia
    increases with age and peaks in the seventh
    decade of life. In childhood, boys and girls are
    affected equally. In contrast to adults, children’s
    symptoms may resolve with age.
265
Q
600. Risk factors for the development of fibromyalgia
syndrome include
(1) physical trauma
(2) febrile illness
(3) family history of fibromyalgia
(4) history of sexual abuse
A
  1. (A) Although research is ongoing, the development
    of fibromyalgia appears to be increased
    if the patient has had a febrile illness, a history
    of physical trauma, or a family history of
    fibromyalgia syndrome. Approximately onethird
    of patients with fibromyalgia report that
    another member of their family has previously
    been diagnosed with fibromyalgia
266
Q
  1. Sleep disturbances are common in patients
    with fibromyalgia. Difficulties the patient may
    encounter include
    (1) problems initiating sleep
    (2) awakening in the middle of the night
    (3) light, unrefreshing sleep
    (4) difficulty napping throughout the day
A
  1. (E) Greater than 90% of patients with fibromyalgia
    suffer from chronic insomnia. Some patients
    may have problems falling asleep. Other may
    awaken a few hours after going to sleep and feel
    alert, thus disrupting their sleep throughout the
    remainder of the night. After a night of sleep,
    patients with fibromyalgia may feel stiff, tired,
    and “cognitively sluggish.” These patients also encounter difficulty napping throughout the
    day. Patients with fibromyalgia have disrupted
    sleep architecture with alpha wave intrusions in deep, delta wave sleep.
267
Q
  1. Pathophysiologically, fibromyalgia
    (1) is a disorder of abnormal processing of
    sensory information within the CNS
    (2) exhibits a narrow array of recognized
    objective physiological and biologic
    abnormalities
    (3) patients demonstrate abnormally low
    regional cerebral blood flow in thalamic
    nuclei and other pain-processing brain
    structures that is inversely correlated
    with spinal fluid substance P levels
    (4) demonstrates abnormal spinal cord
    windup
A
  1. (E)
  2. In fibromyalgia patients, CT has revealed
    unusually low cerebral blood flow in the
    thalamic nuclei, the left and right heads of
    the caudate nucleus, and the cortex that
    correlates with spinal fluid substance P
    levels.
  3. Windup is a frequency-dependent increase in
    the excitability of spinal cord neurons, evoked
    by electrical stimulation of afferent C fibers.
    Glutamate (NMDA) and tachykinin NK1
    receptors are required to generate windup
    and therefore a positive modulation between
    these two receptor types has been suggested.
    Whatever the mechanisms involved in its
    generation, windup has been interpreted as a
    system for the amplification in the spinal
    cord of the nociceptive message that arrives
    from peripheral nociceptors connected to C
    fibers. This probably reflects the physiological
    system activated in the spinal cord after an
    intense or persistent barrage of afferent
    nociceptive impulses. On the other hand,
    windup, central sensitization and hyperalgesia
    are not the same phenomena, although
    they may share common properties. Spinal
    cord windup is abnormal in fibromyalgia
    syndrome.
    Physical trauma or a fever/infection may
    be provisionally related to the onset of
    fibromyalgia in over 60% of cases.
268
Q
  1. That same patient (who happens to be a neurobiology
    graduate student) starts to ask about
    the role that cytokines play in fibromyalgia.
    You tell her that
    (1) IL-8 has been found to be significantly
    higher in the serum of fibromyalgia
    patients, especially in depressed patients
    (2) IL-6 was not found to be increased in
    the blood of fibromyalgia patients
    (3) the production of IL-8 in vitro is stimulated
    by substance P
    (4) cytokines do not play a role in the
    pathogenesis of fibromyalgia
A
  1. (B) The levels of serum IL-8 were higher in
    fibromyalgia patients, and IL-6 was statistically
    higher in cultures of fibromyalgia peripheral
    blood mononuclear cells compared with in controls.
    The IL-8 increase was most dramatic in
    depressed patients, but there was also a correlation
    with the duration of fibromyalgia and
    the pain intensity. The production of IL-8 in
    vitro is stimulated by substance P.
269
Q
  1. Which of the following medications are FDA
    approved for the treatment of fibromyalgia?
    (1) Cyclobenzaprine
    (2) Duloxetine
    (3) Tramadol
    (4) Pregabalin
A
  1. (D) Pregabalin has the potential to raise the
    threshold for pain fiber depolarization. It is a ligand for the α2δ subunit of voltage-dependent
    calcium channel receptors, which has analgesic,
    anxiolytic-like, and anticonvulsant activity. It
    decreases the release of numerous neuropeptides,
    including noradrenaline, glutamate, and
    substance P. It has already been approved for
    treating partial seizures, pain following the
    rash of shingles and pain associated with diabetes
    nerve damage (diabetic neuropathy). Two
    double-blind, controlled clinical trials, involving
    about 1800 patients, support approval for
    use in treating fibromyalgia with doses of 300
    or 450 mg/d. It is effective in reducing the
    severity of body pain, improving quality of
    sleep, and reducing fatigue in fibromyalgia
    patients. Pregabalin was approved by the FDA
    for use in fibromyalgia patients on June 21,
    2007.
    The most common side effects of pregabalin
    include mild to moderate dizziness and
    sleepiness. Blurred vision, weight gain, dry
    mouth, and swelling of the hands and feet
    also were reported in clinical trials. The side
    effects appeared to be dose-related. Pregabalin
    can impair motor function and cause problems
    with concentration and attention. The
    FDA advises that patients talk to their doctor
    or other health care professional about whether
    use of pregabalin may impair their ability to
    drive.
  2. Cyclobenzaprine, have been used in the
    past with success in fibromyalgia patients,
    they have not been approved by the FDA.
  3. Duloxetine is an FDA-approved treatment for
    major depression, neuropathic pain from diabetic
    peripheral neuropathy, and generalized
    anxiety disorder. The drug is a serotonin and
    norepinephrine reuptake inhibitor that
    exhibits nearly equal serotonin and noradrenaline
    reuptake inhibition. Atrial of duloxetine
    for patients with chronic pain and/or major
    depression indicated that for the fibromyalgia
    patients, 80% of the observed effect on pain is
    a direct analgesic effect rather than an indirect
    antidepressant effect. Common adverse
    events were: nausea, headache, dry mouth,
    insomnia, constipation, dizziness, fatigue,
    somnolence, diarrhea, and hyperhidrosis.
    Two placebo-controlled randomized studies
    on the treatment of fibromyalgia-associated
    pain with duloxetine have been published.
    Both studies demonstrated that duloxetine
    treatment improved fibromyalgia-associated
    pain in women. However, the medication
    has not yet been approved for the treatment
    of fibromyalgia. Another type of serotonin
    and noradrenaline reuptake inhibitor is represented
    by milnacipran, where noradrenaline
    reuptake inhibition is favored over that
    of serotonin. Some reports state that milnacipran
    may also be effective in treating
    fibromyalgia body pain.
  4. Tramadol has only recently been shown to
    improve the pain of patients with fibromyalgia.
    It combines weak μ-agonist with
    NMDA antagonist and noradrenaline and
    serotonin reuptake inhibition. In the combination
    preparation that comes with acetaminophen,
    a considerable synergy has been
    noticed. Nausea and dizziness can be limiting
    at first in about 20% of patients, but starting
    with just one tablet at bedtime for 1 to
    2 weeks can decrease the prevalence and
    allow later increases but about one tablet
    every 4 days to full therapeutic levels. Atypical
    tramadol regimen for fibromyalgia is
    300 to 400 mg/d in three or four divided
    dosages, concomitant with acetaminophen
    at 2 to 3 g/d in divided doses.
    Administering 5-hydroxytryptophan can
    augment the synthesis of serotonin. Onehundred
    milligrams, orally, three times daily
    has been shown to be an effective dose in
    treating fibromyalgia.
270
Q
  1. The pain in fibromyalgia is, at least in part,
    mediated by central sensitization. Studies have
    shown that
    (1) dextromethorphan and ketamine may
    improve pain and allodynia in
    fibromyalgia patients
    (2) the majority of patients with fibromyalgia
    that tried ketamine benefited
    (3) ketamine’s efficacy was limited because
    of its side effects
    (4) dextromethorphan had a similar sideeffect
    profile
A
  1. (B) Central sensitization can be inhibited by
    NMDAreceptor blockade. Two NMDA receptor
    antagonists, ketamine and dextromethorphan
    (an oral preparation) have been found to
    exhibit favorable effects on pain and allodynia
    in fibromyalgia patients. With ketamine, 50% of
    patients benefited. Fibromyalgia subgroups of
    responders and nonresponders were perpetuated
    by these findings because all the
    fibromyalgia patients in the study were otherwise
    comparable. Ketamine’s effectiveness was
    limited because of its frequently occurring
    psychotropic side effects, such as feelings of unreality, altered body image perception,
    aggression, anxiety, nausea, dizziness, and
    modulation of hearing and vision.
    Dextromethorphan has a better side-effect
    profile than ketamine. It was administered
    with tramadol to increase the antinociceptive
    effect, to hold adverse effects low, and to
    decrease the development of opioid tolerance.
    A good response was obtained in 58% of the
    fibromyalgia patients who tried this regimen.
    It may be a consideration for the patients who
    respond positively to IV ketamine.